Search for notes by fellow students, in your own course and all over the country.

Browse our notes for titles which look like what you need, you can preview any of the notes via a sample of the contents. After you're happy these are the notes you're after simply pop them into your shopping cart.

My Basket

You have nothing in your shopping cart yet.

Title: SAT - Barrons Writing Book
Description: The best SAT Barrons Writing book on the market, with detailed concepts explained totally to give out that 800 on the Writing Section

Document Preview

Extracts from the notes are below, to see the PDF you'll receive please use the links above


© Copyright 2012, 2009 by Barron’s Educational Series, Inc
...
,
under the title Writing Workbook for the New SAT
...

No part of this book may be reproduced in any form or distributed by
any means without the written permission of the copyright owner
...

250 Wireless Boulevard
Hauppauge, New York 11788
www
...
com
e-ISBN: 978-1-4380-8388-9
First e-Book publication: August, 2012

Greetings from the Author

Hello and welcome to the world of the SAT Writing Test
...

If you’re unsure what the test is all about, read Part I
...
To fine-tune your test-taking skills, go to Parts II and VI, where you’ll find practice exams just
like those on the SAT
...

The book’s chapters follow the format of the SAT
...
In short, my book is a
complete guide to the SAT Writing Test
...
But even if the test is tomorrow, the book can still give you a quick fix on what to
expect when you open your exam booklet
...
Use it regularly and often
...
(The SAT gives you exactly twenty-five minutes
...
Review the
concepts of English grammar, usage, and rhetoric on which you’ll be tested
...
Take the practice tests at the
back of the book, and watch your test scores soar—I hope
...
Now it’s time for you to start climbing
...


Contents

Greetings from the Author

PART I
The Basics: Getting Acquainted with the Writing Test
Overview of the Writing Test
The Essay
Multiple-Choice Questions

PART II
Dress Rehearsal: A Sample Test
Sample Test
Answer Key
Performance Evaluation Chart
Conversion Table
Answer Explanations

PART III
How to Write an Essay in 1,500 Seconds
Steps for Writing the Perfect Essay
Prewriting: Getting Set to Write
Reading and Analyzing the Topic Carefully
Narrowing the Topic
Choosing a Main Idea
Gathering and Arranging Ideas Purposefully
Composing: Putting Words on Paper
Writing a Gripping Introduction
Building an Essay with Paragraphs
Paragraph Development
Transitions
Using Plain and Precise Language
Fresh Language and Surprises
Varying Sentence Structure
Sentence Types
Repetition of Ideas
Short and Long Sentences
Ending Your Essay
Editing and Proofreading: The Final Touches

Editing for Clarity
Editing for Interest
Checking for Standard Usage and Mechanics
Review
Answer Key to Practice Exercises

PART IV
You Be the Ump: Essays for Evaluation
How Essays Are Judged and Graded
Guidelines for Evaluation
Essays for Evaluation
Essay Topics for Practice

PART V
The Heart of the Test: Multiple-Choice Questions
Introduction
Improving Sentences Questions
Sample Sentence Improvement Questions
Problems in Style and Expression
Problems in Sentence Structure
Problems in Standard Usage
Review
Identifying Sentence Errors
Errors in Expression and Style
Errors in Grammar and Usage
Improving Paragraphs Questions
Answering the Questions
How to Answer Improving Paragraphs Questions
Review
Answer Key to Practice Exercises

PART VI
Tests for Practice, Practice, Practice
Practice Test A
Answer Key
Performance Evaluation Chart
Conversion Table
Answer Explanations
Practice Test B
Answer Key
Performance Evaluation Chart

Conversion Table
Answer Explanations
Practice Test C
Answer Key
Performance Evaluation Chart
Conversion Table
Answer Explanations
Practice Test D
Answer Key
Performance Evaluation Chart
Conversion Table
Answer Explanations

PART I
THE BASICS: GETTING ACQUAINTED WITH THE WRITING TEST
• A Preview of the Test
• How Colleges Use the Test
• Format of Test Questions
• How the Essay Is Scored
• To Guess or Not to Guess on Short-Answer Questions
• How to Find the Best Answers

Overview of the Writing Test
The entire SAT lasts three hours and forty-five minutes, including one hour for the Writing Test
...
During that
time you will be asked to write an essay in response to a given topic
...
It consists of three types of multiple-choice
questions that ask you to (1) correct poorly written sentences, (2) find grammar and usage errors in a
set of sentences, and (3) revise an early draft of a given essay
...
It contains additional
multiple-choice questions on correcting poorly written sentences
...


FORMAT OF THE SAT

NOTE: Every SAT also contains a tenth section that doesn’t count in calculating your score
...
The experimental section is not
identified
...
Please adjust accordingly
...
It shows
admission officials how well you write, especially how well you write under the pressure of time
...
In
particular, your essay provides evidence of
The depth of your thinking
...
Your response also shows whether you can devise a thesis, or main idea,
and develop it insightfully
...
You show your ability to organize ideas by arranging material
according to a logical, sensible plan
...
You reveal your ability to express yourself by accurately and
succinctly conveying your thoughts to the reader
...
You demonstrate your use of standard written English
by writing an essay relatively free of errors in grammar and usage
...
Instead of asking you about obscure matters of grammar, the
questions will ask you to identify common sentence errors and to improve sentences and paragraphs
...
A high score may entitle you to waive a
freshman composition course
...
To understand just
how your score will affect you, consult the literature of the colleges to which you are applying
...


HOW THE TEST IS SCORED
Your essay will be read by two experienced evaluators, most likely high school or college teachers
trained to judge the overall quality and effectiveness of students’ essays
...
Nor will they know your name or the name of your
school
...
Your essay’s
subscore will be recorded as the sum of the two scores (2 to 12)
...
25) for each wrong answer
...
A machine will score your responses to forty-nine questions and will report a subscore on a

scale of 20 to 80
...
Your total for the Writing Test, along with your scores in math and critical reading, will be
reported to you, to your guidance counselor, and to the admissions offices of the colleges you
designate
...
If you haven’t a clue about how to answer a question, leave it blank
...
The odds are one in four that you’ll be
right
...
If you guess right just once, you’ll earn a point and lose three-quarters of a
point, a net gain of one quarter
...
Yes, it’s a gamble
because you could make four incorrect guesses, but the chances of losing every time are only one in
four
...


TIP
If a question stumps you completely, don’t try to answer it
...
Testing experts and psychologists agree that
there’s a better than average chance of success if you trust your intuition
...

Another piece of folk wisdom about guessing is that if one answer is longer than the others, that
may be your best choice
...
In fact, since economy of
expression is a virtue in writing, a shorter choice may more often be the best answer
...


HOW TO PREPARE
By reading these words you’ve already begun preparing for the exam
...

But that was then
...

Once you have finished reading these introductory pages, take the diagnostic test in Part II
...
By doing so, you can tell not
only how much studying you need to do but what material to study
...
Or, if you missed a couple of multiple-choice questions related to
pronoun choice or parallel structure, study the relevant pages in Part V, and do the practice exercises
...
Even professional journalists,
accustomed to working under the pressure of deadlines, would be hardpressed to produce a good
essay in twenty-five minutes
...
If you’ve been a reasonably proficient essay writer in the past, be confident that you’ll
perform equally well on the SAT
...


TIP
The goal of your essay: To organize your thoughts and express them clearly,
interestingly, and correctly
...
Chances are you’ve done it
before
...
The numerous inclass essays you’ve produced over the years have no doubt trained you for the kind of instant essay
asked of you on the SAT
...
That’s not true on the SAT
...
Because you don’t know the topic, you must
quickly process your thoughts and get them onto paper
...
The verb
essay, in fact, means to assess thoughtfully: not on the SAT, however
...
If you manage to come up with one or more profound
insights, more power to you, but keep in mind that the objective of an SAT essay is more mundane—
to show colleges that you can organize your thoughts and express them clearly, interestingly, and
correctly
...
What you need to know is
already lodged inside you
...
It is not a measure of what you know but rather a demonstation of what you
can do
...
You
must first think critically about the issue presented in the essay assignment, forming your particular
perspective on the topic
...
An essay completed in twenty-five minutes is bound to be shorter than most
essays required in high school or college courses
...
But it represents what you can do during the initial phase of the writing
process, and twenty-five minutes should give you enough time to prove that you have what it takes to
write a respectable first draft
...
Although no one can predict the
subject of the prompt, you can be sure that the directions for writing the essay will always say
something like:
Think carefully about the issue presented in the following quotation and the assignment below:
“People rarely stand up and speak out for what they truly believe, particularly when their views
will not be popular among their peers
...

Assignment: Do people generally avoid expressing unpopular beliefs? Plan and write an essay in
which you develop your point of view on this issue
...


Based on these instructions, you must write an essay explaining whether you accept or reject the
proposition that people seldom express beliefs that differ from those of the majority
...
On the other hand, an essay that adopts a
contrary point of view would develop the idea that people in general prefer to have their voices heard
even when they disagree with prevailing beliefs and values
...

What you say in the essay is completely up to you
...
You won’t
be penalized for an unusual or unpopular point of view unless it is based on a faulty premise or pure
fantasy
...
Concentrate on expressing
your thoughts coherently and correctly
...
If the evidence you provide wanders from the main idea or raises additional issues that you
don’t have time to discuss, the effect of the essay will be diluted
...

Once you have decided on your position on the issue, develop your thoughts clearly and
effectively
...
In short, you may
use facts, statistics, common sense, historical background—anything, really, to demonstrate that your
opinion is grounded on something more solid than a feeling or a personal preference
...

The assignment urges you to plan your essay before you begin to write
...
Whether you
prefer to write lists of ideas or just think about an approach before committing words to paper, on the
SAT you must write an essay—not a play, not a poem, not a short story, not dialogue, not a fable, just
an essay
...
Variations are possible, but twenty-five
minutes doesn’t give you much time to be inventive
...


The directions for writing the essay don’t tell you how long it should be
...
Just remember that quantity counts less than quality
...
Two paragraphs might do, but three
or more suggest that you have the capacity to probe pretty deeply into the subject
...
Three, in fact, may be preferable to two, although that’s a generalization that
doesn’t apply to every essay
...
) In the end, the number of
paragraphs is less important than the substance of each paragraph
...

A plain, natural writing style is probably best
...
Readers will be turned off by formal, pompous, or
overblown prose
...

As SAT day draws near, review these suggestions for writing an essay
...


Multiple-Choice Questions
Two separate sections of the SAT (25 minutes and 10 minutes) give you a total of forty-nine multiplechoice questions that deal with errors in grammar, usage, word choice (diction), and expression
(idiom)
...
Improving Sentences (25 questions)
2
...
Improving Paragraphs (6 questions)
Of the three types, Identifying Sentence Errors are the briefest—rarely more than two or three
lines
...
The Improving Sentences
questions take a bit longer because they require more reading, and the Improving Paragraphs
questions take longer still because they relate to problems embedded in the text of an essay that you
are given to read
...
The test has been carefully calibrated to
coincide with the time allotted, provided you work steadily
...
Don’t assume, therefore, that question 7 will be
harder than question 6, or 14 harder than 13
...
If you come to a question that
baffles you, don’t agonize over it
...
Paragraphimprovement questions are arranged differently: They follow the progress of the passage
...
Try different techniques while taking practice exams to find the one that works best for you
...
In each question, part of a sentence—or sometimes the whole sentence—is
underlined
...
Your task is to choose the best one
...
In any case, never choose an alternative that substantially changes the meaning of
the original sentence, even if its grammar and style are perfect
...
The custom of awarding huge scholarships to college athletes have gotten out of hand
...
That is, a singular subject must have a singular verb, and a plural subject must be
accompanied by a plural verb
...

Choice A uses have, a plural verb that fails to agree with custom, a singular subject
...

In choice D, both the subject, custom, and the verb, is, are singular, but the sentence contains an
extremely awkward phrase, “out of hand by which
...

2
...


(A) Both of my cousins who live in San Francisco
(B) Both of my two cousins living in San Francisco
(C) My two cousins, who lives in San Francisco
(D) My two cousins in San Francisco
(E) My two San Francisco cousins of mine
Explanation: Because sentences cluttered with unnecessary words are less effective than tightly
written sentences, one of your tasks while answering Improving Sentences questions is to recognize
unnecessary and redundant words and phrases
...

Choice A is grammatically correct, although it could be stylistically improved by eliminating the
repetition of the word both
...

Choice C is more economical, but it contains the singular verb, lives, that disagrees with its
plural subject, cousins
...


How to Find Answers to Sentence Improvement Questions
• Read the entire sentence, paying close attention to its meaning
...

• Try to hear the sentence in your head
...

• Search for wordiness and awkward expression in the underscored segment of the sentence
...

• Eliminate all choices that contain obvious errors
...
(See Part V for details about
precisely what to look for
...

• If no change is needed, mark A on your answer sheet
...


of the ceremony, the new members
handshake
...


that

would never

the secret

2
...


Your job is to read each sentence carefully and identify the item that contains an error
...

Sometimes a sentence may contain no error, in which case the correct answer will be E (No error)
...

The verb swear doesn’t adhere to the usual pattern of verbs—that is, creating the past tense by adding
-ed to the present tense, as in walk/walked or love/loved
...

Knowing about irregular verbs could have led you to the right answer
...
In other words, your language “ear” may have told you that something was amiss
...

The correct answer to question 2 is C because the underlined word is a double negative
...
Therefore, a phrase containing both words constitutes an error in
standard usage
...


How to Find Answers to Identifying Sentence Errors Questions
• Read the whole sentence
...

• Focus your attention on awkward sounding words and phrases
...
(Likely errors are discussed fully in Part V
...

• If all the underscored words are correct, mark E on your answer sheet
...
The questions are not meant to stump or trick you
...

Questions are based on an unedited draft of a student’s essay
...
Expect to answer one or two questions about problems of
grammar and usage
...


Sample Questions
The excerpt that follows is part of an essay written in response to the topic: Preserving the
Environment—Everybody’s Job
...
[2] And with the process of the earth
aging, we must keep recycling our waste products
...
[4] Those who don’t do it should be criticized
greatly
...
” [6] Not
everyone does, even though recycling is an effective place to start
...
[8] Also, join the masses of people who will
no longer buy ordinary drinking water sold in plastic bottles
...
[10] There are separate bins for paper, glass, and plastic
...
[12] It is so easy to drive a few blocks to a
center to drop off what needs to be recycled
...
[14] Anyone who cannot see its simplicity should be criticized
for not doing their part to help make the world a better place
...
” [17] Such people are ignorant
and deserve to be taught a lesson about how wastefulness is slowly destroying the earth
...
Considering the essay as a whole, which of the following best explains the main purpose of the
second paragraph?
(A) To explain the historical background of the topic
(B) To provide a smooth transition between the first and third paragraphs
(C) To define terms introduced in the first paragraph
(D) To develop an idea presented in the first paragraph
(E) To present a different point of view on the issue being discussed
Explanation: To answer this question, you must read the whole essay
...

All the choices name legitimate uses of paragraphs, but only choice D applies to this essay because
it develops by example an idea originating in the first paragraph—how easy it is to recycle
...
Choice B is a possibility because in a unified essay each
paragraph, aside from the opening and closing paragraphs, in some way serves as a bridge between
paragraphs
...

2
...
[10] There are separate bins for
paper, glass, and plastic
...

(A) Recycling centers offer recyclers convenience by providing separate bins for paper, glass, and

plastic and by being located in almost every town
...

(C) Almost every town has a recycling center with separate bins for paper, glass, and plastic, and
this is a convenience for recyclers
...

(E) For the convenience of recyclers in almost every town, paper, glass, and plastic are separated
into provided bins at its recycling center
...
You probably know that in a series of short sentences each idea carries
equal weight
...
To answer this question, then, you must decide which idea expressed by the
three sentences deserves to be given the greatest emphasis
...
Because sentences 10 and 11 refer to the convenient arrangement of recycling bins, they
are more important to the development of the paragraph than sentence 9, about the location of
recycling centers
...
Knowing that, read each of the
choices
...

Choice D stresses the location rather than the convenient arrangement of bins in recycling centers
...
Therefore, choice B is the best answer
...


How to Find Answers to Improving Paragraphs Questions
• Read the entire essay, paying attention to its main idea and to the writer ’s purpose
...

• Carefully read each question and the five choices
...
Also discard
choices that contain flaws in grammar and usage
...
)
• As you answer the questions, keep in mind the main idea of each paragraph and the point of the
whole essay
...
)

A WORD OF ENCOURAGEMENT
The multiple-choice sections of the SAT Writing Test pertain to matters of grammar, usage, and
rhetoric typically taught in English classes
...
This book, after all, is a thorough test-prep guide that explains
virtually everything you need to know for the test and describes how you can earn a score to make

you proud
...
Good luck!
Please Note: All directions which appear for the Diagnostic and Practice Tests are similar to those
on the actual exam
...


This e-Book contains hyperlinks that help you navigate through content, bring you to helpful
resources, and allow you to click between questions and answers
...

You’ll also begin to identify your strengths and weaknesses as a writer
...
The Writing Test resumes in Section 4 or 5 of the SAT with thirty-five multiplechoice questions and then, after still more math and reading questions, concludes with fourteen
additional multiple-choice questions on writing
...

Despite this difference, try to simulate actual test conditions as you administer this test to yourself
...

Use a timer, a watch, or a clock to time each section
...

Don’t skip ahead to the next section before the allotted time is up
...

Write the essay on separate pieces paper no larger than 8½×11 inches, the size of an official
SAT essay response sheet
...
An SAT essay written in ink will
be scored “zero
...

When you’ve completed the test, check your answers with the Answer Key and fill in the Performance
Evaluation Chart
...
The chart will also tell
you the types of questions you answered most successfully
...
On second thought, read all the explanations
...

Note that each question is rated by its level of difficulty—EASY, MEDIUM, or HARD
...
A
“medium” rating suggests that more than 65 percent will answer the question correctly
...

In an ideal world everyone will answer all easy, medium, and hard questions correctly
...
The SAT is a test, after all, that is meant to differentiate students
from each other
...
The answer explanations will direct you to
relevant pages to study
...
Let the essay
cool for a while—maybe a day or more
...

Rate your essay using the Self-Scoring Guide
...

Finally, convert your raw scores into the SAT’s 200–800 scaled score
...


Are you ready to begin? Good luck!

Sample Test
SECTION 1
ESSAY
TIME: 25 MINUTES
Directions: Plan and write an essay in response to the assigned topic
...
Present your
thoughts logically and precisely
...
A plain, natural writing style is probably best
...
(See Part III of this book for tips on writing first-rate essays
...
You’ll have enough space if you write
on every line and avoid wide margins
...

BE SURE TO WRITE ONLY ON THE ASSIGNED TOPIC
...

If you finish in less than twenty-five minutes, check your work
...

See Practice Test A
...

Whenever Social Studies teacher Karen Greene sits down to grade a stack of papers, she
wonders whether the grades convey useful information about student learning to the students
themselves, to parents, counselors, or even to colleges
...
Should Karen reward
high grades to diligent, hard-working students with very low skills and limited achievement? Or
should she risk discouraging such students by giving them the Ds that their work really
deserves? What about grading students capable of doing excellent work when they put their
mind to it but who rarely bother? An F for lack of effort might prod them to try harder, but
would it accurately reflect the real quality of their work?
Adapted from Lisa Birk, Harvard Education Letter, October 2004

Assignment: Should students who work very hard in a course earn very high grades, or should
achievement rather than effort determine students’ grades? Plan and write an essay in which you
develop your point of view on this issue
...


Section 1
ESSAY
Time allowed: 25 minutes
Limit your essay to two pages
...
Write only inside the box
...

Do not proceed to Section 2 until the allotted time
for Section 1 has passed
...
Read each sentence carefully and
identify which of the five alternate versions most effectively and correctly expresses the meaning
of the underlined material
...
Choice A always repeats the original
...

EXAMPLE
ANSWER


My old Aunt Maud loves to cook, and
eating also
...
The Broadway tradition of “musical comedy” has changed over the years because of their
stories and music exploring serious themes, especially the consequences of war, tyranny, and
death
...
Mr
...

(A) has been the first who turned
(B) had been the first who turned
(C) was the first having turned
(D) was the first to turn
(E) having been the first to turn
3
...

(A) too recklessly, without sufficient planning behind it
(B) too reckless, without sufficient planning behind it
(C) too recklessly, without sufficient planning behind them
(D) too reckless, and there is not sufficient planning behind them
(E) too recklessly, and there is not sufficient planning behind it
4
...

(A) online, it results from not knowing much about computers and being a bit scared of it
(B) online resulting from not knowing much about computers and being a bit scared of it
(C) online for the reason being that they don’t know much about computers and are a bit scared
of them
(D) online because of knowing little about computers and being scared of it
(E) online because they know little about computers and are a bit scared of them
5
...

(A) were cut from the school budget in spite of them being regarded
(B) was cut from the school budget in spite of them being regarded
(C) was cut from the school budget in spite of their regard as being
(D) were cut from the school budget in spite of regarding it
(E) was cut from the school budget in spite of it being regarded
6
...

(A) humankind’s concept of the heavens has changed
(B) humankind’s concept of the heavens have changed
(C) there has been changes in humankind’s concepts of the heavens
(D) humankind have undergone a change in its concepts of the heavens
(E) humankind has had changes in their concept of the heavens
7
...

(A) Although whales can grow
(B) Whereas whales can grow
(C) Despite a whale growing
(D) While a whale’s size can grow
(E) Since a whale can grow
8
...

(A) When the baseball rulebook is followed too closely, they often spoil instead of enhancing
the game

(B) When the baseball rulebook is followed too closely, it often spoils instead of enhancing the
game
(C) The baseball rulebook, if too closely followed, often spoils the game, not enhancing it
(D) The baseball rulebook, if followed too closely, often spoils rather than enhances the game
(E) If you follow too closely the baseball rulebook, it often spoils rather than enhances the
game
9
...

(A) detectors, but there are not frequent body searches of passengers
(B) detectors, and a bodily search of passengers is not frequent
(C) detectors but are rarely subject to body searches
(D) detectors, but the searching of their bodies is rare
(E) detectors, but the search of bodies is rare among them
10
...

(A) as much school spirit as any other school
(B) so much school spirit as any other school
(C) spirit like any school
(D) as much school spirit as those of any other school
(E) as much spirit if not moreso than that of any school
11
...

(A) there was a swimming pool lined with Italian marble
(B) a swimming pool lined with Italian marble
(C) lined with Italian marble was a swimming pool
(D) the swimming pool was lined with Italian marble
(E) a swimming pool with Italian marble was there

Identifying Sentence Errors
Directions: The underlined and lettered parts of each sentence below may contain an error in
grammar, usage, word choice (diction), or expression (idiom)
...
Indicate your choice by filling in the corresponding
space on the answer sheet
...
Some sentences may contain
no error
...

EXAMPLE
Jill went
ANSWER

to the

of the hill in a

faster time

her friend, Jack
...


12
...

13
...

14
...


15
...

16
...


17
...


found a high

why Paulie, one of the

players, is

colleges
...

19
...
The professor



the realistic painter

reality,

the

impressionistic or abstract painter reveals a personal or emotional response
21
...


the legislation

in Yellowstone and other national parks
...
An examination of
in the

current economic statistics



the sales tax



24
...

25
...


the most notorious gem

have suffered one misfortune after another
...


a story as

Lady Gaga and Beyoncé

the people in
popular singers, but Beyoncé has the

number of fans
...
During this past year, a rapport
call
29
...

easily draw a distinction between a so-called literary classic and an escapist piece

of pulp fiction by considering

the books

to think



Improving Paragraphs
Directions: The passage below is the draft of a student’s essay
...
Read the passage and answer the questions that follow
...
Choose the answer that best follows the requirements of standard
written English
...

[1] No student of American history can avoid having learned about a great technological feat, the
building of the transcontinental railroad in the middle of the nineteenth century
...
[3] In
1869, the dream became a reality
...
[5] In accomplishing this
great feat of engineering, the workers were exploited, and many of them died
...
[7] They performed unskilled labor and
also the highly specialized and dangerous jobs as well
...
[9]
Methods were unsafe and cost lives
...
[11] If they were not pulled fast enough, or the rope broke, they fell to their
deaths
...

[13] With the help of the U
...
government, which contributed millions of dollars’ worth of public
land and funds for construction, the project was run by a group of four well-to-do but corrupt
businessmen from California
...

30
...

(A) Make no changes
...

(C) Change “having learned” to “learning
...

(E) Delete the comma and insert “which was
...
In context, which of the following is the best way to revise the underlined words in order to
combine sentences 4 and 5?
It was a great physical achievement
...

(A) It was a great physical achievement, in accomplishing this great feat of engineering
(B) The fact is that
(C) But students don’t learn that this great physical feat of engineering came at a large price
because
(D) As a result, historians say that this great achievement meant that
(E) Although building the railroad was a great physical achievement,
32
...
Which of the following best describes the relationship between sentences 9 and 10?
(A) Sentence 10 provides material that illustrates the statement made in sentence 9
...

(C) Sentence 10 introduces sources of information that confirms the truth of sentence 9
...


(E) Sentence 10 restates opinions expressed in sentence 9
...
Which of the following would be the best sentence to insert before sentence 13 to introduce the
last paragraph?
(A) Building the railroad was such an expensive undertaking that no private individual of that
era could afford to finance the whole thing
...

(C) One set of construction crews started building from the east to the west, while another
began in the west and built eastward
...

(E) The Pacific Railroad Act, a document rushed through Congress, was grossly overgenerous in its benefits to the builders
...
What material is the most appropriate to add immediately after sentence 14?
(A) How the four business tycoons happened to meet and form a partnership
(B) The facts that convinced the four men to build the railroad
(C) Reasons why shoddy construction methods were used
(D) Details about unethical business practices during the construction of the railroad
(E) An account of how the eastbound and westbound tracks met in Utah in 1869

End of Section 2
...
Do not proceed to Section 3 until the allotted time for Section 2 has
passed
...
Read each sentence carefully and
identify which of the five alternate versions most effectively and correctly expresses the meaning
of the underlined material
...
Choice A always repeats the original
...

1
...

(A) Tony showed three college acceptance letters to his counselor, he
(B) Three college acceptance letters, which were shown to his counselor by Tony, who
(C) Three college acceptance letters were shown by Tony to his counselor, then he
(D) After showing three college acceptance letters to his counselor, Tony
(E) Tony, having shown three college acceptance letters to his counselor, he
2
...
Atkins took over as principal of the
school; these problems diminished both the reputation and the performance of the school
...
Despite being called “reality” television, the program about the plane crash in the Rockies
seemed about as real as a cow jumping over the moon
...
To think that only money motivates people to choose a career in professional athletics is wrong
because in sports many people do it to find personal satisfaction
(A) wrong because in sports many people do it to find personal satisfaction
(B) wrong because sports would have had an effect on finding personal satisfaction

(C) wrong, and the reason is because of the finding of personal satisfaction from a career in
sports
(D) wrong, because many athletes find personal satisfaction out of sports
(E) wrong because many athletes find personal satisfaction in their sport
5
...

(A) he gave military secrets to the enemy
(B) giving military secrets to the enemy
(C) gives military secrets to the enemy
(D) military secrets were given to the enemy
(E) the enemy received military secrets from him
6
...

(A) cell phones, they are increasingly popular
(B) their popularity is growing
(C) they have become more popular
(D) cell phones are increasingly popular
(E) cell phones, they have grown more popular
7
...

(A) country, this explaining why
(B) country, this is why
(C) country; this fact explains why
(D) country; this fact explaining the reason why
(E) country, and explains why
8
...

(A) When you read at a very fast rate, your eyes often skip words, and your mind grasps the
meaning nevertheless
(B) When you read at a very fast rate, your eyes often skip words, your mind nevertheless
grasps the meaning
(C) Because you read at a very fast rate, your eyes often skip words, and your mind grasps the
meaning nevertheless
(D) When you read at a very fast rate, your eyes often skip words, but your mind grasps the
meaning nevertheless
(E) Reading at a very fast rate, words are skipped by your eyes even when your mind grasps
the meaning
9
...

(A) The author, guiding the reader through Emma’s most intimate dreams and fantasies,
accurately

(B) The reader is guided through Emma’s most intimate dreams and fantasies by accurately
(C) The reader, guided through Emma’s most intimate dreams and fantasies by the author who
is accurately
(D) The author, who guides the reader through Emma’s most intimate dreams and fantasies,
accurately
(E) The author guides the reader through Emma’s most intimate dreams and fantasies,
accurately
10
...

(A) through many countries in honoring their
(B) through many countries to honor its
(C) in many countries; it is to honor its
(D) by many countries to honor their
(E) by many a country to honor their
11
...

(A) considerably, while continuing to grow
(B) considerably, and it continued to increase
(C) considerably, with continuing growth
(D) considerably, it continued growing
(E) considerably, continuing increasing
12
...

(A) The Black Death of the fourteenth century, possibly the world’s deadliest epidemic, whose
origin is thought to be central China
(B) The Black Death of the fourteenth century, possibly the world’s deadliest epidemic, its
origin is thought to be central China
(C) Possibly the world’s deadliest epidemic, the origin of the Black Death of the fourteenth
century is thought to be central China
(D) The origin of the Black Death of the fourteenth century, possibly the world’s deadliest
epidemic, is thought to be central China
(E) The fourteenth century’s Black Death is thought to have its origin in central China, was
possibly the world’s deadliest epidemic
13
...

(A) a parental permission slip must be filled out for each student
(B) a student must have their parental permission slips filled out
(C) their parents must fill out a permission slip for each student
(D) a student must have a parental permission slip filled out
(E) permission for each student must be filled out by their parents

14
...

(A) Residents of Chicago have just as much right to complain about the cold as the city of
Minneapolis, a city that
(B) Residents of Chicago have just as much right to complain about the cold as residents of
Minneapolis, a city that
(C) Residents of Chicago have equally the right to complain about the cold as residents of
Minneapolis, a city that
(D) Residents of Chicago, having the equal right to complain about the cold as Minneapolis, a
city where the population
(E) Residents of Chicago, rightfully complaining about the cold as the population of
Minneapolis, a city where it

End of Section 3
...


END OF WRITING TEST
...
Rate each one on a scale of 6 (high) to 1 (low), and write a comment about
your impressions in the spaces provided
...
) Then
compare your comments with those of two SAT evaluators
...

The original essays were written by hand, but these have been typed exactly as written
...
There are many students who work
hard, yet do not receive high grades
...
If the system were to change and every
student who tried hard received a high grade, how would we differentiate between those who are truly
gifted and those who merely make an effort? This is especially important in high school and college,
where intelligence matters a lot
...
I would want the best there is to take care
of me
...
This division should be made apparent within
their grades
...

When electives are involved, a different process could be used
...

However, their artistic and athletic classmates should be awarded a grade for ability
...

*Remember: Since this is an eBook, please record all of your impressions about the essay
separately
...
” Yet, you avoid being
dogmatic about it by thoughtfully considering the use of different grading criteria at different stages
of education
...
Your reference to doctors and dentists is particularly apt
...
Sentences are varied and
generally well-structured
...
g
...

Each reader gave the essay a score of 5, for a total score of 10
...
For example, I think it depends on what
kind of student you are what kind of classes you are in and if you’re an all around prepared student
...
If your lazy and take everything as a joke
...
Late a lot
...

If you re all around prepared student and you really try hard you should give someone high grades
...
In Math Class I had in 9th Grade but I didn’t do so good so Summer School here I come
...
I think that during summer school I set a goal for myself and the teacher helped me
...

*Remember: Since this is an eBook, please record all of your impressions about the essay
separately
...
Your writing suggests that English may be your
second language
...
Numerous problems in the essay point to a need for remedial work in writing before you
attend college
...


Johnny’s Essay
Ever since there were schools there has been controversy over grades, because grades in school in
some ways determine the course of your life
...
On the other hand, a student who is naturally gifted
in the area of the hard course and achieves say a 95 test average with little or no work, should remain
with their test grades for their final average
...
Effort should be regarded as the basis for grading of a very young student,
because grades K–6 are crucial years when children must be shown the importance of effort
...
They should be graded with emphasis on achievement rather than effort,
because trying hard doesn’t matter if when they go out into the real law world, their effort is not
irrelevant
...

It is ridiculous to expect that we can use the same basis for everyone in the educational world
...

*Remember: Since this is an eBook, please record all of your impressions about the essay
separately
...
Then you ask a couple of questions that suggest you are still searching for an idea to
write about
...
It contains a strong topic sentence, but its development could be clearer and more
economical
...

Sentences are varied and occasionally highly effective
...
The concluding idea, however, is not totally justified by the content of your
essay
...

Each reader gave the essay a score of 4, for a total score of 8
...
I believe they deserve high grades for their
effort
...
If they like a certain subject they tend to make an effort and do well in the class
...
If the subject is disliked, the student still should strive and make an
effort
...
If this student does badly, even if they try their best, I
believe they deserve a high grade anyway
...
If this type of
student tries hard they should receive a high grade
...
Grades are not very important for this type of student
...


They don’t go to college
...
If they apply for a job, their employers won’t ask to see their transcript All they want to
know about the student is if they passed and got a diploma
...


Comment to Gavin: Your essay, which focuses on the issue of grades and effort, takes into account the
different needs of certain types of students
...
Expression
is awkward, however, and the repeated use of sentences beginning with “If” suggests a limited
awareness of sentence variety
...

Each reader gave the essay a score of 3, for a total score of 6
...
No necessarily for their achievement
...
However, a system based entirely on effort might allow for an
illiterate child who tries very hard to read to get excellent marks however never learn to read
...
Some players try hard, but they shouldn’t earn a
starting position and cause the team to lose just for that
...
These exams would allow for regular “checks” on what
a student’s actual understanding of the “concepts at hand” were
...

On the aspect of lessening the motive to cheat, this “reward by effort” system could have great
advantages
...
It would be really very
difficult to copy someone else’s effort during a student’s “lunch period
...


Comment to Tad: Your essay starts well and contains some interesting, although awkwardly worded,
ideas about grades and effort
...
Toward the end of the essay, the point is lost in a puzzling array of quotation marks and a
hard-to-follow structure
...

Each reader gave the essay a score of 3, for a total score of 6
...
So many
teachers today (my mother, for example) say they are unable to choose between achievement and
effort to determine their students’ grades
...
When a student doesn’t do well on
tests, their grade must be low regardless of the amount of effort they make
...
The media regularly reports reading
and math test scores of students at all levels, but also history, geography, and science
...
When scores decline, it’s crisis
time
...

As evidence of important of high scores on tests, Atlanta, Georgia is an example
...
The schools were under so much
pressure to earn high scores that the staff resorted to cheating
...
Their only concern was high test scores and creating an image of a high
performing school system
...
They use as an example a doctor who tries hard but
is not skillful in the practice of medicine
...
That argument has some
validity, but I think that grading students should not be an either-or situation
...

Fortunately, there are some teachers in my school who take a balanced approach to grading
...
We have to go the extra mile by showing initiative beyond just doing homework,
participating in class, and taking tests
...
Some kids say lack of time to do extra
work puts them at a disadvantage and that grades should be only based on performance on tests and
assignments
...
He asserts that intelligent people have a responsibility to make a greater effort
than others and often tells the class, “You can’t succeed on brains alone
...


Comment to Tiffany: Your essay begins with the bold and perceptive comment that the essay question,
in the context of present-day educational priorities, is flawed
...
You also provide evidence, using
the example of the incompetent physician, to illustrate the opposite—the inherent absurdity of
rewarding effort alone
...
” As you say, a “balance is better
...

Your essay is well organized
...
Its diction is mostly appropriate, and its sentence structure is sufficiently
varied
...
g, “As evidence of important of high scores…
...

Each reader gave the essay a score of 6, for a total score of 12
...
Enter your scores in the spaces
provided, and calculate the average of the six ratings to determine your final score
...
The score will be reported to you as the sum of the two ratings, from 12
(best) to 2 (worst)
...
Or
better, recruit two different readers to evaluate your essay
...
Because you are judging your own essay apart from others, this
guide may not yield a totally accurate prediction of the score you might earn on the exam
...
C
2
...
C
4
...
E
6
...
A
8
...
C
10
...
B
12
...
C
14
...
D
16
...
B
18
...
B
20
...
B
22
...
B
24
...
A
26
...
D
28
...
E
30
...
E
32
...
A
34
...
D


SECTION 3
1
...
A
3
...
E
5
...
D
7
...
D
9
...
D

11
...
D
13
...
B


Performance Evaluation Chart

Conversion Table
This table will give you an approximation of what your score would be if this practice test had been
an actual SAT Writing Test
...

For example, if your Multiple-Choice Raw Score was 35 and your Essay Subscore was 6, the table
indicates that your final score on the test would be approximately half-way between 500 and 710, or
600
...
Page numbers refer to relevant material for study or review
...
C Choice A contains a pronoun their, which fails to refer to any specific noun or other pronoun
...
Neither refers to any specific noun or other pronoun
...
The pronoun its refers to tradition
...

Choice E violates standard English idiom
...
)
EASY

2
...

Choice C is in the past tense, but it also contains the clumsy and pointless phrase having
turned
...
It maintains a verb tense consistent with the rest of the sentence
...
)
EASY

3
...

Choice B uses an adjective, reckless, instead of the adverb recklessly
...

Choice D, like B, uses an adjective where an adverb is needed and also includes the clumsily
worded construction and there is not
...

(See Pronoun–antecedent agreement, and Faulty diction,
...
E Choice A contains a comma splice
...

Choice B includes an awkward construction, resulting from not knowing, and uses a singular
pronoun it to refer to the plural noun computers
...
Also, the construction for the reason being that is not
expressed in standard English
...

(See Pronoun–antecedent agreement, and Comma splices
...
E Choice A uses a plural verb, were, with a singular subject, program
...

Choice C uses a plural pronoun, their, to refer to a singular noun, program
...

Choice D includes an awkward phrase, of regarding it
...

Choice E is the best answer
...
)
MEDIUM

6
...

Choice B uses the plural verb have changed with a singular subject, concept
...

Choice C uses the singular verb has been with a plural subject, changes
...

Choice D uses the plural verb have undergone with a singular subject humankind
...

Choice E uses the plural pronoun their to refer to a singular antecedent, humankind
...

(See Subject–verb agreement, and Pronoun–antecedent agreement
...
A Choice A is the best answer
...

Choice C contains the singular antecedent whale that disagrees with its plural pronoun they
...
It is the whale itself, not its size, that grows
...
The size of a whale’s ear is not caused
by the creature’s overall dimensions
...
)
HARD

8
...

Choice B is unsatisfactory because the verbs spoils and enhancing are in different tenses
...

Choice D is the best answer
...

(See Faulty pronoun reference, Shifts in verb tense, and Faulty idiom
...
C Choice A is unsatisfactory because the shift in grammatical subject from passengers to
searches leads to the awkward usage but there are not frequent
...
The sentence would be more effectively
expressed if one clause were subordinated to the other
...

Choice D shifts the subject from passengers in the first clause to searching in the second
clause
...

Choice E violates standard English idiom
...
)
MEDIUM

10
...

Choice B uses so instead of as in making a comparison
...

Choice C illogically compares students to school spirit
...

Choice E uses a singular pronoun that to refer to the plural noun students
...
)
HARD

11
...

Choice B is the best answer
...

Choice C, by inverting the usual word order, is awkwardly expressed
...

(See Faulty parallelism, and Awkwardness
...
C Faulty verb tense
...
Use decided
...
C Faulty comparison
...

MEDIUM

14
...
The plural noun dilemmas requires a plural verb
...

EASY

15
...
When referring to a person, use the pronoun who rather than which
...
E No error
...
B Wordiness
...

HARD

18
...
The past perfect tense should be used to express action completed prior to
some other event or action
...

HARD

19
...
The singular pronoun this fails to refer to any specific noun or
other pronoun
...
B Faulty parallelism
...
Use represents
...
B Noun–verb agreement
...
Use have opposed
instead of has opposed
...
C Faulty parallelism
...
Use gives them or an
equivalent verb in the present tense
...
B Noun–verb agreement
...
Use does
instead of do
...
E No error
...
A Faulty verb tense
...

HARD

26
...
In standard English usage, the idiom is significant to
...

HARD

27
...
A double comparison is created by adding –er to the adjective
...

MEDIUM

28
...
Pronouns in a phrase beginning with a preposition (between) must be in
the objective case
...

MEDIUM

29
...

HARD

SECTION 2—IMPROVING PARAGRAPHS
30
...
(For example, Having learned about the impending hurricane, the
residents evacuated their homes
...
Therefore, a different form of the verb is a better choice
...

Choice B suggests that sentence 2 would serve as a better opening sentence of the essay
...

Choice C is the best answer
...

Choice E improperly deletes the comma and adds needless words to the sentence
...
E Although sentences 4 and 5 are grammatical, they are wordy
...
” By combining the sentences, one of the repetitive phrases can be eliminated,
thereby making the sentence more concise
...
Two independent sentences may not be joined by a
comma
...

Choice B is unsatisfactory because it eliminates one of the essay’s important ideas—that the
railroad was a technological marvel
...

Choice E is the best answer
...

HARD

32
...
A
transitional word or phrase is needed to explain the reason for unsafe conditions
...

Choice B is the best answer
...

Choice D introduces an awkwardly expressed idea suggesting improperly that only the
Chinese workers were exploited
...

Whether a piece of evidence is good should be left for the reader to decide
...

HARD

33
...
The vagueness of the phrase “unsafe and cost
lives” in sentence 9 might well leave readers wondering about what perils of railroad work
the writer had in mind
...
Along with sentence 11, it vividly details one of the perils faced
by workers on the railroad
...

Choices C, D, and E fail to describe accurately how sentence 10 supports or develops
sentence 9
...
A The short last paragraph of the essay lacks a main idea
...
What the
paragraph needs is a topic sentence that somehow unifies these disparate concerns
...
It provides a reason for the government’s financial participation
in the project, and it alludes to the fact that the railroad’s construction was basically a private
enterprise
...
It is better suited for a
paragraph on the debt incurred by the government to pay for the railroad
...

HARD

35
...
It asserts that greed drove the businessmen to engage
in fraud but provides none of the gory details
...

Choices A, B, and E are related to the topic of the entire essay but have nothing to do with the
issues raised in the last paragraph
...
First, the paragraph should focus more generally on the men’s unethical
business dealings
...
It correctly describes the material that should follow sentence 14
...
D Choice A is unsatisfactory because it joins two independent clauses with a comma
...

Choice B is a sentence fragment
...

Choice C is written in the passive voice and also contains a comma splice
...

Choice E contains two clauses with no grammatical relation to each other
...
)
MEDIUM

2
...

Choice B contains a misplaced modifier
...

Choice C is wordy
...

Choice D contains the pronoun its, which refers ambiguously to both faculty and school
...
The sentence would be more effectively
expressed if one clause were subordinated to the other
...
)
EASY

3
...

Choice B contains the awkwardly worded construction Although its being
...

Choice D includes because, an illogical word choice in the context of the sentence
...

(See Comma splices, and Misplaced modifiers
...
E Choice A contains a pronoun, it, that lacks a reference to a specific noun or other pronoun
...

Choice C is excessively wordy
...
In context, satisfaction out of sports is nonstandard
English
...

Choice E is the best answer
...
)
HARD

5
...
The first two accusations are stated as nouns
—cowardice and desertion
...

Choice B is the best answer
...
See A
...
See A
...
See A
...
)
MEDIUM

6
...

Choice B uses a pronoun, their, that fails to refer to a specific noun or other pronoun
...

Choice D is the best answer
...

(See Shifts in grammatical subject, and Faulty pronoun reference
...
C Choice A uses an awkward phrase, this explaining
...

Choice C is the best answer
...

Choice E contains a nonsensical construction: The subject restaurants is unrelated to the verb
explains
...
)
MEDIUM

8
...

Choice B contains a comma splice
...

Choice D is the best answer
...

Choice E contains a dangling modifier; the clause that begins Reading at should modify your
eyes or your mind instead of words
...
)
HARD

9
...
The –ing form of a verb (e
...
, portraying) cannot serve as the main verb without a
helping verb, as in is portraying or had been portraying
...
It is the author, not the reader, who portrays the
plight of women
...
See A
...

Choice E is the best answer
...
)
MEDIUM

10
...
In context the phrase through many countries is
nonstandard
...

Choice C uses the singular pronoun its to refer to the plural noun countries
...

Choice E contains the plural pronoun their to refer to the singular noun country
...
)
MEDIUM

11
...
The use of while suggests that 2011 and 2012
occurred at the same time
...

Choice C contains with continuing growth, a construction grammatically unrelated to the main
clause of the sentence
...

Choice E is expressed in awkward, nonstandard language
...
)
HARD

12
...
Its grammatical subject, The Black Death, lacks a verb
...

Choice C contains a misplaced modifier
...

Choice D is the best answer
...

(See Sentence fragments, Comma splices, Misplaced modifiers, and Mixed construction
...
D Choice A contains a dangling modifier; the clause that begins Before going should modify
student instead of parental permission slip
...

Choice C uses a pronoun, their, that lacks a specific reference to a noun or other pronoun
...

Choice E contains a dangling modifier; the clause that begins Before going should modify
student instead of permission
...
)
HARD

14
...

Choice B is the best answer
...

Choice D illogically compares residents of Chicago to the city of Minneapolis
...

(See Faulty comparisons, and Sentence fragments
...
It’s more of a come-on than a promise
...
An essay
is basically the product of an author ’s thinking
...
When given the topic less than half an hour before the
paper is due, you can’t expect to pore over the assignment
...

Another reason that the heading is illusory is that you don’t learn to write very well by reading
about how to do it
...
So, it pays to have a process
in mind ahead of time, one that helps you work rapidly and efficiently
...
The plan that follows is a place to start
...


First Stage: Prewriting
Prewriting consists of the planning that needs to be done before you actually start writing the
essay:
• Reading and analyzing the question
...

• Gathering and arranging supporting ideas
...

• Developing paragraphs
...

• Structuring sentences for variety and coherence
...


Third Stage: Editing and Proofreading
• Editing for clarity
...

• Checking for standard English usage and for mechanical errors, including spelling, punctuation,
and capitalization
...
Writers compose, revise, and proofread
simultaneously
...
In fact, no stage really ends until the final period of the last sentence is put in
place, or until your proctor at the SAT site calls “Time!”
No book can tell you exactly how much time to devote to each stage
...
But most students get good results by devoting more than half
the time—about thirteen to eighteen minutes—to composing and no more than five minutes each on
prewriting and editing/ proofreading
...
practice
2
...
practice some more
...
During the weeks before the SAT, or even sooner, pick same sample essay topics
...
Pace yourself and keep track of how much time you spend thinking about the
topic, how many minutes you devote to composing the essay, and how long it takes you to proofread
and edit
...
Don’t count words, and don’t expect to recopy your first draft
...
If necessary, number the
sentences to make clear the order in which they are to be read
...
Just make sure that the essay is readable
...
Essays of more than 400 words are
unnecessary
...
In fact, less can
be more, for a shorter essay of, say, 250 to 300 words can focus sharply on a limited subject
...
But don’t be
satisfied with an abbreviated one-paragraph essay that could suggest a shortage of thinking ability
...


PREWRITING: GETTING SET TO WRITE
Reading and Analyzing the Topic Carefully
At the risk of stating the obvious, begin by reading the assigned essay topic, or prompt
...
Read it twice or three times, or until you are absolutely sure of what you have been asked to

do
...
Their intention is to provoke thought and suggest an idea or two to discuss in your essay
...
Weave it
into your essay if you wish, but only if it’s appropriate and advantageous to do so
...
Consider your essay as a kind of contract or agreement between you and
readers who’ll be spending time with your words
...
As the writer, you are being challenged to write something so riveting that readers
will resist the temptation of moving their eyes off of the page or letting their minds wander
...
You think about what they’d like
...
If all goes well, you get
a reward for your efforts
...
Underline the key words that
define the task to be performed
...


Topic A
Think carefully about the issue presented in the following statement and the assignment below
...
Failure is delay, not defeat
...
Failure is something we can avoid only by saying nothing, doing nothing,
and being nothing
...
Support your position with reasoning
and evidence drawn from your reading, studies, experience, or observation
...
The position you take is less crucial
than your ability to support your ideas with specific examples from your knowledge, background, or
observation
...

An interesting and readable response to the question might be based on your own life
...
Briefly describe the failure and explain its cause
...
Explain how your lack of success may have taught you a
valuable lesson
...
In that case, your essay might show that the prompt is dead wrong
...
Regardless of your position, however, be sure to
include more than one example
...


Topic B
Think carefully about the issue presented in the following quotation and the assignment below
...

John Stuart Mill (1806–1873)

Assignment: Does the need to question or evaluate your happiness mean that you are not as happy as
you think? Plan and write an essay in which you state and develop your point of view on this issue
...

Required task:






Explanation: What you write depends largely on your interpretation of the word happiness
...
Questioning your own happiness may indeed lead you to the startling discovery that you
are less happy than you thought
...
Once they
tasted of the Tree of Knowledge, they lost their paradise forever
...
Think of Socrates’ famous saying, “The unexamined life is not worth living
...
May be you believe that absolute
statements about happiness are foolish because happiness comes in so many forms
...


Narrowing the Topic
A well-focused essay on a limited topic is always better than an essay that tries to cover too much
ground in just a few paragraphs
...
The sharper your focus, the better
...


If you are a fast writer, you might scribble 350 well-chosen words onto the page in twenty-five
minutes
...
) If the
topic is too broad, you are likely to state a few obvious generalities, resort to hackneyed ideas, and
maybe even throw the bull a little bit
...
On the other hand, if you’ve narrowed the topic
sufficiently, you stand a far better chance of saying something sensible, scintillating, meaningful,
provocative, and interesting
...

It’s impossible to predict the topic you’ll be asked to write on
...
Your first
task, therefore, is to think small—to reduce the topic to a size snug enough to fit into a short essay
...

Here is what she said about narrowing her topic:
I was a little nervous at the beginning when I opened my test booklet to find an essay topic on
conformity in groups of people
...
But the prompt
cleared it up for me
...
It also determines who the group leader is going to be and the amount of freedom that
members will have
...

The instructions told me to answer the following question in my essay: “Do groups that
encourage nonconformity and disagreement function better than those that discourage it?”
Luckily, during the previous few weeks I’d written a few SAT essays for practice and knew that
I needed to back up my opinion with evidence from my studies, reading, observation, or
experience
...
So, off the top of my head I began to jot down the names of groups in the margin of my
test booklet—names like Chinese-Americans, airline employees, Netflix subscribers, and
graduates of Penn State University
...
Besides, I realized that in each
group there would be many people who probably had little in common with each other and
wouldn’t share a unanimous view on the need for group conformity
...
debate club
B
...
platoon of U
...
Marines
D
...
school faculty
F
...
There was no way could I cover all these groups
in a short essay
...
So, I pruned my list after thinking for a few more seconds about
each group
...
A debate club wouldn’t be a debate club
if its members always agreed
...

Suddenly I felt confident that my initial idea was correct
...

But after appraising C and D, I wasn’t so sure
...
On battlefields and basketball courts, I asked myself, wouldn’t
success depend on strict adherence to the group’s values and goals? I realized, that a yes answer
might not work after all
...
It would be so depressing to go to a school where every teacher
was the same and every class was run in exactly the same way
...
The ideal would probably be a faculty that embraced a set of
guiding principles but also valued a school climate that promoted reasonable dissent and
individuality
...

…All these thoughts sped through my mind in a fraction of the time it takes to read them
...
Instead, my main
idea would take a middle-of-the-road position, like: A group’s purpose and goals determine
whether dissent is a help or a hindrance
...
As I began to write, I reminded myself that
somewhere in my essay I ought to talk briefly about groups with other types of goals, such as the
faculty and the school board where they had to let circumstances determine when dissent is helpful
and when it’s not
...

I came to the end of the essay with about three minutes to spare and used the time to proofread
my essay and change a few words to make it sound more mature
...

While planning what to say in your essay, take Megan’s story to heart
...
Don’t let yourself be deluged with
ideas
...

P
...
One reader gave Megan’s SAT essay a 5, the other a 6, for a total score of 11
...
If the essay
strikes them as dull or disappointing after a few sentences, they may realize that their approach is too
vague, too broad, too boring (and if the writer is bored, imagine what the essay will do to prospective
readers)
...
Time restraints on the SAT won’t give you more than one chance to start over
...


Choosing a Main Idea
Once you’ve narrowed the topic, it’s time to decide what to say about the topic
...
An essay shouldn’t simply be
“about,” say, hard work, heroism, beauty, or any other topic
...
Essays may be written with
beautiful words, contain profound thoughts, and make readers laugh or weep
...
You don’t want readers coming to the end
of your essay scratching their heads and asking, “Huh? What’s the point?”
Every bit of your essay from start to finish should contribute to its main idea
...
) Any material that wanders from the main idea should be
discarded
...
Naturally, the main
idea of your essay will depend on your response to the particular issue presented by the prompt
...

Let’s say the issue relates to the fundamental rights of high school students
...
Your main idea might be any of
the following:
1
...


2
...

3
...

4
...

5
...

6
...

7
...

Using one of these main ideas as its starting point, the essay would then discuss the validity of your
opinion
...
Let’s say, for
instance, you are asked to write about seat belt laws that require everyone in a car—driver and all
passengers front and back—to buckle up
...
Or the essay’s point might be that safety laws supercede a person’s right
to choose whether to wear a seat belt
...

CHOOSING A MAIN IDEA FOR YOUR SAT ESSAY
Topic: The prompt gives you an issue to write about
...

Purpose: The essay gives you an opportunity to develop support for your opinion using reasoning
and examples taken from your reading, studies, experience, or observation
...
SAT readers won’t find fault with
opinions with which they disagree
...
Likewise, because you don’t want to sound
pompous or pretentious, avoid picking a main idea solely to show off intellectual superiority or
political correctness
...

At the same time, however, steer clear of main ideas that are clichés and platitudes
...
As they plod through scores of SAT essays on the same topic, they’ll appreciate and reward
those that contain fresh ideas
...
Not that your main idea should be off the wall
...
You’ll
never be penalized for a clearly written, sober essay that demonstrates insightful thinking and beliefs
...
But what if you hate
to write and don’t care for the given topic? Is it possible to write a decent essay on a topic that makes
you yawn?
The answer is a resounding YES! because you have no choice
...
But raising a stink about it won’t get you far
...
Show your resilience
—a quality that college admissions officials value and admire
...


Practice in Choosing a Main Idea
Directions: Respond to each of the following prompts by writing three or more sentences that
could serve as main ideas for an essay
...
“Whether you think you can, or that you can’t, you are usually right
...


2
...

Assignment: Which is a more effective way to teach children to behave—to promise rewards or to
instill a fear of punishment?

3
...
Yet, many lottery
winners have suffered unexpected negative consequences
...

Assignment: Should state and local governments sponsor lotteries that can leave both winners and
losers worse off than before? Plan and write an essay that develops and supports your views on this
issue
...
“There is nothing like returning to a place that remains unchanged to find the ways in which you
yourself have altered
...


5
...
It is a matter of choice
...

William Jennings Bryan (1860–1925)
Assignment: Do you think that a destiny achieved by the decisions and choices you have made is
preferable to a destiny that comes from chance or luck? Plan and write an essay that develops and
supports your views on this issue
...
List your
thoughts on paper—just a word or two for each idea
...
Then draw circles around key words, connect related ideas with arrows, or just underline
the thoughts you’ll definitely use in your essay
...
While you plan, one idea may trigger a flood of others
...
(Everyone should have such a problem!) Your task then
would be to pick out and develop only the best of the best
...
The best order is the
clearest, the order your reader can follow with the least effort
...
The route you plan depends on the purpose of the trip
...
In storytelling, the events are usually placed in the
sequence in which they occur
...
An essay that compares and contrasts two books or two people may deal with
each subject separately or discuss the features of each point by point
...


TIP
Rank your ideas in order of importance
...
To guard against aimlessness, rank your ideas in order of importance
...
Giving away your pièce de résistance at the start
is self-defeating
...
Launch the essay with your second best, and sandwich your least favorite
between the other two
...
Coming last, it is what readers have
fresh in their minds when they assign the essay a grade
...
Why three? Mainly because three is a number that works
...
One is
insufficient, and two only slightly better
...
Psychologically, three
also creates a sense of rhetorical wholeness, like “blood, sweat, and tears,” and “of the people, by the
people, and for the people
...
A narrative essay, for instance,
breaks naturally into a beginning, middle, and end
...
In
an essay of comparison and contrast, you should be able to find at least three similarities and
differences to write about
...

Each of three ideas may not demand an equal amount of emphasis, however
...

But whatever you emphasize, be sure that each idea is separate and distinct
...


The Formula
The five-paragraph essay formula is a simple, all-purpose plan for arranging ideas into a clear, easyto-follow order
...
Its greatest
virtue is clarity
...


You needn’t follow the formula to the letter
...
Yet many essay writers, even those who take
a circuitous path between the beginning and end, use some version of it
...
The body says it, and the conclusion tells readers what they’ve been told
...


Practice in Gathering and Arranging Ideas

Directions: For the following essay topics, write three different and distinct ideas that could be
used to support each one
...
Then rank them in
order of importance
...
Topic: The advantages (or disadvantages) of requiring all students to take gym every semester

B
...
Topic: For (or against) an honor code in your school

D
...

Benjamin Franklin

COMPOSING: PUTTING WORDS ON PAPER
Writing a Gripping Introduction
Introductions let readers know what they’re in for
...
Distinctions will be made between early childhood (ages 3–7) and
middle childhood (8–12)
...
Rather, just state your point
...

Jill B began her essay on the rights of high school students this way:
On Monday morning, October 20, I arrived in school to find every locker door in my corridor
standing ajar
...
I believe that this illegal action was a violation of both my civil rights and the
civil rights of every other student in the school
...
Because she can’t cover all there is to say about students’
rights in one or two pages, Jill focuses on one issue raised by her personal experience on a particular
Monday morning
...
Jill’s opening is effective because it tells an informative anecdote that leads directly
to her essay’s main idea—that locker searches violate students’ civil rights
...
Many teenagers
smoke weed or do other drugs
...
Society is working
on the problem but has not found an effective solution
...
Why? Because nothing in that four-sentence
paragraph says anything that you don’t already know
...
Not only that, the topic
being introduced is far too broad for a short essay
...
And it’s a problem that won’t go away until someone figures out how to get kids
to just say NO!
This introduction uses a compelling image of young children out of control
...

Here is another example of a dull opening:
Photography is one of the most popular hobbies in the world
...
A
more lively opening evokes a different response:
I took my brand-new digital camera on spring break, but when I came home the box was still

unopened
...
It implies
that the writer is about to tell a story that explains why he took no photos
...

Sharp: Why did the brilliant author George Orwell goof?
The first opening, written as a nondescript statement of fact, won’t interest anyone unfamiliar with
1984, but the second one, a pithy question, is more powerful
...
” The very idea entices readers to find out what happened
...

As you write practice essays, try using the following five common techniques, each illustrated with
an example from an essay by a high school student
...
Start with brief account of an incident—real or invented:
By lunch period, Megan, a senior at Brookdale High School, had already traded text messages with
her brother in college, with her dad at work, and with a friend who was absent from school that
day
...

Lisa N
...
Her list of text
messages suggests that the essay will be about the effects of staying constantly in touch with others,
or about some other aspect of communication
...
State a provocative idea in an ordinary way or an ordinary idea in a provocative way
...

As any football hero will tell you, on-the-field brains count for more than brawn
...

This unusual idea may cause readers to question Ollie’s sanity or maybe to analyze their own
images of football players
...

3
...
Whatever the source, its sentiment must relate to the essay’s topic
...

Gary F
...
He gives a clever new twist to a common adage

and with a few words has introduced his main idea—the futility of changing people’s basic
characteristics or core values
...
Refute a commonly held assumption or define a word in a new and surprising way
...

Rebecca V
...
” How can someone who
has written neither rhymes nor verses be called a poet? That the label “poet” applies to her boss is
intriguing because most poets don’t work as supermarket managers
...

5
...

Why are stories of crime so fascinating?
Doug T
...


Doug’s opening has wide appeal because most of us read the police blotter in the newspaper, tune
to news channels that hype crime stories, talk about the latest murder, or watch CSI on the tube
...

Although an effective introduction always helps to draw readers into an essay, you needn’t feel
obligated to contrive a super-catchy opening
...
Because there’s no time to dawdle during the twenty-five minutes
allotted for the essay, a plain statement conveying the topic and main idea of your essay may be all
you need
...

Praise them for important things, even if you have to stretch them a bit
...
They live
on it like bread and butter and they need it more than bread and butter
...
The statement that children should be praised, “even if you have to stretch them a bit” makes good
sense for at least three reasons
...
Is it a good idea to praise children even when they don’t really deserve it? I don’t think so
...

3
...

None of these openings will win a prize for originality, but they all do the job—introducing the
topic and stating the essay’s main idea
...
The first
essay is likely to discuss three reasons for agreeing with the prompt
...

Another virtue of these sample openings is that they are short
...
An opening that comprises, say, more than a quarter of your essay reflects
poorly on your sense of proportion
...
A
good idea may strike you at any time
...
Once they hit their stride, they figure out the point of their essays
and work on openings sure to hook their readers
...

Delete your first paragraph unless it contains ideas you can’t live without
...
Try to write an appealing opening for each
...
The courage of one’s convictions





2
...
“Keep it! You may need it someday
...
The wrong time in the wrong place





5
...
Each
new paragraph alerts readers to get ready for a shift of some kind, just as your car ’s directional

blinker tells other drivers that you’re about to turn
...
The writer may simply want to nudge the
discussion ahead to the next step
...

The paragraph you are now reading, for instance, is linked to the one before by the connecting word
Yet
...
Abrupt starts may be useful from time to time to keep readers
on their toes
...

In an essay, paragraphs usually play a primary role and one or more secondary roles
...
The concluding paragraph leaves the reader with a thought to remember and provides a sense
of closure
...
They carry forward the main
point of the essay by performing any number of functions, among them:
• Adding new ideas to the preceding discussion
• Continuing or explaining in more detail an idea presented earlier
• Reiterating a previously stated idea
• Citing an example of a previously stated idea
• Evaluating an opinion stated earlier
• Refuting previously stated ideas
• Providing a new or contrasting point of view
• Describing the relationship between ideas presented earlier
• Providing background material
• Raising a hypothetical or rhetorical question about the topic
Whatever its functions, a paragraph should contribute to the essay’s overall growth
...
Similarly, any idea within a
paragraph that doesn’t contribute to the development of the paragraph’s topic needs to be changed or
eliminated
...
Such guiding
sentences differ from others because they define the paragraph’s main topic; hence the name topic
sentence
...
The topic of some paragraphs is so obvious
that to state it would be redundant
...


TIP
Use topic sentences to guide readers through your essay
...
What they all have in common is their helpfulness
...
To drive from your home to school, for example, you turn left at the stop
sign, take a half right under the railroad trestle, and a right at the Exxon station
...
Similarly, in a piece of writing, a topic sentence often marks a turning point that
tells readers the direction they’ll be going for a while
...
And some
paragraphs don’t even need a topic sentence
...
For instance, a description of a fast-food restaurant might detail the
crowd, the noise, the overflowing garbage cans, the smell of cooking oil, the lines of people, the
crumb-strewn tables, and so on
...
To state explicitly “It was a
busy day at Burger King” would serve no purpose
...
Highlight the topic
sentence in each
...


1
...
[2] The first time we moved I was
only four years old, and it didn’t bother me
...
[4] But then we heard it was time to move on, and we settled in Minnesota
...
[6]
From there it was two years in England and a year in Washington, D
...
[7] We’ve been in
Massachusetts for almost six months now, and my main problem is answering that question,
“Where are you from?”
2
...
[2] A conviction for a felony can prevent a person from being able to
enter a profession such as medicine, law, or teaching
...
[4] Special hearings are required before an ex-convict
can hold a government job
...
[1] Music blasts from twenty boom boxes
...
[3] Teenagers throw frisbees at each other
...
[5] A single-engine plane, trailing a long sign—EAT PIZZA AT SAL’S—
buzzes overhead
...
” [7] During the
summer the beach is a noisy place
...
[1] Clothing designers create new styles every year
...
[3] Forgotten
styles hang in closets gathering dust
...
[5] People fall in love with
new cars and sell their old models long before they are obsolete
...
[7] And for
what? [8] Just to look good
...
[10] If

people would get in the habit of buying goods only when they need replacement, waste would
become an exception in America instead of a way of life
...
[1] Perhaps it’s true that “all the world’s a stage,” as Shakespeare said, because I have noticed that I
act one way with one group of people and another way with a different group
...
[3] I may act very shy or silly
...
[5] The circumstances just make me act that way
...
[7] I won’t stop talking, and people think I am about 20 years old
...
[9] That’s a very scary thought
...
[1] During these years, my family has had about sixty foster children come into our house to live
...
[3] Each child brought to our
door brings a different tale of misfortune
...
[5] When we first started, the parents of the child usually wanted him or her but were
temporarily unable or unprepared to care for their son or daughter
...
[7] Most of the time the mother is a
combination of those
...
[9] Three of their four
parents are in jail, and one of the fathers is unknown
...

7
...
[2] All
personal goals and desires should be thrown aside unless they coincide with the common good of
society
...
[4] With this philosophy, drastic improvements may be made in a relatively short time
...

8
...
[2] Childlike boys and
girls suddenly blossom into young men and women
...
[4]
Decisions need to be made about what is important and what is not
...

9
...
[2] To
answer the question, one must first define “man
...
[4] There
is the Arnold Schwarzenegger type who solves all of life’s problems with physical strength and
advanced weapons
...
[6] Finally, there is the Willy
Loman type, a character in Death of a Salesman, who struggles his whole life pursuing an illusion
...

10
...
[2] American history textbooks justify the bombings as
something that needed to be done in order to prevent even more deaths during a longer war
...
[4] If the United States had lost the war, then the
bombings would have been thought to be criminal actions
...
[6] In fact, throughout history, the war crimes of the
victors have repeatedly been justified
...
After reading each
paragraph, write an appropriate topic sentence
...


1
...
She enjoys dancing, going to
parties, being with lots of people, and spending money freely
...
He looks at things logically and practically, not giving in to his emotions
...

2
...
As other countries learned American techniques of
training, however, their runners improved
...

3
...
In order
to maintain their grip on the domestic market, Chrysler, General Motors, and Ford squelched the
competition
...
Automobiles
were changed very little from year to year
...

4
...
Hundreds of people wielding sticks and
knives and pastry rollers screamed at the figure who emerged on the balcony
...

5
...
The author must have lived with the family in
the book because she describes the members in lifelike detail
...
By the end, you know them as though they were your
own brothers and sisters
...


One day I was smoking in the boys’ bathroom when a teacher walked in
...
My mom grounded me for a month,
and I didn’t get the loan my dad had promised me to buy my friend’s used car
...


Probably the most important part of this new life is learning to get along with your roommates, the
people you see most often
...
The person should be
a nonsmoker and have similar interests to mine
...
In
a nutshell, she should be like me
...


In childhood I never hesitated to take chances, to jump over wide cracks in the rocks
...
I scraped my knees, bled a little,
but came back daring to try again
...
I live a style of life in which being in control and on top of
things is paramount, where being the best and being perfect is what I yearn for
...
I take fewer chances
...


He knew that he grew irritable more frequently
...
They must have
thought he was hard of hearing or didn’t understand
...
Sometimes he grew angry
about the way he was ignored after he asked for something
...

10
...
D
...
The foot-thick walls of oven-baked adobe brick, plastered over
smoothly with clay, kept the occupants warm in winter and cool in summer
...


PART C
Directions: What follows is a three-paragraph excerpt from the journal of a visitor to the South
Pole
...


Antarctic Adventure

Bellies flattened on the snow, they pant and claw their way across miles and miles of frozen
landscape
...
After a day’s run, the dogs eat supper and sleep soundly
...


The scale is unreal, almost as if it were a landscape from another planet
...
It is antiseptic and can
only be compared with life under the ocean or in space
...
Ice blocks and sinister piles of snow tell a tale of avalanches tumbling regularly

from the mountains all around
...
On foot, sunk to the hips in snow, you might cover less than a mile before
dropping from exhaustion
...


PART D
Directions: Print out or photocopy an essay you’ve written recently
...
Let a friend do the same on the second copy
...
If you
agree, you can be pretty sure that your topic sentences are doing what they are supposed to do
...
Ideas can be arranged from
general to specific, or vice versa
...
In a cause-and-effect paragraph, logic dictates that the cause precedes the
effect, but the opposite may sometimes be preferable
...


TIP
Develop your ideas with more than one sentence or single example
...
Most of the time, thorough
development of an idea calls for several sentences
...
But the bulk of contemporary nonfiction consists of paragraphs
of four to eight sentences
...
Disjointed paragraphs, on the
other hand, consist of sentences arranged in random order
...
Meaning serves as the primary glue that holds a coherent paragraph
together, but transitional words and phrases such as for example, also, but, and on the other hand also
help
...

[1] Tom Joad, the protagonist of John Steinbeck’s Grapes of Wrath, faces a dilemma after his
release on parole from an Oklahoma state penitentiary
...
[3] Although he claimed self-defense at his trial, Tom
was found guilty
...
[5] When he finally catches up with his family,
however, they are en route to California
...

Sentence 1 expresses the paragraph’s most general idea and serves as the topic sentence
...
Five years earlier, in sentence 2, explains what had occurred before the events in the first
sentence
...
Upon his
release, in sentence 4, adds still another link to the topic sentence
...
And in sentence 6,
Consequently introduces the precise nature of Tom’s dilemma
...


Practice in Developing Paragraphs
Directions: The sentences in each of the following groups make up a coherent paragraph
...
Rearrange sentences logically
...


1
...
A particular worker, for example, may lack the skill to do a certain job
...
Another important variable is inclement weather, which can set a project back for days on
end
...
In spite of the best laid plans and preparation, building projects sometimes work out badly
...
The main reason is that the foreman can’t always predict what the workers will do
...
Then, the project can’t proceed until another worker is found
...
____ a
...

____ b
...

____ c
...

____ d
...

____ e
...

3
...
College students swipe IDs to open doors, buy tickets to athletic events, operate ATMs, do
their laundry, and even indicate their presence in a lecture hall
...
It also identified students when they took books from the library
...
Most colleges issue ID cards to students
...
But magnetic strips and wireless chips have converted this modest piece of plastic into an
essential, multiuse appendage
...
At one time, a student ID was just a laminated card good for gaining access to campus
buildings
...
____ a
...

____ b
...

____ c
...


____ d
...

____ e
...

5
...
Soon thereafter, some colleges began to ask applicants whether they received professional
help in completing their application essays
...
They also found that teachers, counselors, and other adults were giving more than casual
lessons in essay writing
...
Wondering what caused the change, admissions officials soon discovered that many high
schools had made instruction in writing an application essay a part of the curriculum
...
In recent years, students from certain parts of the country started sending in polished
college application essays in large numbers
...
For up to $250 an hour, some “consultants” were all but composing essays for anxious
students
...


Practice in Identifying Paragraph Unity and Coherence
Directions: The following paragraphs may suffer from either lack of unity, lack of coherence, or
both
...
Some
paragraphs may not need revision
...
[1] Lord of the Flies is about a group of English schoolboys stranded on a remote island after an
airplane crash
...
[3] There are groups at this high
school, too
...
[5] He is a thinker, but he gets killed by another group, the savages, led by Jack
...
[7] The
different groups in the novel are amazingly similar to groups in this school, known as the nerds,
the jocks, and the preps
...
[1] Under the present law, doing illegal drugs can have serious consequences for young people
...
[3]
An arrest or conviction for a felony can complicate their lives and plans
...
[5] Parents like to brag about their children’s
accomplishments
...
[1] Today there is general agreement that we are experiencing unprecedented change
...
[3] The majority of people no longer live in traditional
families that consist of two natural parents and their children
...
[5] The United States has an increasingly diverse population
...
[7] Children at all social
and economic levels learn to expect that lying, cheating, and stealing are rampant in business,
politics, and almost every other endeavor
...


4
...
[2] Most parties try to unite divided interests within
their ranks in order to appeal to the widest number of voters
...
[4] In dictatorships,
criticism of the party in power may be regarded as treason
...
[6] Elections mean little in such countries, for the people have no real
choice among the candidates
...


5
...
[2] A large number of workers engage
in buying, pricing, and selling merchandise
...
[4] In recent years, mail-order buying on the
Internet has forced many department stores to go out of business
...
[6] The personnel
department hires employees and deals with employment problems
...
[1] The porpoise, or bottlenose dolphin, is one of the most intelligent animals
...
[3] Scientists rate their intelligence between that of the chimpanzee, long held as the most
intelligent nonhuman animal, and the dog
...


7
...
[2] Defoe was born in London in 1660 and started

writing only after he went bankrupt in a business career
...
[4] Today, he is best known
for Robinson Crusoe, which is but a tiny fraction of his work
...
[1] Aristotle made valuable contributions to the study of logic
...
[3] Socrates fought the
Sophists all his life because he believed in truth, and the Sophists denied the existence of truth
...
[5] The period of
ancient philosophy reached its climax in Greece in 600–500 B
...


9
...
[2] They made
pitchforks, for example, by attaching long handles to deer antlers
...
[4] They whittled wooden spoons, bowls,
platters, and used gourds and the horns of sheep and other animals for drinking cups
...
[6] Then they would rub kernels of corn across
the jagged surface to make cornmeal
...
[1] You can’t find Potter ’s Field on a map
...
[3] Rather, it is the name given
to any plot of land reserved for the burial of unidentified and destitute people
...
[5] After Judas betrayed Christ for thirty pieces of
silver, the priests used the money to buy “the potter ’s field to bury strangers in
...
[7] Land is too valuable to use for burying
unknown and unclaimed corpses
...


Transitions
Consider your readers as tourists in a foreign land and your essay as a journey they take from one
place to another
...
As
their guide, you must tell them where they are going (the introduction) and remind them of the
progress they’re making (the body of the essay)
...
To keep readers well informed,
you needn’t repeat what you’ve already written but rather plant key ideas, slightly rephrased, as

milestones along the way
...
The phrase “To
keep readers well informed” cues you to keep in mind the topic of this paragraph—helping readers
find their way
...


TIP
English is crowded with transitional words and phrases
...
This can be done with such words as this, which actually ties the sentence you are now
reading to the previous one
...
Fortunately, the English language
is brimming with transitional words and phrases for tying sentences and ideas together
...
With a bit of thought, you could probably add to the list
...
Ideas
themselves can create strong links
...

[1] As a kind of universal language, music unites people from age eight to eighty
...


[1] At the heart of Romeo and Juliet is a long-standing feud between the Capulets and the
Montagues
...

[1] To drive nails into very hard wood without bending them, first dip the points into grease or
soap
...

One of your goals on the SAT is to assure readers a smooth trip through your essay
...

Before long, they’ll give up or get lost like travelers on an unmarked road
...


Practice in Using Transitions
Directions: Use as many transitions as you can while writing paragraphs on the following
suggested topics
...
Write a paragraph on how to do something—drive a car from home to school, pull a practical
joke, avoid doing homework, burn a CD, get on the good side of a teacher, give your cat/dog a
bath
...

2
...
Use as many
RESULT transitions as you can, but don’t go overboard
...
Write a paragraph that compares and contrasts one of the following: the way people respond to
pressure, groups in your school, two athletes, then and now, boredom and laziness, two books, a
friend who turned into an enemy, an enemy who became a friend
...

4
...
Use as many ADDITION transitions as you can, but only where they make
sense
...
To write clearly, use plain words
...
Why?
Because an elegant word used merely to use an elegant word is bombastic…er…big-sounding and
artificial
...
Or worse, they
make writers sound phony if not foolish
...
Nor would you call your
teachers pedagogues or your dog a canine
...

How much clearer and more direct it would have been to write:
I had to walk the dog before starting my history homework
...
A sign
that says STOP! conveys its message more clearly than CEASE AND DESIST
...
” Simple doesn’t necessarily
mean short, however
...
The word fid
is short, but it’s not plain unless you are a sailor, in which case you’d know that a fid supports the
mast on your boat or is an instrument used to pry open a tight knot in your lines
...
Yet it is a plain and simple word because of its frequent use
...

The more I thought of her insults, the more determined I grew
...

Lester is a liar
...

The runners learned that teamwork pays off in races
...

Hemingway’s own detector worked well
...
(That’s already been done by
countless imitators
...

Euphemisms
Of course there are occasions when the plainest words won’t do
...
In some contexts—a funeral service, for instance—the
verb die may be too coarse or painful
...
Think also of toilet, a word almost never posted on a public bathroom door
...
C
...

Euphemisms unquestionably have their place and function
...


Don’t interpret this admonition to use plain words as a reason to use the language of blogging,
IMs, or texting in your SAT essay
...

If you insist on using slang, that’s okay, but don’t use quotation marks to draw attention to the fact that
you can’t think of standard or more original words
...
After all,
colleges want to know that you can write good, standard prose
...
The language should sound
like you
...
But SAT essays encourage more casual responses in which references to yourself
are perfectly acceptable
...
But an essay that expresses the writer ’s personal opinion will sound most natural when cast
in first-person singular
...
Let your genuine voice ring out, although the way you speak is not necessarily the way you
should write
...
Consider
writing as the everyday speech of someone who speaks exceedingly well—grammatically correct and
free of pop expressions and clichés
...
Or
maybe even the way this paragraph sounds
...
Tell your garage
mechanic vaguely, “This car is broken,” and he’ll ask for more information
...
If a patient in the E
...

says, “I feel pain,” a surgeon might at least like to know exactly where it hurts before pulling out her
scalpels
...

In the first draft of an essay, Jeff S
...
Everything was going wrong
...
Things weren’t going
well in school
...
I also had lots of things to do at home and no time to do them
...
She yelled at me for all kinds of things
...
I had trouble with the speakers on my laptop, and I couldn’t pay for repairs
...

Reviewing this paragraph a few days later, Jeff realized the writing begged for more precise
language
...
The next draft
took care of that:
On a cold and rainy November day, my life was as miserable as the weather
...
In school I got a D on a history paper about the Bubonic Plague, and I
was sure that I had failed the chemistry quiz
...
I didn’t have time or energy to do it all, especially when my
mother started to pick at me about my messy room and the thank you letters I’m supposed to

write to my grandparents
...
When I tried to watch Harry Potter and the Deathly Hallows Part
2 again, the sound was distorted
...
On the phone
the repairman said he would charge $100 just to look at the damn thing, but I don’t have that kind
of money
...
M
...

In this version Jeff included many precise details that vividly illustrate the wretchedness of that
miserable day
...
But an essay consisting solely
of abstractions will leave readers at sea
...
But they often cover up a lack of clear and
rigorous thinking
...
” But what readers should be told is precisely why you think so
...
” A student who calls her teacher “ugly” sends a different image of
ugliness to each reader
...

Or if the teacher ’s personality is ugly, show her ill-temper, arrogance, and cruelty as she curses her
hapless students
...
To write precisely is to
write with pictures, sounds, and actions that are as vivid on paper as they are in reality
...
As the following pairs of sentences illustrate,
precise writers turn hazy notions into vivid images:
Hazy:
Precise:

Hazy:
Precise:

Hazy:
Precise:

Skiing is a fun sport
...

On the ski slope, I marvel at the snow-decked pines and brilliant sky and thrill to the challenge of weaving
gracefully down steep mountains
...

Carolyn snarled, “Get out of my face” as she punched the Tigers’ goalie in the nose
...

The letter thrilled my parents
...


Clearly, the precisely worded sentences are richer than the hazy ones
...
In fact, it’s not always desirable or necessary to define every abstraction with precise details
...
When you use an
abstract word in an essay, ask yourself what is more important—to give readers a more detailed
account of your idea or to push on to other, more important, matters
...
Just remember that nobody likes reading essays that fail to deal concretely with
anything
...
Please provide the
verbal antidote to their vagueness
...
The barn was old and run-down
...
She did not take it lightly when the accusation was leveled against her
...
Winning the overwhelming approval of the people gave the candidate great satisfaction
...
She tried diligently to study, but one could see that it made no difference
...
The atmosphere at the graduation party was intense
...
One must do many things to earn a place on the roster of an athletic team
...
It’s rewarding to visit places where customs are different because unusual customs are always
interesting
...
She met with little success during her high school career
...
The family was very poor
...
In a perilous situation, Rod showed that he was brave
...
So, do your SAT
readers a favor by giving them a verbal surprise
...
(Ha! SAT readers doing
cartwheels—that’s kind of a surprise, isn’t it?) It takes courage and imagination to use fresh language,
but here’s a guarantee: A verbal surprise may not turn SAT readers into acrobats, but it will
unquestionably give your essay a boost
...
That
doesn’t mean use odd words like twit or fop
...

Moreover they’ll sound more natural
...

I was ten before I met my first pigeon
...

Ordinary:
Surprising:

The shark bit the swimmers
...


Changing the verb makes a common sentence surprising because dined suggests gentility and good
manners, qualities that most sharks lack
...

The gunshot filled the sky with frightened pigeons
...
In the revision,
the shot becomes a vital force with the power to fill the sky
...

Words can also surprise readers by suggesting certain sounds
...
Yawn has a wide-open sound that can be stretched out indefinitely
...
Murmuring streams evokes the
sound of—what else?
Readers find unexpected pleasure, too, from the repetition of sounds—both consonants and vowels,
as in the dark, dank day smelled of death; the machine sucked up sewage from the swamp; and the cold
wind moaned over the ocean waves
...
In short, an occasional treat for the ears will go far to
captivate your readers
...


SURPRISE WITH COMPARISONS
English is filled with wonderful words to describe virtually anything
...
At such times, you can depend on figures of speech such as
metaphors and similes to make meaning clear
...
That bus driver,
for instance may have looked at you “as though you were something on the sole of his shoe
...

In addition, comparisons are economical
...
To describe elderly men fishing from a pier, for instance, you might mention their
lined faces, the folds of papery skin at their throats, the pale and cracked lips, and the white stubble on
their chins
...
Instantly your reader will see the resemblance: Gray men lined up on the
pier like boards on a weather-beaten fence
...
By
nature, therefore, they make up comparisons: “Daddy, when my foot goes to sleep it feels like ginger
ale
...
” As people grow older, they often lose
this knack of making colorful comparisons and have to relearn it
...

Similes (Tim wrestles like a tiger) and metaphors (Tim is a tiger) point out likenesses between

something familiar (tiger) and something unfamiliar (how Tim wrestles)
...
Therefore, comparing the cry of the
Arctic tern to the song of a tree toad won’t enlighten a reader familiar with neither water birds nor
tree toads
...


TIP
Clichés belong in the clichés graveyard, not in your essay
...
Don’t rely on old stand-bys such as “life is like a box
of chocolates,” or “like a bat out of hell,” or “dead as a doornail
...
The
fact is that every familiar combination of words, such as “I could care less” or “you’ve got to be
kidding” or “what a bummer,” was once new, cool, even poetic
...

Let clichés rest in the cliché graveyard
...
That is an
admonition easier to say that to follow because clichés crowd our conversations, swamp our
airwaves, and deluge the media
...
In an essay,
however, especially one that is supposed to demonstrate your unique cast of mind, you must avoid
clichés like the plague
...
No
doubt you could add many more
...
Get into the habit, then, of purging all trite
phrases from your writing vocabulary
...
The other half—expelling them—is still to be fought and won
...
Because you see the world
differently from everyone else, you can invent memorable comparisons that no one—not
Shakespeare, not Milton, not Whitman, nor any other immortal—ever thought of
...
Avoid clichés
...
as comfortable as


2
...
as gorgeous as


4
...
as serious as


6
...
as wild as


8
...
as impetuous as


10
...
If you can’t think of one, try one of these:
In what ways is life like a river? A carousel? A hero’s journey?
How does school resemble a zoo? A shopping mall? An airport?
How is music like a clearing in the woods? A chapel? A painting?











(Add paper, if necessary
...
But readers prefer a variety of sentences
...
But variety that clarifies meaning or gives emphasis to selected
ideas is something else
...


TIP

Varied sentences can bring a dull essay to life
...

The sentence is simple because it contains one grammatical subject (Terry) and one verb (fell )
...

Compound : The competition is stiff, but it won’t keep Mark from winning
...
Other coordinating conjunctions used in compound sentences are
and, yet, or, for, nor, and so, as in:
The competition is stiff, and Mark is worried about winning
...

Notice that the structure of each of these compound sentences gives roughly equal emphasis to
its two main ideas
...

The sentence is complex because it is made up of two parts—a simple sentence (Mark will win)
and a clause (Although he has a bad cold) that is not a complete sentence in itself but depends on
the simple sentence for its meaning
...
Subordinate clauses contain ideas related to the
complete sentence (called the independent, or main, clause), but they are usually less important
...

Not every simple, compound, and complex sentence is structured in the way just described
...

Most simple sentences start with the grammatical subject followed by the verb, as in:
Cats (subject) fall (verb) asleep in about three seconds
...

I (subject) wish (verb) to be a cat in my next life
...
To be sure that you write in a more mature and engaging
way, analyze one of your recent essays
...
Try leading off with a prepositional phrase, or with an adverb,
adjective, or some other grammatical unit
...
Bennett is one of the most popular teachers in the school
...
Bennett is one of the most popular teachers
...
Bennett) has been moved further along in the
sentence
...

Enthusiastically, she taught the novel The Grapes of Wrath to our eleventh-grade English class
...

Before:
After:

Students were less excited about the book than she was
...


Well, here the subject (students) is stated after an opening connective
...

Although the book turned out to be an intolerable drag for most of my classmates, I loved it
...

Before:
After:

Ms
...

To make the book more meaningful, Ms
...


To revise this sentence the writer begins with a verbal, in this case “to make,” the infinitive form of
the verb
...
Verbals, though, come
from verbs, hence their name and their resemblance
...

Hoping that it would never end, I read the book in two days
...
The -ing ending often indicates that a word is a participle
...

Awed by the tenacity of the Joad family, I was absorbed by every soul-stirring syllable of the story
...

Still another variation to try now and then is the sentence constructed from matched ideas set in
juxtaposition
...

The power of such sentences lies in the balance of parallel clauses
...
Another famous example, from Shakespeare’s Julius
Caesar:
“Not that I loved Caesar less, but that I loved Rome more
...
Out of context a sentence in

which the predicate precedes the subject may seem awkward
...
“Dull the book is not” packs more wallop than “The book is not dull” or
“The book is exciting
...
” Inverted sentences should be used
sparingly, however
...

No rule of thumb says that a certain percentage of sentences in an essay ought to be different from
the usual subject–verb structure
...
But if you find
yourself repeating the same sentence pattern, restructure some of your sentences
...


Sentence Types
Our language offers a rich menu of sentence types
...
(Just to refresh your memory, a declarative sentence, such as the one you are now reading,
simply makes a statement
...
Take
interrogative sentences, for example
...

Don’t forget about imperative sentences (Keep in mind that imperative sentences make requests or
give commands) and exclamatory sentences (What strong emotion an exclamatory sentence can
express!)
...
Direct and indirect quotations are
useful, and on occasion you can drive home a point with a single emphatic word
...
Be guided by what expresses your ideas most
clearly and seems varied enough to interest your readers
...

“You can say that again!”
“Repetition cuts both ways: some…”
“Okay, okay, you’ve made your point
...
When your sweetheart says, “I
love you
...
If a coach admonishes his
team, “Okay, guys, knock it off
...

The following paragraph may suggest that the writer has a one-track mind:
In the fall Bethany will be going to college
...
She is
psyched to break away from her small town and live in a big city
...
But first, she is psyched to
take the SAT
...
Yet, the overall effect is anything
but monotonous
...
Repeating the verb psyched
five times emphasizes Bethany’s frame of mind
...

Or take this passage written by an incorrigible bagel freak
...
I stop at the bagel shop on my way to school each
morning and grab an onion bagel and coffee
...
At snack time I’m not picky
...
Before bed I wash down a plain
toasted bagel with a glass of milk, and in case I have insomnia, I stash two or three garlic bagels
on my nightstand for a tasty middle-of the-night pick-me-up
...
But the repetition won’t allow you to
forget the point—that the writer has eyes not for pizza, not for burritos, not for onion rings, but only
for bagels
...
Therefore, stay
alert for accidental repetition:
In a corner of the room stood a clock
...

Columbus made three ocean voyages
...

Combining such sentences will keep you from ending one sentence and starting the next one with the
same words:
The clock in the corner of the room said four
...

Sentences can also be marred by words or sounds that draw attention to themselves:
Maybe some people don’t have as much freedom as others; but the freedom they do have is
given to them for free
...

The members of the assembly remembered that November was just around the corner
...
Had they read their sentences aloud, they
may have noticed that voices were stuck in a groove
...
(Hold it! Those two words—aloud and allows—sound jarring
and should not be permitted to stand side by side
...
Whenever possible, let each of your practice essays cool for a

while
...
Hearing it in another ’s voice lends objectivity to the
process of self-evaluation
...
Long
sentences (like this one) demand greater effort from readers because, while stepping from one part of
the sentence to the next, they must keep track of more words, modifiers, phrases (not to speak of
parenthetical asides), and clauses, without losing the writer ’s main thought, which may be buried
amid any number of secondary, or less important, thoughts, while short sentences are usually easier
to grasp
...
Take, for example, the last sentence in this passage:
For three days, my parents and I sat in our SUV and drove from college to college to college
in search of the perfect place for me to spend the next four years
...
But
mostly, we fought
...
Indeed,
its purpose is to startle the reader
...

Overuse dilutes its impact
...
A balance works best
...
Here, to
illustrate, is an overweight sentence that needs a complete makeover:
In the 1870s, the archaeologist Heinrich Schliemann dug in the correct spot and discovered not
only one ancient city of Troy, but nine of them, one lying on top of the other, since every few
centuries a new city had been built upon the ruins of the old, causing Schliemann to dig right past
the layer containing the ruins of the famous city of the Trojan Horse without realizing he had
done so, a mistake not corrected until almost fifty years later by Carl Blegen of the University of
Cincinnati, by which time, unfortunately, it was too late for Schliemann because he had been dead
for forty years
...
Cut it down to size
...

When you’re done, the restyled sentence might sound something like this:
In the 1870s, the archaeologist Heinrich Schliemann dug in the correct spot and discovered not
only one ancient city of Troy, but nine of them, one lying on top of the other
...
Without realizing it, he
had dug right past the layer he was seeking, the layer containing the ruins of the famous city of
the Trojan Horse
...
By then, however, it was too late for Schliemann
...

Likewise, a string of four or five equally long (or short) sentences can be combined to create a
more balanced and varied paragraph
...
It belonged to the Roman Empire
...
In 79 A
...
, the volcano on Vesuvius erupted
...
In
less than a day, the city was buried
...
More than seventeen centuries later, an Italian
peasant found Pompeii
...
He was digging in a field
...
That was two hundred years ago
...
About two-thirds of the city has been unearthed
...

With repetition eliminated and some ideas subordinated to others, here is what you get:
The ancient Roman city of Pompeii lay near the base of Mt
...
In 79 A
...
, Vesuvius
erupted, burying the city with tons of hot, wet ash
...
More than
seventeen centuries later, an Italian peasant digging in a field with a shovel accidentally struck
the top of a wall
...
Today, two hundred years later, the city is still being
unearthed
...

For more details and practice in sentence combining, turn to Part V
...

Create variety by starting sentences with a:
Prepositional phrase: From the start, In the first place, At the outset
Adverbs and adverbial phrases: Originally, At first, Initially
Dependent clauses: When you start with this, Because the opening is
Conjunctions: And, But, Not only, Either, So, Yet
Adjectives and adjective phrases: Fresh from, Introduced with, Headed by
Verbal infinitives: To launch, To take the first step, To get going
Participles: Leading off, Starting up, Commencing with
Inversions: Unique is the writer who embarks…
Balance long and short sentences
...

Dismember very long sentences
...
Divide some of the long sentences and
combine some of the short ones
...


1
...
Finn is the teacher
...
He runs the class like a dictatorship, however
...
” He knows nothing about freedom
...


He doesn’t allow talking
...
He won’t permit the wearing of hats
...
After-school detention is a consequence of lateness to class
...
A girl once came without homework
...
Finn lowered the boom
...
No one dares to come unprepared to class
...
I have taken numerous science classes
...
We
didn’t do experiments
...
We had to make do with obsolete equipment
...
The theories were not practiced in labs
...
The science department needs $1 million
...







3
...







4
...
The planet has not changed
...







5
...
It has different meanings for different
people
...
It also means getting married
...
The home has a white picket fence and a two-car
garage
...
They say that the dream should also
include a good education, friends, a feeling of well-being, good health, and above all, the
blessings of liberty
...
The dream
must also have the freedom to choose to be part of an untraditional family made up of same-sex
partners or any other combination of adults and children
...


Because it comes last, the final sentences of your essay should be written with care
...
When an essay is short to begin with, it’s insulting to review for
readers what is evident on the page in front of them
...
Trust them to
remember what your essay says
...
A good essay can easily be spoiled by a grating conclusion
...
Also stay away from endings that are too common or cutesy, such as: that’s
it; so long for now; happy reading; well, I can’t think of anything else; sorry, I’ve run out of time;
good-bye and God bless you
...

A short ending is preferable to none at all
...

There are no guarantees, of course, but readers are bound to be touched by a memento of your
thinking, your sense of humor, or your vision
...

Here are some common techniques for writing conclusions:
1
...

Topic: King of the World, a biography of Muhammad Ali
Purpose of the essay: To criticize David Remnick’s biography of Ali
...

Conclusion: With this book, Remnick has dealt Ali’s admirers a cruel blow below the belt
...
The essay concludes with an
anecdote about an elderly gray-haired man of about seventy on a crowded city bus
...
At

one point a young, gum-chewing woman stood up and pointed to the unoccupied seat
...

He looked at her in amazement
...

2
...

Topic: Surviving high school
Purpose of the essay: To describe an incident in which the writer found herself in need of a safe
haven
...

Topic: Electronic gizmos
Purpose of the essay: To show that, because many consumers are uninformed, they waste lots of
money when purchasing the latest digital devices
...

3
...
If appropriate, add a short
tag line, a brief sentence that creates a dramatic effect
...

Conclusion: As long as positions of authority are given to sexists, women must be prepared to
fight against gender abuse in this institution
...
The
writer concludes with a popular texting symbol
...
Bring your readers up to date or project them into the future
...

Topic: Vandalism in school
Purpose of the essay: To condemn the daily carnage of smashed windows, graffiti, and broken
ceiling tiles
...

Conclusion: How long can this go on? How can we turn away meekly? How much longer can we
let the vandals make us their victims?
Topic: Helping others
Purpose of the essay: To explain why it is imperative to save the world from global warming
...

So, what happens if you can’t think of a satisfactory ending or time is called before you finish? For
one thing, don’t despair
...
SAT readers will know how well you write long before reaching your
essay’s last sentence
...


Practice in Writing Conclusions

Directions: Try your hand at writing an appealing ending for each of the essays described here
...
Topic: Language taboos
Our society prohibits or frowns on the use of certain categories of words
...
After citing several examples, the
writer wonders about language usage in the future
...
Topic: The value of school sports
The writer, in comparing athletics in school to life, makes the point that in both endeavors you
need to develop a winning strategy
...
Topic: High school vs
...








TIP
The following pages are meant to be a guide to editing your SAT essay
...


EDITING AND PROOFREADING: THE FINAL TOUCHES
Once you’ve ended your essay, spend whatever time is left editing and proofreading
...


Editing for Clarity
Check your essay for clarity by asking yourself whether a reader could misconstrue anything you’ve
written
...
wrote her essay about runaway teenagers—those desperate kids who leave home in
search of a different life
...
” Coming to that sentence, a reader might well wonder whether Penny means that parents should
talk to their kids as a last resort, or, that in a list of what parents ought to do, the final step is talking to
their kids
...
” Penny certainly understood what she intended to say, but a reader can’t
tell whether Ellen took a dim view of Debbie’s actions or whether Ellen herself had second thoughts
about her own flight
...

That’s why you should work hard to arrange your words in the clearest order
...


MISPLACED MODIFIERS
Modifiers are words, phrases, and clauses that tell something about or limit the meaning of a
particular word or statement
...

The adjective broken is a modifier because it tells something about the condition of the window
...

Jessica bought a mouse that was guaranteed to work with her computer
...
It modifies the noun mouse
...

Here only modifies the verb loves
...
If, however, Mike has but one
love, and she is Sharon, then only is misplaced
...
or Mike loves Sharon only
...

In this sentence, when she had finished the essay is the modifier
...
If it modifies decided, Naomi finished her essay and then made a decision
to watch TV
...

When she had finished the essay, Naomi decided to watch TV
...

Now the meaning of both sentences is unambiguous
...
To avoid the problem, place
modifiers as close as possible to the words they modify:
Misplaced:

Philip donated his old car to a charity that no longer ran well
...

Clear:
Misplaced:

Philip donated to a charity his old car that no longer ran well
...


The modifier of all sizes should be closer to shoes, the word it modifies
...


DANGLING MODIFIERS
In a sentence words must fit together like pieces of a jigsaw puzzle
...

(1) While running to English class, the bell rang
...

(3) When only eight years old, my father warned me about smoking
...
Do you
see that these sentences describe a surreal world in which bells run to class, summers hold full-time
jobs, and youthful fathers dispense advice? The problem is that these sentences try to mate two groups
of words that can’t go together
...
After the comma in sentence 1, you expect
to find out who is running, but you are not told
...
In short, you’re left dangling
...
To correct the error, add the
noun or pronoun to be modified, as in:
While the boys were running to English class, the bell rang
...

When I was eight, my father warned me about smoking
...

My mother thought I might be sick because I was still sound asleep at noon
...

While I talked on the phone, the stew burned in the pot
...
Some
sentences may be corrected by shifting the placement of one or more words
...


1
...



2
...



3
...



4
...



5
...



6
...



7
...



8
...



9
...



10
...



PARALLELISM
A lack of parallelism in phrases and clauses is not just bad form but can be a source of confusion
...
Take, for
example, a sentence that lists the characteristics of a restaurant in which to have a family birthday
party:
We are looking for a place that is private, plenty of space, has a friendly staff, and that people like
to look at
...


The sentence makes some sense, of course, but it’s awkward because the four qualities of a desirable
restaurant are not expressed in parallel form
...
One way to fix the problem is to use only adjectives, as in:

We are looking for a place that is private, spacious, friendly, and attractive
...

When you arrange the pieces of a sentence in parallel form, the writing becomes clearer and
stronger
...
Abraham Lincoln,
for example, used parallelism at Gettysburg: “We cannot dedicate, we cannot consecrate, we cannot
hallow this ground
...

John F
...

Like Lincoln and Kennedy, good writers everywhere know and apply the following principles of
parallel construction
...
Parallel ideas in a series should be expressed in the same grammatical form
...
Use conjunctions such as and, but,
for, or, yet, so, and nor to join parallel ideas
...

Hazel’s parents objected to the loud music she played and to the late hours she kept
...

2
...
In a
comparison, for example, an idea expressed in a phrase must be paired with another idea also
expressed in a phrase
...

Faulty:

They are worried more about public opinion than for what the effect of the proposal may be
...

Parallel:

They are worried more about public opinion than about the effect of the proposal
...
Parallel ideas can also be expressed with pairs of words such as either/or, neither/ nor, whether/or,
both/and, and not only/but also
...

Poor:

I either plan to invite my aunt or my uncle to go shopping with me
...
Its
placement misleads the reader into thinking that the verb plan is one of the parallel ideas
...


4
...

Unclear:

Our mechanic did a better job on my car than his
...


Sometimes repeating both a preposition and an article is necessary:
Unclear:

Before signing the contract, Tiffany spoke with the president and treasurer of the company
...


5
...

Absurd:

Terry is six feet tall, kind, and a Texan
...

Less absurd:

Terry, a six-foot Texan, is kind
...

Absurd:

On Sunday, Meredith not only painted her toenails but got married
...

Regardless of what you may have been told, painting toenails and getting married are not
equivalents
...
or On Sunday, after painting her
toenails, Meredith got married
...


Practice in Identifying Parallel Structure
Directions: Look for faulty parallelism in the following sentences
...
Some sentences may be correct
...
Mr
...



2
...



3
...



4
...



5
...



6
...



7
...



8
...



9
...



10
...



11
...



12
...



13
...



14
...



15
...



16
...



17
...



18
...



19
...



20
...



Editing for Interest
Your essay will be read by people—real people
...
Like any readers, they will be put off by writing that is dull
...
Because active verbs describe or show movement, they excel
all other words in pumping life into your prose
...

Being verbs, in contrast, have almost no life in them
...
In fact, a being verb functions
much like an equal sign in an equation: “Five minus two is three” (5 – 2 = 3), or “Samantha was
happy” (Samantha = happy), or “Your SAT scores are going up” (That = good news!)
...


TIP
Whenever possible, replace being verbs with active verbs
...
If
more than, say, one out of four sentences uses a form of to be as the main verb, try the following
revision techniques
...

Most students struggle to write immortal essays
...

Monica won the essay contest
...

I enjoyed my afternoon at the ballgame
...
When that happens,
try turning subjects into verbs and verbs into nouns
...
A fullscale revision will often result in sentences that bear little resemblance to the original
...

You may notice, however, that some nouns limit your options for using active verbs
...
There are few verb options, too, when
the subject of the sentence is thought, concept, idea, issue, way, cause, and several other abstract
nouns
...

In contrast, nouns that stand for specific places, people, events, and objects invite the use of active
verbs
...

As a bonus, concrete, easy-to-define nouns, when substituted for abstractions, tend to tighten and
energize sentences:
Abstract subject:
Definite subject:

The cause of Sharon’s worry was her lack of tuition money
...


Abstract subject:
Definite subject:

The issue behind the strike was the workers’ demand for higher wages
...


Being verbs are not the only verbs that sap the life out of sentences
...
Webster’s International
Dictionary lists sixteen different meanings for the verb get and a dozen more for make and move
...
Otherwise, trade them in for more vivid verbs, as in:
Dull:
Livelier:

The line to the box office moved very slowly
...


Note that by using a more animated verb, you eliminate the need for “very slowly,” which has
suddenly become redundant
...

The police officer permitted drivers to turn left on red
...


Practice in Using Active Verbs
Directions: Replace the weak, lifeless verbs in these sentences with stronger, active ones
...
Shock was the feeling of most American people from the attack of 9/11
...
In New York City, there were nearly three thousand people killed
...
Afterwards, there was a controversy over who was to blame for America’s vulnerability to
terrorism
...
There was an effort made to strengthen homeland security
...
Many people were willing to give up some of their rights in order to be secure
...
The issue of how much freedom to give up for the sake of security is difficult to resolve
...
The war in Afghanistan was a significant event that was a result of 9/11
...
Sweatshirts and baggy pants was our manner of dress whenever we went out
...
There was quite a lot of commotion because of there being an all-American high school
basketball player playing in the game
...
It is obvious that there should be more emphasis on math and science for the average collegebound student
...
In an active sentence the person or
thing performing an action is usually mentioned early in a sentence so that readers know right away
who or what you are talking about
...

That’s when a passive sentence—a sentence structured in the passive voice—is more appropriate
...

At 8:30 sharp, a stagehand (or Maryanne, the production assistant) raised the curtain
...
Who pulled the rope or pushed the button is
beside the point
...

The agency’s computer system was hacked into by Tommy, an eight-year-old boy
...

Active:
Passive:

Sorry, I lost the library book
...


• When you want to avoid using gender-based pronouns
...

Uniforms must be returned by every member of the marching band
...

This sentence needs revision because it fails to tell who performed the action—that is, who prepared
for the carnival
...

This version contains more information than the original, but it still emphasizes the action instead of
who performed the action
...

In the active voice, this sentence gives the performers of the action top billing
...
Somebody does something; a person or thing acts
...
Marriages don’t just happen; couples deliberately go out and

marry each other
...
People do these things
...

The town Rotary Club presented an award to Carrie
...

My brother, my cousin, and three uncles went to Annapolis
...
Change them to active
sentences unless you have a good reason not to
...


1
...



2
...



3
...



4
...



5
...



6
...




7
...



8
...



9
...



10
...



OMITTING NEEDLESS WORDS
Never use two words when one will do
...

Brevity works best
...
Readers value economy
...
Omit needless words!

Hold it, again! Look at that last phrase, briefly and succinctly
...
The author
should have used one, but not both, adverbs
...

In truth, that’s twenty-four words to the wise—probably more than are needed
...

That’s better—eleven words of free advice, but still too many
...

This seven-word model is less than a third of the original twenty-four word clunker
...

Your sentences, like muscles, should be firm and tight
...
To trim
fat, wring your sentences through this four-step word trimmer:
1
...
Then combine sentences
...
Jesse is Elena’s brother
...
(8 words)

Granted, cutting ten words to eight may not seem like much
...

Fat:
Trimmed:
Re-trimmed:

When Maria was sixteen years of age she accepted a position at Wilkens’ Fabrics
...
(27)
At sixteen years old, Maria accepted a position at Wilkens’ Fabrics, where she learned about fabrics
and handling customers
...
(15)

2
...
They often signify the presence of fat
...
(12)
The wonders of electricity obsessed Edison
...

Fat:
Trimmed:

What he most wanted was that the terrorists would release the hostages
...
(9)

3
...

Fat:
Trimmed:

Hamlet returned home as a result of his father’s death
...
(7)
...
(16)
The mine field endangered the troops
...
(12)
Use an awl
...
(22)
His grandfather suffered from bipolar disorder
...
Search for redundancies
...

Fat:

A cloud of black soot rose up to the sky
...

Trimmed:
Fat:

A cloud of soot rose to the sky
...
(7)

Where else but on a face would a smile appear?
Trimmed:
Fat:

He wore a smile
...
(16)

Scrutinize means “to study carefully,” and engrossed means “to think fully
...

Trimmed:

After scrutinizing the X-ray, the doctor seemed engrossed in thought
...
Even though it may hurt to take out
what you worked hard to put in, don’t whine
...


1
...




2
...




3
...




4
...
Harmful
criticism is criticism that tears a person down instead of helping the person overcome or deal with
a problem
...
Every American should have a good knowledge of our country, and the best way to gain a good
knowledge and familiarity with the United States is to visit and see places of historic interest and
significance to our country
...
Please trim
its fat
...
They are the latest, most upto-date set of procedures available anywhere
...
Yet, there are many people of all kinds
who need to gain weight for a variety of diverse reasons
...















For additional practice in eliminating wordiness, turn to Part V
...
TELLING
Remember the principle that a picture is worth a thousand words? Whether that’s true is arguable, but
the point is not
...
Therefore, show more than you tell ! Instead of describing
your uncle as “absent-minded,” show him stepping into his morning shower dressed in his pj’s
...
The
same principle applies to smells: “Her breath was foul with the stench of stale whiskey
...
” To touch: “the feel of cool linen bed sheets
...
” In short, showing recreates
experience for the reader, ultimately making the prose more interesting
...
Blair
...
Blair’s office, bounded down the steps four at a time, and shouted into the wind,
“Hurray, I did it
...
Franks, doesn’t care to hear that I don’t have the time to do math homework after
school
...
Franks that I’m kept from math homework by driving my brother Timmy to
piano lessons or karate, by yearbook meetings on Tuesdays, by Peer Leaders and Students Against
Driving Drunk, by French tutoring, and by a part-time job at the florist, he muttered, “That’s your
problem
...
The fact is that writers struggle for years to perfect the technique
...
A balance is best
...
You need time to get the feel of it, like riding a bike or doing a back flip
...
To
develop the knack, study a written passage that you admire
...

For practice, use the passage as a model for writing a paragraph of your own
...


1
...


2
...


3
...


4
...


5
...


6
...


7
...


8
...


9
...


10
...


Checking for Standard Usage and Mechanics
Practice these guidelines to minimize writing errors:
• Write correct sentences
• Use correct verbs
• Use adjectives and adverbs correctly
• Choose correct pronouns
• Correct punctuation and capitalization

WRITE CORRECT SENTENCES

Time won’t permit you to analyze meticulously every sentence in your SAT essay
...

To avoid these common errors, always look for the noun or pronoun that functions as the
grammatical subject of the sentence and for the verb that it goes with
...
), in which
case the subject is understood to be the addressee—you, the dog, a slowpoke…whomever
...


USE CORRECT VERBS
Of all the parts of speech, verbs are the most apt to be used incorrectly
...
Do all nouns and pronouns agree in number with their verbs?
2
...
Is every verb in the correct form?
Learn to answer these questions accurately by studying the following sections of Part V: subject–verb
agreement; proper use of verb tenses; and correct form of verbs
...
The reverse—using
an adverb in place of an adjective—occurs less often
...

For details and practice in adjective/adverb use, turn to Part V
...
If you can’t depend on your sense of what sounds right and wrong, keep in
mind that most common pronouns fall into two groups:
Group 1: I, he, she, they, we, you, who
Group 2: me, him, her, them, us, you, whom
The pronouns in the first group are nominative case pronouns (sometimes called subjective case
pronouns) and are used in grammatical subjects and predicate nominatives
...
Because pairs must come from the same case,
problems arise when pronouns from different cases show up in the same phrase, as “Him and I went
to the movies
...
If you don’t know either pronoun, here’s a handy rule of thumb to
follow: Substitute I or me for one of them
...

For more details and a practice exercise in choosing the case of pronouns, turn to Part V
...
That is, be sure also that every
pronoun refers clearly to its antecedent—usually a noun or another pronoun
...

Because the pronoun her may refer to either the librarian or to Sarah, the sentence needs revision:
The librarian told Sarah that one of her responsibilities as a library clerk was to shelve books
...

Finally, don’t use pronouns to refer to possessives, as in:
In Eminem’s latest hit, he stumbles over several words
...

Because the possessive noun Eminem’s is not a grammatical equivalent to Eminem, the revised
sentence should be:
In his latest hit, Eminem stumbles over several words
...

Stay alert also for shifts in pronoun person within individual sentences, within paragraphs, and
within the whole essay
...
Likewise, a sentence or passage written in the
first or third person should stay that way throughout
...

Finally, take a look at the agreement between all the pronouns and their antecedents
...
Sometimes such
words are meant as plurals, however, and should be followed by plural pronouns
...


CORRECT PUNCTUATION AND CAPITALIZATION
Because error-free essays tend to earn higher scores, it pays to review your essay for proper
punctuation and use of capital letters
...
The hardest thing about the rules is
knowing where and when to apply them
...
Apostrophes are used in only three places:
1
...
Apostrophes mark places where letters
have been omitted
...
In plurals of letters, signs, or numbers, as in A’s and B’s, the 1960’s, and 10’s and 20’s, although
many experts simplify matters by writing 1960s, Ps and Qs, and so forth
...
In possessive nouns such as the student’s class and women’s room and in indefinite pronouns such
as anybody’s guess
...
Some possessive forms use both an apostrophe and of, as in a
friend of the family’s; some others that specify time, space, value, or quantity also require
apostrophes, as in an hour’s time, a dollar’s worth, at my wit’s end
...
Commas divide sentences into parts, clarify meaning, and prevent confusion
...
Use a comma to signal a pause, as in:
No pause:
Pause:

After brushing his teeth gleamed
...


Commas are needed after some introductory words and in forms of address:
Well, you can open it whenever it’s convenient
...

2
...

Jennie, on the other hand, was included
...

The three bikers, whose map of the course was out of date, arrived two hours later
...

The judge, Mr
...

3
...

Pete had better call my mother, or I’ll be in big trouble
...
Commas separate items in a series:
Rosie’s car needs new tires, a battery, a muffler, and an oil change
...

Some writers prefer to skip the comma before the last item in a series, but just in case clarity may
suffer, it can’t hurt to put it in
...
Commas separate parts of addresses, dates, and place names:

Who lives at 627 West 115th Street, New York, NY?
Richard was born on May 27, 1996, the same day as Irene
...

Note that, because each item in the last example already contains a comma, semicolons help to avoid
confusion
...
Commas separate quotations from attributions in dialogue
...

“I want it open,” protested Ben
...
Semicolons may be used between closely related sentences, in effect, shortening the
pause that would naturally occur between two separate sentences:
Mother was worried; her daughters never stay out this late
...

A caution: Because semicolons function like periods, use them only between independent clauses
or in a series in which one or more items contains a comma, as in:
On his trek, Norwood met Allen, a carpenter from Maine; Dr
...

Louis; Jonathan, an airline pilot; and me, of course
...
Quotation marks usually surround direct quotations, as in:
Rita said to Bob, “I’m nuts about you
...
Don’t use them for longer works
...

Avoid calling attention to clichés, trite expressions, or slang terms by using quotation marks
...

Finally, quotation marks may enclose words that express the silent thoughts of a character, as in:
Carlos glanced at his watch
...

Periods and commas are placed inside close-quotation marks
...

“When will the seminar start?” asked Regis
...
POSSESSIVES
Directions: Check your mastery of possessives by writing the correct possessive form of the
italicized word in the space provided
...


1
...



2
...



3
...



4
...



5
...



6
...



7
...



8
...
Both of the computers keyboards need repair
...
He’ll be back in two months time
...
COMMAS AND SEMICOLONS

Directions: In the following sentences, insert or remove commas and semicolons as necessary
...


1
...

2
...

3
...

4
...

5
...

6
...

7
...

8
...

9
...

10
...

11
...

12
...

13
...

14
...

15
...

16
...

17
...

18
...

19
...

20
...

21
...

22
...

23
...

24
...

25
...


Capitalization
Capitalization isn’t totally standardized, but it’s not a free-for-all either
...
Capitalize the first words of sentences, direct quotations, and lines of poetry (most of the time)
...

2
...

3
...
Don’t capitalize
north, east, south, and west unless you are referring to a particular region of the country, as in:
They went camping in the West
...

4
...

5
...
Don’t, however, capitalize the common noun associated with the brand name, as in Crest
toothpaste or Starbuck’s coffee
...
Capitalize titles of persons that indicate rank, office, profession, when they are used with the
person’s name: Congressman Kelly, Doctor Dolittle, Coach McConnell, Judge Judy, Lieutenant
Lawlor
...
Don’t capitalize titles
when referring generically to the position: the superintendent of schools, the assistant librarian,
the clerk of the highway department
...
Capitalize family relationships, but only when they are used with a person’s name: Uncle Wesley,
Grandma Jones, Cousin Dave
...
Capitalize titles of books, plays, stories, articles, poems, songs, and other creative works: The
Grapes of Wrath, Hamlet, “An Occurrence at Owl Creek Bridge,” “Ode to a Grecian Urn,” “Box
of Rain
...

9
...
Also capitalize pronouns referring to Him or Her
...
Capitalize historical names, events, documents, and periods: Battle of Gettysburg, Alien and
Sedition Acts, War of 1812, Bill of Rights, Middle Ages
...
Capitalize days of the week, months, holidays: Monday, May, Mothers’ Day
...

12
...
While course names are capitalized, subjects are not
...
Similarly, you

attend high school at Brookvale High School and go to college at Columbia
...


1
...

2
...

3
...
c
...
the shackleton expedition nearly met its doom on georgia island in antarctica
...
for christmas he got a black & decker table saw from the sears store next to the old bedford
courthouse
...
according to georgetown’s high school principal, eugene griffiths, georgetown high school
attracts students from the whole west coast
...
mr
...

7
...

8
...
louis
...
This irish linen tablecloth was bought at k-mart in the emeryville mall off powell street
...
yellowstone national park is located in the northwestern corner of the state of wyoming
...


Prewriting
Analyze the topic carefully
...

Choose a main idea that matters
...


Composing

Hook readers with a gripping introduction
...

Use plain and precise words
...

End the essay memorably
...

Edit for interest
...


Answer Key to Practice Exercises
Choosing a Main Idea
These are suggestions only
...

1
...
Henry Ford knew more about cars than about people, because talent, ability, and a little bit of
luck are the most important ingredients of success
...
If Henry Ford’s statement is correct, the world is filled with self-deluded people
...
From spelling bees to Nobel Prizes, nobody with a negative attitude has ever been a winner
...
A
...

B
...

C
...


3
...
Those who say “Money is the root of all evil” know what they are talking about
...
Dreaming of wealth is as American as apple pie—part of the great American dream
...
Only a ding-a-ling, or someone named Jay Gatsby, would truly believe that money can buy
happiness
...
A
...

B
...

C
...


5
...
Bryan is definitely on the right track
...

B
...

C
...
Therefore, it’s
simplistic to think that your destiny can be achieved without it
...
The order of ideas is strictly a matter of personal preference
...
Advantages:
1
...

2
...

3
...

Disadvantages:
1
...

2
...

3
...


B
...
Video games provide pleasure and entertainment
...
Many interactive games stimulate the mind and foster problem-solving skills
...
They improve hand-eye coordination
...
Video games tend to be addictive
...
They glorify violence and destructive behaviors
...
Their high cost diverts family funds from more worthwhile pursuits
...
For:

1
...

2
...

3
...

Against:
1
...

2
...

3
...


D
...
War causes people and nations to abandon the qualities that make them human
...
Wars cause death, suffering, and destruction
...
Wars cost money that can and should be used for improving lives, not destroying them
...
War against terrorism provides security for the people
...
War to depose tyrants is of benefit to mankind
...
Wars on poverty, drugs, and other social evils improve the quality of life
...
Be confident that the essay openings you wrote may be no less effective than these
samples
...
Topic: The courage of one’s convictions
Most high school kids would rather be caught dead than be considered out of synch with the
crowd
...
She would consider herself dead
if she couldn’t express her individuality and be different from everyone else
...
Topic: Deadlines
My dad recently bought me a smartphone
...
After all, I’m not a businessman or a lawyer or a
high-powered boomer who needs to keep tabs on meetings and clients and overseas flights to
catch
...


3
...

If you’ve ever seen “The Antiques Road Show” on television, you’d never throw anything away,
never hold a garage sale, never pass up an opportunity to buy an old toy, an ashtray from the
World’s Fair, a miniature Statue of Liberty, or any other piece of junk to fill your shelves and
clutter up your closets
...


4
...
Louis, an ocean liner bound for safety in Cuba
...
The United States also turned the St
...
With no place to go, the
ship sailed back to Europe
...


5
...
Sentence 1

2
...
Implied topic sentence

3
...
Sentence 10

5
...
Sentence 1

7
...
Sentence 3

9
...
Sentence 6

PART B
Answers may vary
...
Mother and Father are very different from each other
...
In the past U
...
athletes dominated the Olympic Games
...
Monopolies often destroy not only themselves but the incentive of businesses to change and make
progress
...
How little the aristocracy understood the needs of the masses
...
Vera Simon wrote a gripping and realistic book
...
Smoking in school is just not worth the trouble it can lead to
...
But here are my requirements for the perfect roommate
...
Age and experience have deprived me of courage and spirit
...
No topic sentence is needed
...
Although backward in some respects, a so-called primitive culture can be technologically
sophisticated
...
The topic sentences you wrote may be as good as or even better than these
examples
...
Of all the equipment needed to traverse the inhuman land of Antarctica, nothing is more important
than a team of well-trained sled dogs
...
Antarctica takes your breath away
...
This is not an idle comparison, because at every turn you are putting your health and safety in
jeopardy
...
a
...
5
c
...
2
e
...
a
...
4
c
...
2
e
...
a
...
1
c
...
5
e
...
a
...
3
c
...
1
e
...
a
...
3
c
...
4
e
...
Sentence 3 destroys the coherence of the paragraph
...
There’s no reason to save it, because
the idea is reiterated in sentence 7
...
The paragraph lacks unity
...
One way to overcome the paragraph’s lack
of unity is to divide it into two parts
...
g
...

If this were done, however, the paragraph would need furtherdevelopment
...
The paragraph is coherent except for sentence 5, which should be deleted
...
The remaining sentences, except 5, support sentence 2, which is the
major supporting sentence in the paragraph
...
Although the entire paragraph discusses political parties, the discussion is not unified
...
Either divide the
paragraph, or add a topic sentence that justifies discussing both topics within a single paragraph
...
Sentence 1 is the topic sentence
...
Delete it
...
The paragraph is mostly unified and coherent, although the topic sentence would be more accurate
if it mentioned the human qualities of porpoises
...
Although the opening sentence leads the reader to think that what follows will be all about
Robinson Crusoe, the paragraph is really about the author Daniel Defoe
...


8
...
Only sentences
3 and 4 connect with each other
...
For coherence, add a topic sentence, possibly using material in sentence 5
...


9
...
Delete sentence 6, but if the idea is
too good to discard, save it for another place in the essay or revise sentence 1, the topic sentence
...
The paragraph is unified and coherent
...


Using Transitions
These paragraphs only illustrate the use of transitions
...

1
...
First you must try to create the impression that you think, say, Ms
...
You must immediately choose a seat that is near to her in the classroom
...
Next, smile at her, laugh at her jokes, and never leave the room right
after class
...
After a while, she’ll think that you are an intelligent, highly motivated
student and with luck will reward you handsomely on your report card
...
Some people are bored with their lives
...
Accordingly, many movie stuntmen ache to put their lives in jeopardy
...
As a consequence, they often get hurt, but their work is more important
to them than their safety and well-being
...


3
...

Unlike my mother, my father is always pretty upbeat, even when he’s worried about his job, about
money, and about me and my sister
...
In spite
of their differences, Dad and Mom get along just fine
...
Nevertheless, I can see where Mom is coming from and love her all the same
...
It’s time to reconsider how the United States squanders billions of dollars every year on probing
Mars, Jupiter, and other remote places in outer space
...
In addition to being a misuse of money that is sorely needed
to solve problems here on the Earth, studying outer space has been less fruitful than predicted
...
What’s more, the cost of developing technology required to make worthwhile journeys
even to the closest planets or asteroids is, if you’ll pardon the pun, “astronomical
...


Using Precise Wording

Answers will vary
...

1
...


2
...
After winning the election by a 3 to 1 margin, the senator grinned from ear to ear and told her
supporters that she was ready to work in their behalf
...
Molly’s reward for six hours at her desk studying physics was a big fat F on the quiz
...
The seniors celebrated their graduation but wept inside, realizing that tonight was the last time
they would ever be together
...
To make it on the swim team, the bowling team, or any other team, there are but three things to do:
practice, practice, practice
...
At the wake, Greg was startled by the joviality of the mourners, who rejoiced over Mr
...


8
...


9
...


10
...


Writing Comparisons
The comparisons that you wrote may be as good as or even better than these examples
...
1
...
as tough as a wrecking ball

3
...
as silly as putty

5
...
as perfect as a circle

7
...
as unpredictable as the lottery

9
...
as reliable as a sheepdog

B
...
Just as no one goes to the airport just to be at the airport, who would go to school in order to
go to school
...
One
spends a certain amount of time there, follows the rules, does the work, and then escapes like a
traveler en route to Aruba, or Italy, or the Far East
...

If you fail to follow the prescribed procedure, trouble can follow, delaying your departure
...


Varying Sentences
These are illustrative answers only
...

1
...
Finn is a good teacher but he runs the class like a dictatorship
...
On the first day he announced his rules, among them no talking, no gum
chewing, no hats in class, no lateness
...
All homework is compulsory
...
Finn lowered the boom on her
...
In the numerous science classes that I have taken, we talked about experiments instead of doing
them because the equipment was obsolete and too costly to replace
...
To give students a better education,
the science department needs money
...


3
...
They are making beaches and swimming dangerous
...


4
...
Earthquakes, droughts,
huge storms, and floods are not happening more frequently, however
...
Rather, the population of the world has increased
...
The planet has not changed but humans have
...
Although the American Dream is a popular concept, it means different things for different people
...
Some people, thinking that dream
shallow, say that the dream won’t be complete without a good education, friends, a feeling of
well-being, good health, and above all, the blessings of liberty, including the freedom of speech
and religion and the freedom to choose to be part of an untraditional family made up of same-sex
partners or any other combination of adults and children
...

1
...


2
...
Others succeed because they are more talented or
smarter than the competition
...
When a split second determines the winner in a race, is it fair to say that the
second-place finisher is not as good as the winner? No, but it’s a certainty that the winner planned
his racing strategy better than the person who lost
...
If by magic I happened to find myself in junior high again, I wouldn’t rest until I’d made my
escape
...

1
...


2
...


3
...


4
...


5
...


6
...


7
...


8
...


9
...


10
...


Identifying Parallel Structure
These are suggested answers
...

1
...
was accused not only of being a bigot but also of being too stupid

3
...
preparing reports, and making various types of telephone calls

5
...
and she had a habit of disappearing

7
...
felt both pride and satisfaction

9
...
plans to live simply

11
...


12
...
Joan’s broken yellow pencil came from this box
...
how to furnish and decorate the house simply

15
...
and being miles from friends

17
...


18
...
and a job in the suburbs

20
...

1
...


2
...


3
...


4
...


5
...


6
...


7
...


8
...


9
...


10
...


Revising Passive Sentences
Your sentences may differ from these, but be sure you’ve used the active voice
...
Dead leaves covered the backyard
...
We discussed the crisis in the Middle East
...
I failed Friday’s quiz because I had rehearsed for the play every night that week
...
We began our weeklong vacation in Oregon by flying to Portland
...
Captain Ahab and his crew pursued the great white whale
...
Fido fetches the newspaper every morning
...
The president and his advisors decided to go to war
...
On Friday night, more than twenty customers took out dinners
...
In three days, our group saw five Shakespearean plays
...
Before you do something physical, the body normally calls on the brain
...
Be sure that your version of each sentence approximates the meaning of the
original
...
Peter Jenkins wrote A Walk Across America, a book about his cross-country walk
...


2
...


3
...


4
...


5
...


PART B
Avoiding discrimination is but one of many reasons for people to gain weight, according to Stanford
University graduate Slim Snyder, who, at a recent conference on health, said “Lean people are victims
of discrimination, just as obese people are
...
But they are ridiculed and kept well hidden because health experts agree
that being lean is preferable to being obese
...
Check your sentences for specific details that show rather than tell
...
Whenever Mike enters a room, he ducks his head to avoid hitting the top of the doorway
...
Her sense of accomplishment grew with every handshake and pat on the back
...


3
...


4
...


5
...


6
...


7
...


8
...


9
...


10
...


Using Punctuation
PART A
...
Paul’s

2
...
Correct

4
...
girls’, boys’

6
...
Correct

8
...
computers’

10
...
COMMAS AND SEMICOLONS
1
...


2
...


3
...


4
...


5
...


6
...


7
...
The boat was seventy-five feet long and eighteen feet wide; its mast was about eighty feet tall
...
To anyone interested in flying, planes hold endless fascination
...
Jeff and Steve, left alone for the weekend, invited all their friends to a party
...
I need street maps of Boston and Portland, Maine
...
Some of the theories dealt with the political, social, and religious ideas of the time
...
Students who want to try out for the chorus have been asked to report to room 330
...
Doug, for example, is both a scholar and an athlete
...
Monica refused to go unless Phil went with her
...
The hero of the book, John Coffey, rode his bike across the United States
...
After all, she did for him what she could
...
Starting in Minnesota, the Mississippi runs all the way to the Gulf of Mexico
...
Harold Watkins, who comes from Chicago, won a full tuition scholarship to Duke
...
Although the characters are stereotypes, they were interesting to read about
...
Yo-Yo Ma, the famous cellist, will perform a recital on Saturday night
...
This test covers Spanish literature, culture, and history, and it lasts for three hours
...
Michelle is pretty, tall, and dark, but her older sister Norma is pretty, short, and light
...
Sean, the twin brother of Ian, was struck by a falling tree limb
...
The window washer dropped by last evening, but he didn’t bring his squeegee
...
On Labor Day Bennington County’s fire department plans to hold a turkey shoot on the field at
Miller ’s Pond
...
The judge gave District Attorney Lipman a book entitled The Rules of Evidence and instructed her
to read it before she ever dared set foot in the Court of Appeals of the Ninth Circuit again
...
The secretary of state greeted the president of Austria at the Ronald Reagan Airport in
Washington, D
...


4
...


5
...


6
...
At Georgetown students may major in drawing and
painting, design, graphics, or sculpture
...
Griffiths said, “I attended a similar high school in
New England just after the Vietnam War
...
We expect to celebrate New Year ’s Eve again this year by ordering a movie of an old Broadway
musical from Netflix and settling down in front of the DVD player with some Pepsi and a box of
Oreos
...
After traveling all the way to the Pacific, the Corps of Discovery rode down the Missouri River
going east on their way back to St
...


9
...


10
...


PART IV
YOU BE THE UMP: ESSAYS FOR EVALUATION
• SAT Essay Readers: What They Do and How They Do It
• Essay Scores: What the Numbers Tell You
• Sample SAT Essay Questions and Student Responses
• Essay Topics for Practice

How Essays Are Judged and Graded
Your essay will be evaluated on a scale of 6 (best) to 1 (worst)
...
That is, they’ll avoid the “Gotcha! Syndrome”—hunting down every
little error
...
Knowing that your essay
has been composed in under half an hour, they won’t hold minor mistakes against you and won’t
deduct a certain amount for every error in grammar, spelling, and punctuation
...
Like other readers, evaluators enjoy good writing and delight in thoughtful, neatly phrased
ideas
...


TIP
SAT readers won’t hold minor mistakes against you
...
When reading your essay, they’ll compare it to other essays written on the same
topic at the same time
...

Handwriting is not supposed to count in the evaluation
...
Frustration may cause them to be
unfavorably disposed toward the essay
...
When readers are forced to stop regularly
to puzzle out the words, your flow of ideas will be interrupted—with adverse effects
...
At this stage in your life, it may be difficult to change your
penmanship
...
For most people printing is slower
than cursive writing, but with practice you can increase your speed
...

Performance Categ ories
Outstanding
Very Good
Good
Fair
Poor
Very Poor

Score
6
5
4
3
2
1

Scores of 4, 5, and 6 reflect a level of proficiency in writing appropriate for first-year college
students
...
(A blank paper or a paper submitted
on a topic unrelated to the prompt will be rated zero
...
Readers are human
...
An essay assigned a score of 5 by one reader, therefore, may not be
vastly superior to an essay given a 4 by another
...
The writer has fashioned a convincing main idea,
amply supported by appropriate and specific details
...
Any errors that occur are
inconsequential
...

A very good essay demonstrates the writer ’s firm grasp of the assignment
...
It also may fall short of the
mastery, sophistication, and control of composition exemplified by an
outstanding essay
...

A good essay deals with the topic competently
...
It gives evidence of the writer ’s acquaintance with
essay organization, coherence, and paragraph development
...

A fair essay suggests mediocrity in writing
...
Although the essay has a recognizable structure, the
organization may be confusing or not fully realized
...
Occasional mechanical
errors may detract from the essay’s meaning although they don’t
necessarily obliterate communication
...
The
essay’s development is meager, and its treatment of the subject is imprecise
and unconvincing
...

A very poor essay reveals the writer ’s inability to interpret the prompt
...
It may
also be excessively brief or undeveloped
...
The writer shows little evidence of control of
sentence structure or the rules of usage and grammar
...
Then review the explanations of how scores were determined
...
They are unedited and
typed exactly as written
...

Then you be the ump
...
Write your
observations in the space provided for comments
...
Because no two people are likely to agree on every
detail, your evaluation will probably differ from those of the pros
...
What counts more than the similarity between your comments
and those of SAT readers is the overall impression left by each piece and the score you gave it
...
Then keep those criteria in mind as you plan and write essays of your
own
...

Stephen Lim tells the following story: “Working my way through school, I delivered special
delivery mail for the U
...
Postal Service
...
Before starting our route, we recorded the number of stops we had to
make
...
This made me look bad in
comparison, lowering the supervisor ’s opinion of my performance
...
But others can suffer more serious
consequences for being honest
...
Job
applicants don’t succeed in getting a coveted position because they refuse to pad their résumés
...
“After a while, you feel like a jerk when other people are getting ahead by taking
shortcuts
...
Support your position
with evidence and reasoning drawn from your studies, reading, experience, or observation
...
It involves no physical labor, no
strenuous activity, no expenses, and no special skills
...

Lying is a major part of getting a job you are not totally qualified for
...
Just in case you are asked during the interview what the award was for,
you prepare a lie ahead of time, maybe something like it’s an award for integrity, for being an
extremely honest and trustworthy person
...
So you fabricate a little more to show that you are used to hard work and
responsibility
...
If you are caught lying, after your face goes back to its normal color, you will most likely be
looking for another job
...
If you claim credit for something that is not
yours and you hurt somebody, then you have crossed over the line
...
Or even worse if someone causes pain to others because
he or she believes in a lie you’ve told, the consequences can be very severe
...
For a long time no one noticed the correlation between him and the death rate, but
by the time it was discovered, he had left a trail of dozens of innocent victims
...


The first reader commented: “This essay combines a serious message with a bit of sophisticated
humor
...
The examples she uses to support
the thesis are well-written and sufficiently detailed
...

Score: 6
The second reader commented: “A thoughtful and insightful piece of writing, this essay deserves the
highest rating
...
’ Just how easy is wittily illustrated in the story of the
fabricated résumé
...
After presenting an example of compelling
evidence—a homicidal nurse—in the last paragraph the writer draws the only possible conclusion:
Lying ‘can lead to very harmful results
...

Score: 6

Vinnie’s Essay
I recently note a bumper sticker that stated, “Want to get ahead? Try Lying
...
I pondered the idea for awhile and realized how ridiculous the statement is
...
Once,
one morning I was very young I told my parents that I was too sick to go to school when the truth was
that I was afraid of a bully in the grade ahead of me who threatened to beat me up
...
My father wanted me to go to the doctor to get checked, I told him that I better
not go, or the doctor would catch my illness
...

Another time, I was in 8th grade I told my parents that I was going over to my friend Sam’s house
on a Friday night when I actually wanted to go bowling with some friends that my parents didn’t
approve of
...
The next day my parents found out that I
had lied, I think they met Sam’s parents at the movies where they had a puzzled look about my being
over at their house
...

There are many other times when my attempts at lying backfired in my face
...


The first reader commented: “This essay is lively and quite readable
...
The piece is unified in theme and tone, and its reference to a bumper sticker at the end
cleverly reminds the reader where the essay began
...

Score: 4
The second reader commented: “The writer of this adequate essay takes the position that it is
‘ridiculous’ to depend on lies to get ahead in life
...

“The essay is organized into coherent paragraphs
...
A few sentence errors such as comma splices and run-ons detract from the overall
quality of the essay
...

Score: 3

Marylou’s Essay
“Want to get ahead? Try lying” is described as a true statement in our society today
...

For example, in our society insider trading tips in the stock market can make an investor pay off
faster than another investor who is playing by the rules
...
He usually tries to exaggerate, in other words, lying,
on their résumé so that he will be a more enticing employee
...

Lying may get a person ahead, but they must also live with a conscious
...
He
did not “get ahead” based on skills or knowledge but on the lie
...
Dependency on lying can be cured by therapy
...
In
general, the writing is inept—crowded with usage errors
...
’ Although the essay
shows evidence of an ample vocabulary, its expression is awkward, and the last paragraph seems
totally irrelevant
...
Starting
with a puzzling and pointless assertion that the quotation ‘is described as a true statement in our
society’ (by whom? one wonders), the writer generalizes that ‘good things will happen to people’ who
lie
...
Then, the third paragraph undercuts the essay’s main idea with a brief discussion
of liar’s guilt
...

Score: 2

Prompt for Essay #2
Think carefully about the issue presented in the following statement:
Fatalists believe that we must accept things as they are, that nothing can be done to change the
world for the better
...

Maya Angelou added, “If you don’t like something, change it
...
Stay up and fight
...
Please plan

and write an essay in which you identify a school, local, national, or world condition or flaw that, in
your opinion, needs to be corrected, and explain why you chose it
...
I am not just a
tree hugger by saying that without improvement in environmental conditions, my generation and the
future generations will suffer in ways too awful to even imagine
...

Many countries exploit the Earth’s resources
...
Not only does this
destroy the habitats of many animals, but kills species of trees and plants that may someday be found
to cure cancer, AIDS, MS, or other diseases
...
The bottom line is that by clear-cutting rainforests, we are beginning to
suffocate ourselves to death
...
By dumping garbage, sewage and other hazardous waste into the
oceans, they pollute the water
...
The pollutants also kill and taint the fish in the ocean with toxic
materials
...
Many people I know have
given up eating tuna fish for the reason they don’t want to put poisonous mercury into their bodies
...
Until the famous Kyoto
Treaty, most countries had no laws controlling the amount of harmful gases released by cars and
trucks that causes global warming
...
I think that governments
around the world, including the US of A needs to force the corporations to reduce their burning of
fossil fuels in order to clean the atmosphere and stop the trend to global warming
...
When all is said and done, all
people must do their part to leave the world a better place for their children and grandchildren
...
He develops examples of environmental issues fully and appropriately
...
It reveals a high
level of proficiency and solid control of the essay-writing process
...
The writer is well-informed on the subject, but he rarely
transcends the environmental movement’s customary talking points
...
Nevertheless, the essay is mostly error-free, and its organization
and varied sentence structure demonstrate reasonably consistent mastery of the art of writing
...
If I could help solve one of
them, I would focus on my high school
...

One thing wrong with this school is that there is not enough leeway for us to take all the classes that
we want
...
Gym takes up so much of our school
week
...

Another criticism I have about this school is the way certain classes operate
...
They
spend too much time on inane things like handing in homework on time, getting to class on time, a
neat notebook, and a cover on your textbooks instead of concentrating on the more important issues
such as making sure that students get the information and the experience they need for a good
education and for college
...

Students go to high school one time in there lives
...


The first reader commented: “The first paragraph clearly states the essay’s purpose
...
But these problems are only
loosely tied to the main idea that the school fails to meet everyone’s needs
...
The third paragraph of the essay refers to the
students’ attitude toward their teachers
...
Tracy’s
use of language is rarely interesting and almost consistently awkward
...
In all, the essay shows modest evidence of Tracy’s promise as a writer
...

Instead, the writer uses the essay to air gripes about her own experience
...
In effect, then, the bulk of the
essay fails to live up to its promise
...
Even though you can’t taste, touch or smell
divorce, you can still feel it
...
“The
hottest places in Hell” that Dr
...

Why do people get married and repeat vows such as “until death us do part,” if they end up ending
their relationship? Where’s the logic in this? not to mention their children suffering
...
This may sound extreme for all divorces don’t end up this way
...

Institutions such as churches and synogogues shouldn’t allow people to be “joined in matrimony”
If the people don’t seem right for each other
...
The fusion of two people may seem to be one of
success, however when the “lawfully wedded” couple splits, the fission is one of disaster
...
The
second paragraph, for instance, suffers from disjointedness, in particular it’s allusion to a ‘threelegged race
...

“The essay is soundly organized, and although the meaning of some ideas is fuzzy, a bit of the
language—the fission/fusion dichotomy, for example—reveals a creative flair
...

Score: 3
The second reader commented: “This piece demonstrates marginal mastery of the essay-writing
process
...
Instead, she
uses the essay to lament the consequences of divorce, quite possibly stemming from her own
experience as a child of divorce, although she doesn’t say so
...
But her progression of
ideas lacks consistency
...

Score: 3

Prompt for Essay #3

Think carefully about the issue presented in the following passage:
The moon belongs to everyone
The best things in life are free,
The stars belong to everyone
They gleam there for you and me
...

“The Best Things in Life Are Free,” song and lyrics by B
...
DeSylva,
Lew Brown, and Ray Henderson for the musical Good News
...
Please plan and write an essay in which you discuss the validity of the sentiment
expressed by the lyrics of the song, “The Best Things in Life Are Free
...


Tucker’s Essay
The idea that “the best things in life are free” is nothing except sentimental garbage, okay for a
musical show but just a fantasy or a self-delusion in reality
...

Take, for example, an ordinary walk in the park with a favorite girl, guy, or dog
...
No out-of-pocket expenses there, but think of all that it costs to have
that walk
...
There is the need for transportation to the park
and home again, and the need to be able to contact a friend by phone or email to arrange the walk
...

Okay, walking in the park may be a trivial example
...
S
...

It would be nice to believe that the best things in life are free, but only the blissfully ignorant could
really believe it
...


The first reader commented: “Tucker’s opening statement hooks the reader instantly and clearly
articulates the essay’s insight that everything has ‘hidden costs that can’t really be calculated
...
The paragraph about
freedom, however, fails to discuss the costs other than those incurred by helping other countries
achieve a measure of freedom
...
Tucker
also provides evidence of his mastery of writing with varied sentences, some colorful word choices,
and a distinctive, natural style
...

Score: 6
The second reader commented: “This is an exemplary essay, not totally free of flaws, but close
enough to rate as a first-rate piece of writing
...
The tone is slightly
glib but nevertheless appealing and effective
...

Score: 6

Emily’s Essay
I totally disagree with the statement “The best things in life are free” because hobbies of mine
which I feel are “the best”, are far from free
...
First you must worry about transportation, air
fare
...
Food and entertainment isn’t
free and if you literally have expensive taste, you must spend money, and lots of it
...
If you need new bras and
underwear, forget about clothes, you are talking a lot of money
...
If you need
presents for special friends, jewelry will cost you especially if you want to buy sterling silver or 14K

gold or precious and semiprecious stones
...
The
whole experience is phenominal
...
You are suspended in water and you can breathe
...

Materialistically the best things in life are not free
...


The first reader commented: “If one overlooks this essay’s abundant sentence errors and errors in
mechanics, the piece has considerable clout, created in large measure by the spirited voice of the
writer
...

“The essay reveals that Emily is a relatively undisciplined writer
...
To give the essay greater coherence, however, some transitional material might have
linked the paragraphs more firmly
...

Score: 4
The second reader commented: “Demonstrating evidence of skill in critical thinking, the writer has
chosen three ‘hobbies’ to argue against the views expressed in the song lyrics
...
As she observes in her
final paragraph, ‘Materialistically the best things in life are not free
...
Taking vacations is a more general activity than the other two, and
although vacations are expensive, they merely provide Emily with opportunities to shop and scubadive
...

Score: 4

Josephine’s Essay
I agree with this sentiment that truly the best things in life are free
...
To obtain these things in ones life
depends on the individual
...
Happiness you make, a smile, a laugh even a tear can make
anyone happy
...
It also depends what you want to buy in life these things that I stated before and
things that won’t last forever!

The first reader commented: “After clearly stating the essay’s purpose—to show why ‘the best things
in life are free,’ the writer presents a series of related but largely incoherent reasons
...
This is a monumental undertaking even for a seasoned writer
...

Score: 1
The second reader commented: “This abbreviated essay demonstrates very little mastery of the art of
writing
...
Indeed, one idea (‘helping someone at your own
expense can cost absolutely nothing’) seems to contradict not only itself but the essay’s alleged
purpose
...

Score: 1

Essay Topics for Practice
In case you’ve just gotten to this point without having read Parts III and IV, follow these directions for
writing practice essays:
Time: 25 minutes
Plan and write an essay in response to the assigned topic
...
Present your thoughts logically and
precisely
...
A plain, natural
writing style is probably best
...

Limit your essay to two sides of lined paper
...
Write or print legibly because handwriting that’s hard or impossible to
read will decrease your score
...

The ten prompts that follow are suggested for essay-writing practice
...
Then compare the results
...
Always support your answer with reasoning and examples
taken from your observations, experience, studies, or reading
...
The familiar admonition to “put your money where your mouth is” suggests that it’s far
easier to speak up for a principle than to live up to it
...
It’s just part of human nature
...
“If you are like most people, your sadness over losing, say, $1,000, would be twice as great
as your happiness at winning $1,000
...
’”
From an editorial in The New York Times, January 16, 2005

Assignment: Are negative emotions stronger than positive ones?
3
...

Assignment: Is Jim Smith correct—that courage is a widespread human trait that most
people never have a chance to use or demonstrate?
4
...
Feeling that a direct
approach would lead to a confrontation, my friend said: ‘I am a biologist with So-and-So
University
...


The guy had his kids stop tormenting the turtle right away
...
An old English proverb says, “What you don’t know can’t hurt you
...
“Destiny is not a matter of chance
...
It is not a thing to be waited for, it
is a thing to be achieved
...
“Suppose we were able to share meanings freely without a compulsive urge to impose our
view or conform to those of others and without distortion and self-deception
...
“Every problem has a gift for you in its hands
...
“They may forget what you said, but they’ll never forget how you made them feel
...
Beuchner

Assignment: Are human emotions more powerful, enduring, and meaningful than our
intellect? Or to put it another way, does the heart matter more than the brain in our lives?
10
...
I think that the purpose of life is to be
useful, to be responsible, to be compassionate
...

Leo Rosten

Assignment: Do happiness and contentment depend mainly on serving others and making a
difference in their lives?
Before you evaluate your essay, reread how SAT essays are scored
...
Because SAT readers will give you credit for what
you’ve done well, try to focus on your essay’s strengths
...
If you happen
to find any, turn to the appropriate section of this book for a quick fix on how to remedy the problem
...

A solid grounding in basic English grammar will help you succeed
...
For one thing, you can
study!
To begin, read this chapter
...
Like any complex system
of rules, grammar takes time to learn
...
But don’t get bogged down trying to
memorize every detail
...

You could also borrow a grammar book from your English teacher or from the library
...
webgrammar
...
refdesk
...
html) and spend many profitable hours
browsing and reading
...
Learn the suggested strategies for answering the questions, do the exercises, and
take the practice tests in Part VI
...


Improving Sentences Questions
More than half of the multiple-choice sections of the SAT Writing Test are Improving Sentences
questions that ask you to recognize two types of errors:
1
...
Errors in style and expression
All the sentence-improvement questions on the SAT begin with a sentence in which a part, or
sometimes the entire sentence, is underlined
...
Your job is to choose the version that makes use of clear and concise standard
English and is free of errors in grammar and usage
...
If you think the original version is
better than any of the alternatives, mark choice A on your answer sheet
...


TIP
The Difference Between Usage and Grammar
Although the words are often used interchangeably, usage describes actual spoken and
written language
...

Grammar, on the other hand, is a set of rules that are followed when you speak and
write “correctly
...

(A) she speaks
(B) and speaking
(C) and she spoke
(D) as she spoke
(E) she spoke
The underlined segment of the sentence contains the subject of the sentence, she, and the verb
speaks
...

A basic rule of English grammar is that the tense of verbs in a sentence must remain logically
consistent
...

Choice A uses speaks, a present tense verb
...
The shift from past to present tense makes the sentence incorrect
...

In choice B, the phrase and speaking makes little sense because it has no grammatical connection to
the words Because Lucy was furious
...
The words Because Lucy was furious suggests that the rest
of the sentence will explain what happened as a result of Lucy’s anger
...

Choice D is wrong because it has the same problem as choice C
...

Choice E is the best answer because the verbs was and spoke are both in the past tense, and she
spoke loudly accurately describes what happened when Lucy lost her cool
...

(A) Great enjoyment was experienced by me at the wedding of my sister
(B) The experience of my sister ’s wedding was greatly enjoyed
(C) Being at my sister ’s wedding was an experience of great enjoyment
(D) I greatly enjoyed my sister ’s wedding
(E) A greatly enjoyable experience for me was the wedding of my sister
Here the whole sentence is underlined
...
Effective writing should be clear and brief its ideas
gracefully expressed
...


Choice A is a passive sentence, one in which the subject of the sentence is the receiver of the action
...

Choice B leaves the reader uncertain about who had enjoyed the wedding
...

Choice E contains many more words than are necessary
...

(For help in making informed decisions about effective style and expression, read “Problems in
Style and Expression”
...

(A) was assassinated, its original name was given back to it ten years later
(B) was assassinated and it got back its original name ten years later
(C) was assassinated; its original name was restored ten years later
(D) was assassinated, it was restored to its original name ten years later
(E) was assassinated; however, with the restoration of its original name ten years later
The underlined text of the original sentence has three problems
...
A comma
improperly separates two individual sentences
...
A third option is to keep the
comma and add an appropriate conjunction (and, but, or, nor, for, yet, or so)
...
The underlined text, which contains thirteen words, is less
concise than any of the other choices
...
The phrase was given back to it has a decidedly ungraceful
sound
...
Reject it
...
In addition, it got back its original name is awkward, due in part to the use of it
and its in the same phrase
...
It is the best answer
...
Also, like choice B, it awkwardly repeats the pronoun it
...
That is, the construction beginning with however is an
incomplete sentence
...
On the
SAT, under the pressure of time, you are expected to do a similar but far quicker analysis
...
Sometimes two or
more choices may be grammatically correct, but the best answer will be the most graceful and
effectively expressed sentence
...
Some assess
your knowledge of standard English usage
...

In short, sentence-correction questions deal with dozens of writing problems
...
To answer the questions you need to apply such
basic principles of good writing as:
Omit needless words
...

Choose precise words
...

Avoid awkward and clumsy expression
...

Read the material and answer the sample questions
...


Wordiness
Sentences cluttered with unnecessary words are less effective than tightly written sentences in which
every word matters
...
Look also for clauses that can be shortened
to phrases and phrases that can be recast as single words
...
During the months of July and August last summer, I had a wonderful summer vacation
...

2
...

The sentence contains too many words
...


Sample Questions Containing Wordiness
1
...

(A) Among the many numerous threats in the contemporary world in which we live are both
the threat of global warming and the threat of terrorism
(B) Among the many threats we face in the contemporary world in which we live are global
warming and the threat of terrorism
(C) Both global warming and terrorism are two of the many threats faced by today’s world
(D) Today’s world faces, among many other threats, global warming and terrorism
(E) We live in a contemporary world facing, among many other threats, the threats of global
warming and terrorism
Choice A, in addition to repeating threat, contains two redundancies
...
(After all, where else do we live
except in the contemporary world?)
Choice B contains the redundant phrases contemporary world and in which we live
...


Choice C contains the redundant words both and two
...
It is the best answer
...

2
...

(A) Because of her gender was the reason why Emma felt she was deprived of a place playing
on the varsity football team
(B) Emma felt that her gender kept her from playing on the varsity football team
(C) Because of her gender, Emma gave it as a reason why she was deprived of a place to play
on the varsity football team
(D) Emma, a girl, feeling the reason why she was deprived of a place on the varsity football
team
(E) As a girl, Emma, felt that she could not play on the varsity football team
Choice A contains the redundancy the reason why
...
In
addition, the phrase place playing is awkwardly expressed
...
It is the best answer
...
Delete to play
...
In addition, the construction is a sentence
fragment
...


Awkwardness
Awkward and clumsy are vague words that cover a great many writing weaknesses, including poor
grammar and flawed sentence structure
...
Awkwardness is difficult to define, but you know it when you
hear it
...


Sample Questions Containing Awkward Construction
1
...

(A) did, this is being why she found the skull and not he
(B) did, therefore Ellie and not him found the skull
(C) did; therefore she found the skull and not he
(D) did, which being the reason why she found the skull and not him
(E) did, being the reason why she found the skull and not him
Choice A contains this is being, an awkward, nonstandard usage
...


Choice C is standard usage and is properly punctuated
...

Choice D contains which being, an awkward, nonstandard usage
...

Choice E contains the redundancy the reason why
...
Also
him should be he
...
Vertical take-off and landing aircraft get their fixed-wing capability from high-speed air
pumped from slots in the trailing edges of their rotors, in which it increases the airflow over
them to create lift
...

Choice B eliminates the awkwardness and is concise
...

Choice C is not awkward but it contains a comma splice
...

Choice E is awkward and ungrammatical
...
For example:
The poem contains illusions to Greek mythology
...

The boys ran a fowl of the law when they shoplifted the DVD
...


Sample Questions Containing Faulty Word Choice
1
...

(A) Marissa holded her father in disrespect by
(B) Marissa showed disrespect for her father by
(C) Marissa disrespects her father by
(D) Disrespecting Marissa’s father by
(E) Having shown disrespect for her father by
Choice A uses the incorrect past tense of the verb to hold
...


Choice B uses the correct words in the correct order
...

Choice C uses the incorrect tense of the verb
...
Use disrespected instead of disrespects
...

2
...

(A) to fire fighters is their reliability on
(B) of fire fighters is to depend on
(C) to fire fighters is they must rely on
(D) to fire fighters is their reliance on
(E) to fire fighters is his reliability for
Choice A uses reliability instead of reliance, an example of faulty diction
...

Choice C contains mismatched sentence parts; in standard usage a noun (quality) may not be
defined by a clause (they must rely …) but only with another noun
...
It is the best answer
...
The phrase his reliability for is not standard English
...
When you “have a
ball,” the experience has nothing to do with a spherical object used on the basketball court or soccer
field
...
Such idioms often puzzle
speakers of other languages, but to native speakers of English, they are as natural as breathing
...
The italicized words in the
following sentence are examples of faulty idiom:
The general was unwilling to pay the price for victory
...

As regards to her future, Tina said she’d go to college
...
Revised, the sentences would read:
The general was unwilling to pay the price of victory
...

With regard to her future, Tina said she’d go to college
...
Stopping at a dime is what the engineers were after when they designed brakes for the highspeed train
...
The expression is on a dime, not at a dime
...

Choice D uses the correct idiom
...

Choice E uses the correct idiom but changes the meaning by saying that the engineers, not the train,
wanted to stop on a dime
...
Einstein’s theory of relativity is, for most of us, one that is with difficult understanding
...

Choice C is idiomatic but it fails to relate logically to the previous part of the sentence
...
It is the best answer
...
It also conveys a more or less complete thought and is grammatically whole,
which means that it has a subject and a verb
...

The bicycle that Martha often borrowed
...
It starts with a
capital letter and ends with a period
...
What makes it a
fragment, though, is that the subject bicycle and verb borrowed don’t fit together
...
It was Martha
who did the borrowing, but the noun Martha cannot be the subject of the sentence because it is part of
the subordinate clause, that Martha borrowed
...

The bicycle that Martha often borrowed was stolen
...

Sentence fragments usually occur when writers fail to distinguish between dependent and
independent clauses, when they confuse phrases and clauses, or when they attempt to use verbals as

verbs
...
That is, deconstruct the
sentence by eliminating dependent clauses, phrases, and verbals
...

To identify the subject of long sentences may take some doing, but the “bare bones” strategy
usually works
...
It’s not very formidable, though, if you remember that the
grammatical subject can never be in (1) a prepositional phrase, (2) a dependent clause, or (3) a phrase
that interrupts the flow of the sentence
...
Sometimes the bare bones are buried deep within long and
complicated sentences
...

To find the “bare bones” of a sentence:
Step 1: Look for prepositional phrases, such as up the wall, around the corner, to the
beach, over the counter, and cross them out
...

Complete sentence:
Bare bones:
Complete sentence:
Bare bones:
Complete sentence:
Bare bones:

In the middle of the night, Pricilla slept
...

Several are
One of Frida’s friends is in need of help
...
Another group of
dependent clauses are statements (not questions) that start with when, where, which,
who, and what
...
That’s where to find the bare bones of each sentence
...

Marnie wept
While Willie waited for the bus, he studied
vocabulary
...

Andy helps out

Step 3: Look for and delete interrupters—those parts of sentences that impede the
smooth flow of the main idea
...
They’re often set off by commas
...

Ellen rejoiced
The boat, a sleek white catamaran, sank
...

Marty paid

Sample Questions Containing Sentence Fragments
1
...

(A) stars that came
(B) stars coming
(C) stars, which are coming
(D) stars came
(E) stars, which came
Start your analysis of this sentence by deleting all the prepositional phrases, namely During the
night, like diamonds and on black velvet
...
The only words left are the stars, clearly not a complete sentence
...

Choice B is wrong because the –ing form of a verb may not be the main verb of a sentence unless it
is accompanied by a helping verb, as in is singing, has been raining, will be arriving
...

By the process of elimination, then, choice D is the best answer
...

2
...

(A) A belief among superstitious people that
(B) Superstitious people believe that
(C) Superstitious people believing that
(D) Among superstitious people the belief that
(E) Among beliefs of superstitious people are that
Analyze the sentence with the same technique used in question 1—by deleting all prepositional
phrases and dependent clauses
...


Choice A has a grammatical subject, belief, but the construction is a fragment because it lacks a
main verb
...

Choice B contains both a subject, people, and a verb, believe
...

Choice C contains a subject, people, but no verb
...

Choice D has neither a subject nor a verb because the construction is made up only of phrases and a
dependent clause
...


Run-on Sentences
A run-on sentence consists of two independent clauses separated by neither a conjunction (and, but,
or, nor, yet, or so) nor an appropriate mark of punctuation, as in:
Birthstones are supposed to bring good luck mine has never brought me any
...

Birthstones are supposed to bring good luck, but mine has never brought me any
...
A comma has also been added because sentences made up of two or
more independent clauses joined by a conjunction usually require a comma
...
Mine has never brought me any
...
In effect, the semicolon
functions like a period
...


Sample Questions Containing a Run-on Sentence
The campers hated the taste of powdered milk they drank water instead
...
A period or a semicolon is needed between milk and they
...

Choice C needs a comma and is awkwardly worded
...
It is the best answer
...


Semicolon Errors
Misuse of a semicolon is a common error in sentence-improvement questions
...
Correctly used, a semicolon must lie between
two independent clauses
...

The clause which raised her final average is not an independent clause
...


Sample Questions Containing a Semicolon Error
Mending a fracture takes from four weeks to a year; depending on the size of the bone, the
location, and the age of the person
...

Choice C needs a comma to be correct
...

Choice E properly uses a comma to separate the two parts
...


Comma Splices
A form of run-on sentence is the comma splice, a construction in which a comma is used between two
independent clauses instead of a period or a semicolon
...

(A) authors, she is known
(B) authors; she is known
(C) authors famous
(D) authors since known
(E) authors being that she is known

Choice A is a comma splice
...

Choice B is correct because it uses a semicolon to separate two independent clauses
...

Choice C needs a comma to be correct
...


Practice in Writing Correct Sentences
Directions: Some of the following are sentence fragments, others are run-ons, and still others
contain comma splices
...


1
...




2
...




3
...




4
...




5
...





6
...




7
...




8
...




9
...




10
...




Mismatched Sentence Parts
Sentences work best when their components fit together harmoniously and grammatically
...
A breakdown in logic or clear thinking may also account for an error, as when two ideas
expressed in a compound sentence are unrelated
...


FAULTY COORDINATION
In everyday conversation people often use lengthy compound sentences made up of several short
sentences joined by and, so, or other conjunctions:
In school on Tuesday the lights went out, and we were in the dark for more than an hour, and the
electricity was off, so we couldn’t use the computers, and we heard that a car had hit a utility pole,
and the driver was killed, and they let us go home early
...
Yet, it is stylistically
flawed, not because it’s monotonous but because each idea appears in an independent clause,
suggesting that all the ideas are equally important
...
Faulty
coordination occurs (1) when it is illogical or inappropriate to assign equal importance to two or
more coordinate clauses, or (2) when the connecting word fails to create a reasonable relationship
between the clauses
...

The two coordinate clauses state seemingly unrelated ideas, obviously of unequal importance
...
In other words,
making clauses dependent reduces the significance of the information they contain, thereby changing
the effect of the sentence:
While Tom was away at summer camp, his parents decided to split up after twenty years of
marriage
...

Ms
...

Making the second clause dependent by using although creates a more sensible connection between
the ideas:
Ms
...

For the sake of unity and coherence, it is usually better not to shift from one grammatical subject to
another between clauses
...

Faulty: The plan will be a great success, or great failure will be the result
...

Unified: The plan will be a great success, or it will be a great failure
...


Sample Questions Containing Faulty Coordination
1
...

(A) and she has not yet sent in her application
(B) and she hasn’t sent her application in yet
(C) but her application hasn’t as yet been sent in by her
(D) yet the sending of the application has not yet been done
(E) but she hasn’t yet sent in her application
Choice A is incorrect because the conjunction and fails to express a reasonable relationship
between the two coordinate clauses
...

Choice C expresses an apt relationship by using the conjunction but, but then it switches subjects
and changes from active to passive construction
...

Choice E conveys the relationship between the clauses and is consistent
...

2
...

(A) My weekend job at The GAP will help me as a marketing major, and I am learning about
retail selling
(B) Learning about retail selling, my weekend job at The GAP will help me as a marketing
major
(C) My weekend job at The GAP, where I am learning about retail selling, will help me as a
marketing major
(D) Helping me as a marketing major is learning about retail selling in my weekend job at The
GAP
(E) My weekend job at The GAP will help me as a marketing major; I am learning about retail
selling
Choice A is a sentence that gives equal weight to its two clauses even though the content of the first
clause is probably more important than the content of the second
...

Choice C properly subordinates the second clause and embeds it in the independent clause
...

Choice D turns two clauses into one, but the subject helping and the predicate nominative learning
make an awkwardly worded combination
...


FAULTY SUBORDINATION
By means of subordination, writers are able to convey not only the interrelationship of ideas but also
the relative importance of one idea to another
...
He ate a tuna sandwich
...

While he rushed to school, Joe ate a tuna sandwich
...

In each sentence, the more important idea appears in the main clause instead of in the subordinate
clause
...
Others include after, although, as if, as though, because, before, if, in
order to, since, so that, that, though, unless, until, when, whenever, where, whereas, wherever, and
whether
...
The sentences that follow illustrate typical problems:
While she is a mature young woman, she is afraid of the dark
...
Here,
however, while obscures both the meaning of the sentence and the relationship between the two
statements
...

A new subordinating conjunction clarifies the meaning
...

The meaning may be clear, but in this context where is not standard usage
...

Another problem concerns the placement of emphasis
...

I arrived home from school and I received my acceptance letter from Ohio State
...


Sample Questions Containing Faulty Subordination
1
...

(A) Pedro is a new student in the school, and he comes from Portugal
(B) Pedro, being from Portugal, is a new student in the school
(C) Pedro, a new student in the school, comes from Portugal
(D) Pedro, a new student in the school and a native of Portugal
(E) Pedro is a new student from Portugal in the school
Choice A is grammatically correct, but it would be more effective if one clause were subordinated
to the other
...

Choice C properly subordinates one idea and embeds it in the main clause
...

Choice D is a sentence fragment
...

2
...

(A) When he suddenly started to grin like an imbecile, I was walking with him in the park
(B) While I walked with him in the park, he suddenly started to grin like an imbecile
(C) Suddenly starting to grin like an imbecile, he was walking in the park with me
(D) He grinned suddenly like an imbecile and walked in the park with me
(E) Walking in the park with me and suddenly grinning like an imbecile
Choice A is incorrect because it places the more important idea in the subordinate clause
...
It is the best answer
...

Choice D is wrong because it changes the meaning of the original sentence
...


FAULTY PARALLELISM
Faulty parallelism occurs most often when an item in a series is not grammatically parallel to the
others, when a sentence is constructed of mixed, or unrelated parts, and when the subject or tense of a
verb changes from one part of a sentence to another
...
That is to say, their structure should be
repeated using the same parts of speech in the same order
...
Without parallelism, you get jumbles such as this:
Today a television newscaster must be attractive and a lot of charm
...
Revise the sentence by
making both words nouns or both words adjectives that modify nouns, as in
Today a television newscaster must have good looks and charm
...

Another example:
Eighteen-year-olds are too young to sign contracts, but they may have been driving for years
...
The second clause, however, takes an unexpected
and perplexing turn by changing the verb to the past perfect
...

Or written more concisely:

Eighteen-year-olds are permitted to drive but not to sign contracts
...
Students lacking financial resources can still go to college because they can borrow money
from banks, hold part-time jobs, and scholarships are available
...

Choice B contains constructions that are not parallel to the structure of borrow money from banks
...
It is the best answer
...

2
...

(A) its durability
(B) if it is durable
(C) the durability of it
(D) the ability of the item to last
(E) how durable it is
The sentence contains three elements that must be in parallel form
...
Only choice E follows this pattern;
therefore, choice E is the best answer
...
Mixed sentence parts suggest that the
writer, in finishing a sentence, ignored how it had begun:
Maggie’s goal is to be a nurse and is hoping to go to nursing school after graduation
...

Maggie aspires to be a nurse, and she is hoping to go to nursing school after graduation
...
But
subordinating one of the clauses is an even better solution to the problem:
Maggie, who aspires to be a nurse, hopes to go to nursing school after graduation
...

The verb was needs a subject
...

The problem has been solved by using coming as the grammatical subject
...
The next morning, after Christie’s car was found abandoned, there was a nationwide search for
the missing author had started
...

Choice B deletes one of the extra verbs in choice A but changes the grammatical subject to
beginning, a weak alternative
...

Choice D contains an error in verb form—begun instead of began
...

2
...

(A) crisis, but where it challenges conventional values
(B) crisis, whereas conventional values are challenged
(C) crisis in which conventional values are challenged
(D) crisis, and the reason is that their challenge of conventional values
(E) crisis because in it there are challenged conventional values
Choice A uses the conjunction but, which has no logical meaning in the context of the entire
sentence
...

Choice C completes the sentence grammatically and logically
...

Choice D is a sentence fragment
...


SHIFTS IN GRAMMATICAL SUBJECT
Still another type of faulty parallelism occurs when the grammatical subject of a sentence is changed

from one clause to another
...

The subject of the first clause is I
...

To fix a flat tire, I jack up the car and then remove the damaged tire
...


Sample Questions Containing a Shift in Grammatical Subject
The board recognizes the school’s troubles and now a giant fund-raising drive was being undertaken
by them
...

Choice B is wrong because sentence is cast in the present tense but shifts improperly to the past
perfect
...
It is the best answer
...

Choice E is a sentence fragment
...

The sentence begins in the past tense, then shifts to the present
...

The English language offers writers and speakers six basic tenses that convey information about the
time when an event or action took place:
Present:
Past:
Future:
Present Perfect:
Past Perfect:
Future Perfect:

I eat pasta every day
...

Phil will eat pasta every day
...

Enid had eaten pasta every day
...


All verbs also have a progressive form, created by adding –ing, so that you can say things like:
They are swimming
...
(Past Progressive)
The dog will be swimming
...
(Present Perfect)
Charles had been swimming
...
(Future Perfect)
Each of these tenses permits you to indicate time sequence very precisely
...

Perhaps the writer ’s intent is clear enough, but because precision is important, the sentence should
read:
When her little brother was born, Sarah had been toilet trained for six months
...
The original sentence actually says that her
brother ’s birth and Sarah’s toilet training took place at the same time—a physical impossibility, since
potty training usually takes weeks or even months
...

There was a condo where the park had been
...
The revision more accurately conveys the idea that the condo replaced
the park
...
Such items help to distinguish between students who use English
precisely and those who don’t
...
Jay had been working out in the weight room for months before the wrestling coach invites him
to try out for the team
...


Choice B uses the present perfect tense instead of the past perfect tense
...
The use of the past tense (invited ) indicates that
Jay’s workouts occurred not only prior to the coach’s invitation but that they were in progress at the
time the coach invited Jay to try out
...
Therefore, it fails to convey the precise sequence of
events, as expressed by choice C
...

2
...

(A) water, and financial woes make that impossible
(B) water, and that is becoming impossible due to financial woes
(C) water, but that it will have been made impossible by financial woes
(D) water, but financial woes made that impossible
(E) water, however, financial woes had made it impossible
Choices A and B contain coordinate clauses with an illogical sequence of verb tenses
...

Choice C contains the pronoun it that fails to refer to specific antecedent
...
It is the best answer
...


SHIFTS FROM ACTIVE TO PASSIVE CONSTRUCTION
Use active rather than passive construction except when: (1) the person or thing performing the action
is unknown or insignificant, or (2) the sentence is meant to emphasize that the subject has been acted
upon
...

Stated actively, the sentence emphasizes who performed the action:
The team scored three touchdowns
...

A shift from active to passive construction has occurred between the subordinate clause and the main
clause
...

Now both clauses are active; in addition, the grammatical subject has been maintained between the
clauses
...

Consider them faulty unless you see a clear necessity for constructing them in the passive voice
...
For the Thanksgiving weekend, Julie went to Richmond; however, for Christmas a trip to
Syracuse was made by her
...

Choice B is like A, but it also contains a comma splice between Richmond and however
...

Choice D maintains active construction but includes the redundancy but however
...
It is the best answer
...
Because the factory owners and their employees worked together to improve efficiency, a big
profit was earned
...
It is the best answer
...

Choice C is active but contains an improper shift in verb tense
...

Choice E is a sentence fragment
...
When they are far apart, sentences like this may result:
The fellow in the blue SUV with the long hair must be on his way to the concert
...

With the misplaced phrase in its proper place, the sentence reads:
The fellow with the long hair in the blue SUV must be on his way to the concert
...
Otherwise,
you may have a dangling modifier on your hands, as in:

Rushing to open the door, the rug slipped and sent Kyle sprawling
...
To fix
this so-called dangling modifier, the object being modified (in this case, the person rushing to the
door) must be included in the main clause
...

The grammatical subject, Kyle, is now properly modified by the participle, Rushing to open the door
...
For example:
Dangling: Climbing the ladder, Pete’s head knocked over the paint can
...
But look again, and you’ll notice that
it says Pete’s head climbed a ladder
...

Adding the noun Pete eliminates the dangling modifier
...
The plaque was presented to the actor that was engraved with gold letters
(A) The plaque was presented to the actor that was engraved with gold letters
(B) The plaque that was presented to the actor engraved with gold letters
(C) The plaque was presented to the actor who was engraved with gold letters
(D) The plaque, engraved with gold letters, and presented to the actor
(E) The plaque presented to the actor was engraved with gold letters
Choice A is wrong because the clause that was engraved with gold letters modifies actor instead of
plaque
...

Choice C is a variation of choice A
...

Choice E has its modifiers in the right place and is a complete sentence
...

2
...

(A) Driving to Litchfield
(B) While we drove to Litchfield

(C) En route to Litchfield
(D) To drive to Litchfield
(E) We drove to Litchfield and
Choice A contains a dangling modifier
...

Choice B contains we, the subject who performed the action
...

Choice C contains the same dangling modifier as choice A
...

Choice E is a sentence consisting of coordinate clauses that would be more effectively expressed if
one clause were subordinated to the other
...
A singular subject must have a singular
verb, and a plural subject must be accompanied by a plural verb
...

1
...

The prepositional phrase of today’s newspapers and magazines blurs the relationship between subject
and verb
...
With a singular
subject and verb properly matched, the sentence reads:
Delivery of today’s newspapers and magazines has been delayed
...

A writer can also err when words and phrases such as including, in addition to, along with, and as
well as come between the subject and verb
...

The bulk of English poetry, including the plays of Shakespeare, is written in iambic pentameter
...
When subjects are composed of more than one noun or pronoun
...
Nouns, both singular and plural, when joined by and, are called compound subjects,
which need plural verbs
...

Several locust trees and a green mailbox stand outside the house
...
Compound subjects thought of as a unit need singular verbs
...

The parents’ pride and joy over the birth of their baby is self-evident
...
Singular nouns joined by or or nor need singular verbs
...

Neither my history teacher nor my economics teacher plans to discuss the crisis
...
When a subject consists of a singular noun and a plural noun joined by or or nor, the
number of the verb is determined by the noun closer to the verb
...

Neither the linemen nor the quarterback was aware of the tricky play
...
When a subject contains a pronoun that differs in person from a noun or another
pronoun, the verb must agree with the closer subject word
...

Either he or I am planning to work late on Saturday
...
When the subject is singular and the predicate noun is plural, or vice-versa, the number
of the verb is determined by the subject
...

Two novels and a story are the extent of Wilkinson’s work
...
When singular subjects contain words that sound plural, use singular verbs
...

The news is good
...

4
...
Collective nouns sound singular but may be plural
...
But if you are referring to separate individuals, family takes a plural
verb
...

Other collective nouns include group, crowd, team, jury, audience, herd, public, dozen, class,
band, flock, majority, committee, heap, and lot
...


The jury is going to decide today
...

5
...
Some indefinite pronouns must be matched with
singular verbs, some with plural verbs, and some with one or the other, depending on the
sense of the sentence
...

a
...

Each man and woman in the room gets only one vote
...

b
...

In spite of rumors to the contrary, both are on the verge of a nervous breakdown
...

c
...

Some of the collection is valuable
...

Some of the bracelets are fake
...

6
...
When the subject of a sentence follows the verb, the
verb takes its number from the subject, as usual
...

Behind the building were an alley and a vacant lot (compound subject)
...
After thinking it over, the solution to most people’s problems with unwanted phone calls are
stricter laws and Caller ID
...

Choices B and C are wrong because they contain dangling modifiers
...

Choice D contains a singular subject and verb and is grammatically correct
...

Choice E has the same problem as choice A
...
In some of the big state universities the problem of giving scholarships and other rewards to
good athletes have gotten out of hand
...

Choice B is a variation of A
...
It is the best answer
...

Choice E is wrong because it uses a plural verb, are, that doesn’t agree in number with the singular
subject, problem
...
Locate the error and
write the corrected version in the space provided
...


1
...




2
...





3
...




4
...




5
...




6
...




7
...




8
...





9
...
McCallum were
told that their jobs were in danger
...
Many Democratic senators contend that reforms in the tax system has not brought about the
economic growth that had been predicted
...
Learning to read the daily box scores printed in the newspaper is a desirable thing to do by any
fan who expect to develop a deep understanding of baseball
...
Politics on both the national and local level have always been one of Dave’s passions
...
Charles Darwin, along with his contemporary, Abraham Lincoln, are among the most
impressive figures in nineteenth-century history
...
Katie Green, one of the hottest jazz pianists in town and known for something she calls “3-D
playing,” and her accompanist Lenny is planning to tour the South in May
...
Nancy, along with her friend Sluggo, appear to be coming down the escalator
...
The sale of computers in a market that has nearly a billion potential customers have created
enormous hope for the company’s future
...
Here’s the two statutes to which the defense lawyer referred during the trial
...
The commissioner ’s insistence on high ethical standards are transforming the city’s police
force
...
No one in the drum corps, in spite of how they all feel about the issue, want to participate in the
rally
...
According to school policy, there is to be two security guards stationed in the playground
during recess to protect the children
...
To express past action, add -ed to the
present form: walk/walked, cry/cried, type/typed
...
For present perfect, past perfect, and future perfect forms, add
have, has, had, or will have, as in have walked, has cried, had typed, and will have arrived
...

The verb to choose, for example, is choose in the present, chose in the past, and chosen in its
participle, or “perfect” form
...

Another error—usually a sentence fragment—occurs when a writer tries to use an -ing form of a
verb as a sentence’s main verb, as in:
Julie, at the box office, selling movie tickets to the 7:00 show
...

The addition of the helping verb has been corrects the error
...


Sample Questions Containing Faulty Verb Forms
1
...

(A) to have underwent the experience of seeing
(B) having underwent the experience of seeing
(C) to have undergone the experience of seeing
(D) to see during the experience of
(E) undergoing the experience of seeing
Choice A is wrong because it uses have underwent, a nonstandard form of the verb to undergo
...

Choice B is a variation of choice A
...
It is the best answer
...

Choice E improperly uses an –ing form of a verb without a helping verb
...
Brian Williams, the TV anchor man, skillfully probing his guest’s knowledge of the scandal,
but showing great tact because he didn’t want to jeopardize his chance for a news scoop
...
Neither has a main verb
...
It lacks a conjunction before the verb showed
...
The construction he was skilled lacks a grammatical relationship
to the earlier part of the sentence
...
It is the best answer
...


Use of Pronouns
A dozen common English pronouns—I, me, he, she, him, her, it, they, them, we, us, and you—cause
more trouble than almost any other words in the language
...

Faulty usage results most often:
• When pronouns in the wrong “case” are chosen
• When pronouns in the wrong “person” are chosen
• When pronouns fail to agree in number or gender with their antecedents
• When the pronoun reference is unclear or ambiguous

FAULTY PRONOUN CASE
Most of the time you can probably depend on your ear to tell you what’s right and wrong
...
” But when you can’t depend on the
sound of the words, it helps to know that those twelve pronouns fall into two groups
...

Remember that you mustn’t mix pronouns from different cases in the same phrase
...
Any time you need a pair of pronouns and
you know that one of them is correct, you can easily pick the other from the same group
...
If I seems to fit, you’re in
Group 1; if me fits better, use Group 2
...

If you insert me in place of one of the pronouns, you’ll get:
Elvis asked that me practice handstands
...
So the pronouns you need come
from Group 1, and the sentence should read:
Elvis asked that he and she practice handstands
...

1
...

The term predicate nominative refers to words not in the subject of the sentence that identify,
define, or mean the same as the subject
...
(he and I = subject) The instructors in the course were Donald and he
...
Use objective case pronouns in phrases that begin with prepositions, as in:
between you and me
to Sherry and her
among us women
at us
from her and him
with me and you
3
...

The waiter gave her and me a piece of cake
...
To find the correct pronoun in a comparison, complete the comparison using the verb that
would follow naturally:
Jackie runs faster than she (runs)
...

Carol is as tough as he (is)
...

5
...
(Deleting seniors leaves
We decided to … )
...
(Deleting students leaves award
was presented to us by the … )
...
Use possessive pronouns (my, our, your, his, her, their) before a gerund, a noun that looks like a
verb because of its –ing ending
...
(Asking is a gerund
...
(Opening is a gerund
...

Fishing is my grandpa’s favorite pastime
...

As a result of all that fishing, he hates to eat fish
...
Don’t confuse
gerunds with the participle form of verbs, as in:
Participle: Fishing from the bank of the river, my Grandpa caught a catfish
...


Not every noun with an –ing ending is a gerund
...
At other times, –ing words are verbs, in particular, they’re participles that modify pronouns in
the objective case
...
(Here intruding is a gerund
...
(Here intruding is a participle
...

(A) than the amount paid by Rosemary and I
(B) in comparison to the fee paid by Rosemary and I
(C) than that which Rosemary and me pay
(D) than the fee Rosemary and me paid
(E) than the one Rosemary and I paid
Choices A and B are incorrect because each contains a phrase beginning with the preposition by,
which calls for pronouns in the objective case
...

Choices C and D call for pronouns in the nominative case
...


Choice E uses the proper pronoun
...


Practice in Choosing the Case of Pronouns
Directions: Circle the correct pronoun in each of the following sentences
...
Judith took my sister and (I, me) to the magic show last night
...
We thought that Matilda and Jorge would be there, and sure enough, we saw (she, her) and
(he, him) sitting in the front row
...
During the intermission, Jorge came over and asked my sister and (I, me) to go out after the
show
...
Between you and (I, me) the magician was terrible
...
It must also have been a bad evening for (he, him) and his assistant, Roxanne
...
Trying to pull a rabbit out of a hat, Roxanne and (he, him) knocked over the table
...
When he asked for audience participation, my sister and (I, me) volunteered to go on stage
...
He said that in my pocket I would find $10 in change to split between (I, me) and my sister
...
When the coins fell out of his sleeve, the audience laughed even harder than (we, us)
...
If I were (he, him), I’d practice for a long time before the next performance
...

Pronouns must be in the same person as their antecedents—the words they refer to
...
Consistency is the
key
...

Consistent: When you (second person) walk your (second person) dog in that park, you
(second person) must carry a pooper-scooper
...

Consistent: If one tries to write a persuasive essay, one should at least include a convincing
argument
...

(A) The more you travel around the country
(B) The more we travel around the country
(C) The more one travels around the country
(D) As more traveling is done around the country
(E) As they travel more around the country
Choice A is incorrect because the second-person pronoun you shifts to the first-person pronoun our
in the second clause
...
It is the best answer
...

Choice D uses the pronoun our that fails to refer to a specific noun or other pronoun
...


PRONOUN–ANTECEDENT AGREEMENT
Singular pronouns must have singular antecedents; plural pronouns, plural antecedents
...
Note the problem of pronoun-antecedent agreement
in these sentences:
Everybody is sticking to their side of the story
...

Neither teacher plans to change their policy regarding late papers
...

Anybody can pass this course if she studies hard
...

Some people, objecting to the use of specific gender pronouns, prefer the cumbersome and tacky
phrase “he or she,” but most good writers avoid using it
...

The audience was asked to turn off their cell phones during the performance
...

The senior class had their portraits taken for the yearbook
...

(A) changed this policy beginning when their budget was cut
(B) begins to change this policy when their budget was cut
(C) began to change this policy when its budget was cut
(D) it changed this policy when their budget was cut
(E) beginning to change its policy, the budget was cut
Choice A is wrong because it uses the plural pronoun their to refer to the singular noun Army
...

Choice C uses the singular pronoun it to refer to the singular noun Army
...

Choice D contains a comma splice
...


Practice in Recognizing Pronoun Shift
and Pronoun Agreement
Directions: Some of the following sentences contain shifts in pronoun person or errors in
agreement between pronouns and antecedents
...
Alter only those sentences that contain errors
...
The English teacher announced that everyone in the class must turn in their term papers no later
than Friday
...
When you are fired from a job, a person collects unemployment
...
The library put their collection of rare books on display
...
Each of my sisters own their own car
...
In that class, our teacher held conferences with us once a week
...
In order to keep yourself in shape, one should work out every day
...
The teacher dictates a sentence in French, and each of the students write it down in English and
hand it in
...
Each horse in the procession followed their riders down to the creek
...
The school’s chess team has just won their first match
...
When one is visiting the park and you can’t find a restroom, they should ask a park ranger
...
Some references are
ambiguous because the pronoun seems to refer to one or more antecedents:
The teacher, Ms
...

Who is responsible? The teacher or Karen? It’s impossible to tell because the pronoun her may refer
to either of them
...
Taylor told Karen that it was her responsibility as the teacher to hand out composition paper
...

Whose car needed fixing? Who helped whom? To answer these questions, the sentence needs to be
rewritten:
Mike and Mark became good friends after Mark helped Mike repair his car
...
One way to set the meaning straight is
to use more than one sentence:
When Mark needed to repair his car, Mike helped him do the job
...

To be correct, a pronoun should refer directly and clearly to a specific noun or another pronoun, or it
should refer by implication to an idea
...

What has been disproved? That an allegation was made? That the mayor is indifferent? The intended
meaning is unclear because which has no distinct antecedent
...


Sample Question Containing an Ambiguous
Pronoun Reference
Ricky, Marti, and Steve were driving nonstop from New York to Chicago when, falling asleep at
the wheel, he drove the car off the road
...

Choice C avoids the pronoun-reference problem by using the Ricky instead of he
...

Choice D contains an error in verb tense
...

Choice E, a compound sentence, would be more effectively expressed with one independent clause
and two subordinate clauses
...
Please eliminate the
problem by revising each sentence
...

1
...




2
...




3
...




4
...




5
...
Rice’s tenure in office, she traveled more than any other secretary of state
...
Henry, an ambulance driver, disapproved of war but drove it to the front lines anyway
...
After the campus tour, Mike told Todd that he thought he’d be happy going to Auburn
...
Peggy’s car hit a truck, but it wasn’t even dented
...
Within the last month, Andy’s older brother Pete found a new job, broke his leg skiing, and got
married to Felicia, which made their parents very happy
...
Eddie grew fond of the novels of John Steinbeck because he had lived in California
...
In addition to knowing about comparative degrees, you need
to know that comparisons (1) need to be complete, (2) must be stated in parallel form, and (3) must
pertain to things that may logically be compared
...

The degree of comparison is indicated by the ending (usually –er and –est) or by the use of more or
most (or less and least)
...

Positive
tall
dark

Comparative
taller
darker

handsome

handsomer or more handsomest

graceful
prepared
happily

more graceful
less prepared
more happily

Superlative
tallest
darkest
handsomer or most
handsome
most graceful
least prepared
most happily

Some words deviate from the usual pattern
...
To form the comparative and superlative degree of one-syllable words, add –er or –est to the
positive form (brave, braver, bravest; late, later, latest)
...
To form the comparative and superlative degrees of most two-syllable words, use more or
most, or less and least (more famous, most nauseous, less skillful, least jagged)
...

3
...

4
...

My younger sister takes dancing lessons
...
)
My youngest sister takes swimming lessons
...
)
5
...
For example, avoid more
friendlier, less prouder, most sweetest, least safest
...
Instead, use adjectives and adverbs in the positive degree: more friendly, less proud,
more sweet, least safe
...

An incomplete comparison made colloquially may suffer no loss of meaning, but standard written
usage calls for unmistakable clarity
...

This could mean either that Mimi spent a longer time with her aunt than Kathy did, or that Mimi spent
more time with her aunt than she spent with Kathy
...

A comparison using as usually requires a repetition of the word: as good as gold, as fast as a
speeding bullet, as high as a kite
...

Complete: On the exam, Nicole expects to do as well as, if not better than, Nat
...

Lieutenant Henry was braver than any pilot in the squadron
...
If he belonged to the
squadron, however, add other to complete the comparison:

Lieutenant Henry was braver than any other pilot in the squadron
...

Diana talks more nonsense than anyone else in the class
...


ILLOGICAL COMPARISONS
Logic breaks down when two or more unlike things are compared
...

This sentence is meant to compare pollution in the Boston harbor with pollution in the harbors of
other cities
...
Properly
expressed, it would read this way:
Boston’s harbor is reported to be more polluted than the harbor of any other city in the country
...

Unlike most cars on the block, Ellie has her Toyota washed almost every week
...

The first sentence is intended to compare Ellie’s car with the other cars on the block
...


Sample Questions Containing Faulty Comparisons
1
...
Kisco and Pleasantville than the one we took
...

Choice B contains an error in parallelism
...
Use more direct
...
It is the best answer
...

Choice E is wordy
...

2
...


(A) as effortlessly as any pop star ever has
(B) as effortlessly as any other pop star ever has
(C) effortlessly, as any pop star has
(D) as effortlessly like any other pop star ever has
(E) as effortlessly, if not more so, than any pop star ever has
Choice A is incorrect because it omits other, a word that must be used when comparing one thing
with a group of which it is a member
...

Choice B expresses the comparison correctly
...

Choice C uses awkward language that obscures the meaning of the sentence
...
Use like, a preposition, to introduce a phrase; use as to introduce a
clause
...
Use as effortlessly as
...

Verbs
...

Nouns
...

Pronouns
...

Adjectives
...

Adverbs
...

Phrases
...

Clauses
...

Participles
...

Punctuation
...
Also sentence structure, including
run-ons, fragments, and comma splices
...
Your job is to identify which underlined portion of each sentence contains
the error
...


Sample Question

the two stories, the class

that the

was the



The sentence contains an error in comparison
...
But when you compare more than two
things, use words in the superlative degree, such as best, lightest, and most able
...
Therefore use better instead of best
...

Although Identifying Sentence Errors questions are shorter and less involved than those in the
sentence-improvement sections of the exam, they deal with an equally wide range of grammar and
usage problems
...
We say “three-foot ruler” when we mean “three-feet
...
” Both flammable and inflammable mean the same
thing—easily set afire
...
We accept these and many other linguistic quirks because
they are simply part of our language
...
For someone just learning English, though, “arrive to the movies” would make
perfect sense
...

On the SAT, you may find sentences containing faulty idiom
...
There are no specific guidelines
to help untangle problems in idiom
...

The First Amendment is invoked in those times when journalists are asked to disclose their sources
...
Replace in with at, a preposition that often refers to time—at
four o’clock, at the turn of the century
...

Here is another example:
A knight was faithful to his king, to his church, and to his lady, and he would gladly die in the name
of them
...

A knight was faithful to his king, to his church, and to his lady, and he would gladly die in their
name
...


Sample Questions Containing Faulty Idiom
1
...
Choice B is the
correct answer
...
Mr
...


Faulty idiom results from the misuse of a verb
...
Choice C is the
correct answer
...


a member of the scholarly panel, Dr
...

Faulty idiom results from the use of up, an unnecessary preposition
...

Choice C is the correct answer
...
Write revised versions
in the spaces provided
...


1
...




2
...




3
...




4
...




5
...




6
...




7
...





8
...




9
...




10
...




11
...




12
...




13
...




14
...





15
...




16
...




17
...




18
...




19
...




20
...




Faulty Diction

Faulty diction means faulty word choice
...

The English language offers abundant opportunities to choose incorrect words, but on the SAT the
range is limited to commonly misused words
...

Use which when referring to animals and nonliving things, as in:
Censors bleeped obscenities out of the film’s TV version, which disturbed free-speech advocates
...

It may look harmless, but poison oak is a plant that infects the skin with severe rashes and
itching
...
It’s fine to say “Those are the geese who are damaging
the grass,” but it’s also acceptable to say “Those are the geese that are damaging the grass
...


Sample Questions Containing Faulty Diction
1
...

The word whereas, meaning “although” or “considering that” is improperly used in the context
...

2
...


The writer has improperly used which instead of who to refer to schoolteachers
...

Choice A is the correct answer
...
A “fault” in tennis
the

the ball

to the opposing player lands outside the lines

of the service box
...
In standard English, the sentence might read something like “A ‘fault’ in tennis is a
stroke that falls outside …
...
The
reverse—using an adverb where an adjective belongs—also occurs, but less often
...


To begin, identify the errors in these two sentences:
Children addicted to television often behave violent in the classroom
...

If you spotted the errors, you should have no trouble with similar items on the SAT
...
But if you didn’t notice or couldn’t explain the errors, you should
definitely read on
...
Good is an example, as in good
apple, good book, and good night
...
Good should not be used after most verbs, so avoid
talks good, drives good, writes good, and so on
...
Therefore, it’s correct to
say sounds good, feels good, and is good
...
)
And to complicate matters still more, linking verbs are sometimes used as active verbs
...
But it is
an active verb when it refers to the act of looking, as in Margie looked sadly at her sick dog
...
If the sentence retains its basic meaning, the verb is probably a linking verb, as in The juice tastes
good/The juice is good
...

Because you wouldn’t say He is badly about your loss, feels is an active verb in that sentence
...
Most of the time they answer such questions as How? When? How
much? Where? In what sequence? To what extent? In what manner? For example:
How does the grass look? It looks mostly brown
...
)
When does Roger run? He usually runs in the morning
...
)
How much did it rain? It rained enough to flood the cellar
...
)
When you need to choose between an adjective and an adverb on the SAT, follow this procedure:
Find the verb and determine whether it is a linking verb
...
More often than not, the verb is likely to be one of those that is sometimes active and
sometimes linking
...
You can also
check its modification
...
If it modifies a
noun or pronoun, use the adjective
...


Sample Questions Regarding Adjective/Adverb Use
1
...


The word careful is an adjective
...

Therefore, it should be carefully, an adverb
...

2
...

The word constant is an adjective
...
Choice D is the correct answer
...
Write the correct words
in the spaces provided
...


1
...



2
...



3
...



4
...



5
...




6
...



7
...



8
...



9
...



10
...



11
...



12
...



13
...



14
...



15
...



16
...




17
...



18
...



19
...



20
...



Wordiness and Redundancies
A sentence needs revision when it includes words and phrases that don’t add meaning or that repeat or
reiterate what has already been stated
...

An important essential ingredient of a hamburger is meat
...

All three sentences contain a needless word or phrase
...
Therefore, necessary requirement is redundant
...
And in the last sentence, the phrase of the life should be removed because a biography
cannot be anything other than the story of someone’s life
...
The commission’s report
small

the contributions made

both big corporations

to the growth of the nation’s economy
...
Substitute and for as well as
...

2
...


The phrase as many as is unnecessary
...


new

Practice in Detecting Wordiness
Directions: Revise the following sentences for economy of expression
...
She constantly irritates and bothers me all the time
...
He spoke to me concerning the matter of my future
...
Is it a true fact that the ozone layer is being depleted?



4
...




5
...




6
...





7
...




8
...




9
...




10
...




Faulty Parallelism
Orderly construction of a sentence keeps parallel ideas in the same grammatical form
...

Every item listed is an object, each expressed in the same grammatical form: a noun preceded by one
or two adjectives
...

The message is clear, but the phrase “to take pictures” is not parallel with the other phrases
...


To recognize faulty parallelism in SAT sentences, you should know that:
1
...

The parallel ideas are expressed as prepositional phrases, to the noisy parties and to the trash
...

Each parallel idea consists of an infinitive followed by a noun and an adverb
...
In comparisons, parallel ideas should be in the same grammatical form
...

The gerund going may not be paired with the infinitive to cook
...

The ideas are now stated in parallel form
...
Parallel ideas are often signaled by pairs of words like either/or, neither/nor, whether/or,
both/and, and not only/but also
...

Revise by changing neither to either, or changing or to nor
...

The signal word both is too far removed from the parallel phrase, basketball and volleyball
teams
...
Correctly worded, the sentence reads:
Jake started on both the basketball and the volleyball teams
...
One of
teacher, and

greatest musicians, Leonard Bernstein


a composer,

, a

The phrase played the piano brilliantly should be a brilliant pianist in order to be parallel in form to
the other items in the series
...

2
...


The phrase than explaining should be to explain in order to be parallel in form to the infinitive to
swim
...


Incomplete Comparisons
Sentences used to make comparisons usually follow a familiar pattern that requires the items being
compared to be stated in parallel form
...


Sample Questions Containing Incomplete Comparisons
1
...


The sentence illogically compares quality with Wilson instead of with Wilson’s presidency
...

2
...


As written, the sentence compares Stewart with all comedians on television, but Stewart cannot be
funnier than himself
...

To make a proper comparison, use “any other comedian
...


Practice in Completing Comparisons

Directions: Find the errors in comparison in the following sentences
...
Some sentences may be correct
...
Jane is more efficient than any member of the committee
...
Andy looks more like his father than his brother
...
In The Great Gatsby, I disliked Daisy as much as Tom
...
Phil works faster than George does on most jobs
...
Oscar was as tired if not more tired than Pete
...
To do the research for my term paper, I read books more than searching the Web
...
Although she’s younger, Lillian looks as old if not older than Dorothy
...
They talked more about Chekhov’s stories than his plays
...
Allen’s canoe was destroyed in the rapids, just like his partner
...
After reading Siddhartha, I admire Hesse more than any author
...
I am more interested in rap music than Pete
...
Experts say that walking is better for you than to jog the same distance
...
Biology is more popular than any science
...
The students respect Ms
...
Green
...
His ears were bigger than Dumbo
...
It took us longer to reach Trenton than Camden
...
Which is cheaper—flying to Washington or to take the train?



18
...




19
...




20
...




ERRORS IN GRAMMAR AND USAGE

Noun–Verb Agreement
Nouns and verbs must agree in number
...

The Identifying Sentence Errors section of the SAT almost always includes questions that expect
you to recognize agreement errors
...
Just as often, though, the error exists between some other noun and verb in the
sentence
...
Intervening words obscure the relationship between the noun and verb
...
A singular noun sounds as though it is plural
...
The noun is one that can be either singular or plural, depending on its use
...
The noun comes after the verb in the sentence
...
Ian’s achievement

in addition to his

as a speaker,

him a popular

on

television cooking shows
...
The verb make is plural
...
Note that words and phrases coming between
a subject and a verb rarely affect the number of the verb
...
Behind the house

one broken-down shed and one pile of rubble

need

to the town dump
...
Because the subject is a
compound subject (two nouns joined by and ), it is considered plural and must be accompanied by a
plural verb
...
Choice A is the correct answer
...


a million and a half dollars
the shoddy construction methods

on repairing the road, but only recently
evident
...
Because has should be
have, choice C is the correct answer
...
Singular pronouns need
singular antecedents; plural pronouns, plural antecedents
...


Sample Questions Containing Errors in Pronoun–
Antecedent Agreement
1
...


The plural pronoun their refers to the singular antecedent woman
...
Because
the pronoun and antecedent don’t agree, choice C is the correct answer
...
The branches of the university,

a “no-layoff” policy, began changing

the pressure of financial losses
...
Because their should be its,
choice C is the correct answer
...
Use the space provided after each sentence to write your corrections
...


1
...




2
...




3
...




4
...




5
...




6
...




7
...




8
...





9
...




10
...




Faulty Pronoun Reference
Another common error occurs when pronouns fail to refer clearly to their antecedents
...


Sample Question Containing Pronoun Reference Errors
Sarah and her colleague, Kate,
got a bigger one this year

equal bonuses from

boss last Christmas but

her outstanding work
...
To fix the
problem, use Sarah or Kate in place of she
...


Practice in Identifying Faulty Pronoun Reference
Directions: The following sentences contain faulty pronoun references
...


1
...
Parker loves to knit and spends most of her time doing it
...
At the end, with all the questions on the test answered, I handed them in
...
Peggy told Eileen that she was sure that she had handed in the homework
...
Bill let his father know that he had only ten minutes left on the parking meter
...
During Truman’s presidency, he sent troops to fight in Korea
...
Henry, a helicopter pilot, regularly flies it on rescue missions
...
In Fitzgerald’s The Great Gatsby, he wrote about the American Dream
...
She decided to buy a high-definition television, which is just what he wanted
...
The agreement between Joan and Jane fell apart after she failed to show up for the meeting
...
After the interview, Mike told Tom that he would probably like spending the next four years at
Dartmouth
...
A sentence cast in first person, for
example, should remain so from start to finish
...


Sample Question Containing Shift in Pronoun Person
Although one

that

is going to be excellent preparation for life,

one actually experiences does not always

expectations
...
Because the pronoun your is in the second person, an
improper shift has occurred, making choice B the correct answer
...
Nominative case pronouns are reserved for grammatical
subjects and predicate nominatives
...
Problems
arise when writers fail to identify grammatical subjects or when they mix pronouns from different
cases in the same phrase
...
I learned

the speeches given by

could

be heard at

the back of the auditorium
...
Use me instead of I
...


2
...


Because the phrase Carl, Mack, and him is the subject of the sentence, the pronoun should be in the
nominative case
...
Choice B is the correct answer
...
Write the correct pronoun in the
space provided
...


1
...



2
...



3
...



4
...



5
...



6
...
If you are scheduled to deliver the speech to the class, one should expect to take your turn on
Monday
...
Him and I plan to drive to Danbury tonight
...
If you really want to get better at the piano, one really needs to practice
...
Them singing at the top of their lungs disturbed the quiet neighborhood
...
Tim is more interested in applying to Oregon State than her
...
The group asked us guys to pitch in on the drive for canned goods
...
Most runners say they have to run every day in order to keep yourself in shape
...
The last time I saw him, he was as tall as me, if not taller
...
I never spoke with them, neither she nor her sister
...
When a sentence contains a verb in the wrong tense or
when an improper shift in tense occurs from one part of the sentence to the other, the meaning of the
sentence suffers
...
The scene involves both Macbeth and Banquo
the three witches,

riding across the heath

they

the three hags or three weird sisters
...
Having been cast in the present
tense, the sentence should remain so throughout, but the verb encountered is in the past tense
...

2
...


The verb shall wear casts the sentence in the future tense
...
Therefore, C is the correct answer
...
Write the
revised verbs in the spaces provided
...


1
...



2
...
Murphy has brought her car in last night
...
For anyone with enough brains to have thought about the problem, now is the time to work out a
solution
...
When Washington was sworn in as president, he rode to New York from his home in Virginia
...
If the wagon train would have reached Salt Creek in time, the massacre would have been
prevented
...
The aircraft controller expects to have spotted the plane on radar before dusk last night
...
The family already finished dinner when the doorbell rang
...
First he built a fire, then dragged a log over to use as a seat, and finally collected enough wood
to keep the fire going all night
...
Rose kept the promise she has given to Charles last year in India
...
When he talks with Horatio, Hamlet began to suspect foul play in the kingdom
...
As they drove to Vermont, they had stopped for lunch at Bucky’s Bagel Shop
...
On Route 684, a trooper pulls him over and gave him a speeding ticket
...
Working all year to improve her writing, Debbie got a story published in the newspaper
...
That night at the show we met many people that we saw that afternoon
...
Once the drought had hit eastern Africa, the Somalis have suffered terribly
...
The vast majority of English verbs follow this
pattern
...
Examples include break (break, broke,
has broken), begin (begin, began, has begun), and rise (rise, rose, has risen)
...


Sample Question Containing Faulty Verb Form
1
...
The verbs

went, combed, and dressed are in the proper form
...
The past form of the verb to
bring is brought
...

2
...
Barnes
it

and

the paper may have been plagiarized, but Ray insisted that he
he could prove it
...
Only one, had wrote, is not in the proper form
...


Practice in Identifying Faulty Verb Forms
Directions: In these sentences, the underlined verbs may not be in the proper form
...
Some verbs may be correct
...
Brian use to arrive late to class almost every day
...
Conflicts between loggers and environmentalists have regularly arose in the Northwest and other
areas of the country
...
Given the choice of Monday, Wednesday, or Friday for her talk, Gwen demanded to speak on
Thursday
...
After dinner, Sarah cleared the table and blowed out the candles
...
They had began practice on their own before the coach arrived
...
When the engine overheated, the radiator hose had bursted
...
To get a front-row seat, you should have went to the play earlier than you did
...
The chorus messed up that song because they had never sang it before
...
Halfway to town I realized that the front tire of my bike had sprang a leak
...
The novels of Judy Blume have managed attracting millions of adolescent readers
...

If a verb is underscored, search for errors in tense, form, agreement with the subject or
other noun, and parallel structure
...

If a pronoun is underscored, search for errors in case, number, gender, agreement with
antecedent, reference to a noun or another pronoun, and agreement with verb and parallel
structure
...

If a phrase or clause is underscored, search for errors in parallel structure and sentence
structure
...


Improving Paragraphs Questions
This section of the SAT asks six questions about how to revise a draft of a short essay
...
Another may ask which revision
of a poorly written sentence is best
...

The wording of paragraph-improvement questions illustrates several of the matters you must deal
with:
1
...
In context, which of the following phrases most logically replaces “them” in sentence 9?
3
...
Which of the following is best to add after sentence 13 as a concluding sentence?
5
...
In context, which is the best way to revise and combine sentences 4 and 5?
7
...
The best way to describe the relationship of sentence 4 to sentence 5 is that…
...
All of the following strategies are used by the writer of the essay EXCEPT…
...
In the second paragraph (sentences 5–9), the author tries to…
...
You may also be questioned on the
structure and function of certain paragraphs as well as the role of individual sentences within
paragraphs
...
Try each of the following methods to find the one that produces the best results:
Method 1: Read the essay carefully from beginning to end
...
Because you
won’t have to re-read the entire essay, you can focus only on those portions singled
out by the questions
...
A thorough
reading at this point wastes time and may distract you from your goal—to answer six
questions correctly
...
Then turn to the questions
...
After two readings, one quick and one slow, you’ll know the essay intimately
...

Which of the three methods works best for you can be determined only by experience
...
Stick to the one that
works best for you and practice it over and over
...
In fact, they should sound familiar
because they are related to matters of essay writing discussed earlier in this ebook
...


ANSWERING THE QUESTIONS
Improving Paragraphs Questions follow the progress of the passage
...
If you are good at
spotting faulty sentences, answer the sentence-error questions before tackling the others
...
If a
question stumps you, go on to the next one, but don’t forget to come back later and give it another try
...
[2] They also go faster in stores
...
[4] Not only are they more
fashionable but they give status to their owners
...

[6] For one thing, a canoe can last for more than thirty years
...
[8] For example, a new aluminum canoe may cost
about $600, but a used one costs about $500 or less
...
[10] Requiring almost no care at all, you only have to paint it every few years or bang out
some dents if you ride it through rapids
...

[12] Besides being economical, a canoe can be used in a variety of ways
...
[14] One can use it on rivers, too
...
[16] As a result, wherever you go, there is bound to be a
place for canoeing
...
[18] In contrast to other boats, canoes don’t depend on wind or fuel
...
[20] Simply grab a
paddle, and you’re off on your own
...
In the first paragraph the author ’s primary purpose is to
(A) inform the reader about several kinds of boats
(B) provide evidence that boating is a popular pastime
(C) poke fun at those who prefer sailboats to canoes
(D) tell a personal story about boating
(E) prepare the reader for an unexpected disclosure
Choice A is not a good answer because the paragraph is more about boaters than about boats
...

Neither choice C nor D is justified by the contents of the paragraph
...
In fact, the paragraph deals favorably with sailboats and motorboats but ends with
a slightly surprising statement—that the author prefers canoes
...

2
...
Choice C is wrong
because sentence 2 states a fact that can be proved, not an opinion
...
An analysis of the paragraph shows that the phrase go faster is used in both sentence 1 and
sentence 2
...
In sentence 2, it refers to the
popularity of boats among consumers
...

3
...

(A) As it is now
(B) Requiring little care, canoes need only to be painted
(C) Requiring little care, paint is all it needs
(D) Caring for it easily, you only have to paint it
(E) Only paint it
The sentence needs revision because it contains an error in modification
...

Choice C is a variation of A
...
Choice B, the only remaining choice, is the best answer
...
Which of the following should be done with sentence 11 (reproduced below)?
Some high-end canoes are made of canvas covering a sturdy wood frame, caned seats and
copper and brass trim
...

(B) Delete it; the sentence is irrelevant
...

(D) Combine it with sentence 4
...

Because the sentence contains information about the materials used to build canoes, it does not fit
logically anywhere in the essay
...

Therefore, choice B is the best answer
...
In context, which is the best way to revise and combine sentences 14 and 15 (reproduced
below)?
One can use it on rivers, too
...

(A) One can use it on rivers, too, as well as marshes and small streams
...

(C) You can use it on rivers, marshes, and small streams
...

(E) One can use it on rivers and small streams and in marshes, which are fine for using a
canoe
...
Choice D is a sentence fragment
...
Therefore, choice C, a concise and clear revision of
the original sentences, is the best answer
...
The primary effect of the final paragraph (sentences 12–20) is to
(A) summarize the ideas introduced in the previous paragraph
(B) reconsider a point made in the first paragraph
(C) support the validity of the essay’s main idea
(D) explain a contradiction within the essay
(E) provide an additional example
Almost every sentence in the final paragraph adds another dimension to the writer ’s appreciation of
canoes
...


TIP
After you’ve read the essay, jot down its main point or purpose
...


Defining the Essay’s Purpose
Once you’ve read the given essay, quickly jot down the essay’s main idea in your test booklet
...

Writers often have multiple purposes and complex attitudes toward their subject
...
Don’t look for subtleties, sophisticated
techniques, or hidden meanings
...
For example:
To inform readers about the progress of America’s fight against terrorism
...


To dispense helpful advice about how to use the Internet
...
Any material that oversteps the
boundaries is fodder for Improving Paragraphs questions
...
Be sure
it introduces, limits, and makes clear the purpose of the essay
...
Subsequent paragraphs
set up signposts along the way to remind readers where they’ve been and where they are headed
...

Say, for example, that a writer deplores the vast amount of cheating by high school students
...


In the outline, each paragraph discusses an important aspect of the cheating problem
...
If the writer, however, had included a paragraph on teenage
shoplifting or had compared the amount of cheating between girls and boys, the essay’s sensible
organization would have been violated
...
An unfocused essay contains distractions and irrelevancies
...
Or worse, the essay’s conclusion may undermine or
contradict its introduction
...

A question may ask you how to revise such a sentence or whether to move or delete it
...


Paragraph Structure, Unity, and Coherence
Knowing the qualities of well-written paragraphs and recognizing paragraphing weaknesses will help
you answer some questions
...
It has a purpose, an
organizational plan, and a progression of ideas
...


TOPIC SENTENCES AND SUPPORTING SENTENCES
Most paragraphs are made up of two kinds of sentences: A topic sentence, which states generally the
contents of the paragraph, and supporting sentences, which provide the particulars that support and

develop the topic sentence
...
The paragraph that follows contains examples of each
kind of sentence:
[1] Children with IQs well below average represent an almost insoluble problem for educators
...
[3] Failure in school is the number one cause of poor
behavior in school and of juvenile delinquency in general
...

[5] But vocational training is very limited in many schools
...

Sentence 1 is the topic sentence of the paragraph
...
Each supporting sentence adds a piece of evidence to prove the point of the paragraph
—that children with low IQs create a problem for schools
...

Location of Topic Sentences
...
It isn’t always a separate and independent sentence; it may be
woven into a supporting sentence as a clause or phrase
...
) Writers vary the location of
topic sentences to avoid monotony
...
Or they might omit the topic sentence, letting
an accumulation of telling details imply the paragraph’s main idea
...
[2] No one in his right mind wants to pry open their eyes and
leave the cozy warmth of bed and blanket
...
[4] The
thought of damp clothes and cold feet keeps you where you are, at least for a few more minutes
...

The supporting details in sentences 1–4 lead inevitably to sentence 5, the topic sentence, which
summarizes the point of the paragraph
...
[2] Reduced speed limits, seatbelt requirements, and increased police patrols had
almost no effect on changing the number of fatalities
...
[4] Front and side airbags were installed in
all new models
...
[6] Stronger steel
frames enabled people to survive crashes that would certainly have killed them before
...
It serves as the pivotal point between the description of the problem
(sentences 1 and 2) and some effective solutions (sentences 4, 5, and 6)
...
On the SAT,
you may be asked to improve a paragraph by tightening that link
...
A transitional

sentence links the ideas in one paragraph with those in a previous or subsequent paragraph
...
In short essays it’s rare to find full transitional sentences
...


UNITY AND COHERENCE IN PARAGRAPHS
When a paragraph deals with more than one main idea, it lacks unity
...
The Paragraph Improvement questions often ask about alien
sentences—sentences that undermine unity or weaken coherence
...
[2] Almost everyone who met him thought that he was
charming, dignified, charismatic
...
” [4] Yet the Father of Our Country had been soundly defeated in 1755, when he first
sought elective office
...
[6] His frame was padded with well-developed muscles,
indicating great strength, and his blue-grey eyes could sparkle with humor at one moment and
grow hard and determined at the next
...

The paragraph’s purpose is to describe the power of Washington’s personality
...


COHERENCE THROUGH SENTENCE COMBINING
Disjointed paragraphs force readers to slow down or even stop dead at the end of each sentence
...

On the SAT, you may be asked to improve a paragraph’s coherence by choosing a revision that
effectively combines two or three disconnected or repetitive sentences
...
[2] It was the annual spring dance
...
[4] Their shiny brass instruments were in their hands
...
[6] All the girls were dressed in pastel shades
...
[8] They were deciding which boys they would ask to dance
...
[10] Soon it was full
...
But it suffers
from incoherence because each detail, no matter how important or trivial, is stated in a separate
sentence
...
[2] Four men
in black tuxedos stood on the stage and provided the musical entertainment with their shiny brass
instruments
...
[4] Soon the floor was filled with dancing couples
...
Some words have been deleted or changed
...
Overall, the revision exemplifies more skilled, more
mature writing
...

As you weigh the five choices given by a sentence-combining question, keep in mind that the most
concise or cleverest revision may not always be the best one
...


Practice in Combining Sentences
Directions: Use the spaces provided to combine the sentences in each of the following groups
...
If necessary,
add, delete, and/or alter words
...


1
...
She is an expert gymnast
...





2
...
The accident was a hit and run
...





3
...
She bought tomato juice
...
The bottle was in the grocery bag
...
The bottle broke
...
The mess was on her hands
...
The baseball hit the picture window
...
Strickman
...
The glass shattered in a thousand pieces
...
There was a storm
...
Snow fell on the roads
...
I could not go out
...
I watched TV
...
Time passed slowly
...
The Earth revolves around the sun
...
The Earth rotates on
its axis
...
The revolution determines the length of the year
...





7
...
He lived in ancient Greece
...
The plays
were tragedies
...





8
...
Music often transports people’s minds
...
People often feel refreshed after listening to music
...
Human beings have skulls
...
The skull has twenty-two bones
...
The cranium protects the brain
...





10
...
The word Hopi means peaceful and happy
...
The culture lacks tension
...
Material
possessions are unimportant
...
So is restraint
...
The family is the highest value
...


Paragraph Development
Like an essay, each paragraph should have a recognizable plan
...
Or it may take the form of a brief
narrative, its events spelled out in the order they occurred
...

Depending on the paragraph’s purpose, details that support the main idea may be arranged
spatially, chronologically, in order of importance, from general to specific or vice-versa—or in any
arrangement that develops the topic
...
Therefore, you should
know the most common patterns of paragraph development
...

1
...
In this organizational plan, a paragraph’s supporting sentences consist of
arguments or examples meant to prove the validity of the topic sentence
...
Euphemisms enable both soldiers and civilians to
keep a psychological distance and turn war into an antiseptic, clinical abstraction
...
” In the Gulf War, as well as the Iraq and
Afghanistan conflicts, “friendly fire” became the phrase of choice
...
” Arms and legs are not blown off in combat; they are severed
in a “traumatic amputation
...

The first sentence is the topic sentence
...

2
...
Paragraphs of definition do more than simply offer a dictionary meaning of a word
or idea
...
For instance, the
following defines the word utopia by describing a utopian society and explaining the word’s
origin
...

Everything in the society, from its economic policies to its social practices, is designed to
keep the society functioning without difficulty
...
Utopia is an appropriate name
...

3
...
A spider ’s web is defined in the following paragraph by comparing a
web and a fine musical instrument
...
It is constructed of many strings of
different lengths under various degrees of tension
...
So sensitive is the
spider ’s sense of touch that from one corner of the web she can locate a struggling victim,
determine its size, and, by the rhythms and tempo of vibrations, judge it to be a moth, a
hapless mosquito, housefly, or other insect
...
Comparison and contrast
...

Albert Perry may have been the model for Hal Roet in Thayer ’s new novel
...
” The real-life Perry was a tobacco farmer for years and was
known throughout Piedmont County as Peripatetic Perry
...
Roet, too, left his farm in the
hands of his wife and traveled around the country with a rodeo
...

Perry was compulsively self-revealing; Roet was quiet and unassuming
...
Finally, Perry craved fame
...
He was in it for the thrill of doing something
dangerous
...
Cause and effect
...
The
following passage describes the effects of one-sixth gravity
...
They walk easily, each step evolving into a
rhythmic, bounding motion that feels like a stroll on a trampoline
...
To stop forward motion, they must dig their
heels into the ground and lean backward
...
Getting up again is difficult and
enervating, however
...
Process analysis
...

When repainting a room, it’s best to remove as much furniture and carpeting as possible
...
Using a roller, paint
the ceiling first
...
Then paint the walls
...
If you expect to
finish a can before you finish a wall, pour the paint from two cans into a large bucket and
mix well
...
Paint the baseboards last
...
Classification
...

Vegetables can be classified according to climate and growing requirements
...
Moderately hardy vegetables, including potatoes and
onions, should also be grown before the intense heat of summer
...
Some vegetables are extremely sensitive to cold and,
therefore, can be planted only weeks after the last frost
...
Such plants as tomatoes, peppers, and eggplant are
usually started indoors and transplanted outside in late spring or early summer
...

To prove a point or make a persuasive argument, for instance, a writer may combine facts with
definition and the analysis of a process
...
To develop an idea thoroughly often takes several sentences
...

Most sentences contain clues that assign them to a place—and only one place—in a paragraph
...
In the following, for instance, observe how
the italicized words and phrases determine the sequence of sentences:
[1] Part-time jobs for high school students are a mixed blessing
...
[3] It is also satisfying for young people to help with their family’s
finances
...
[5] Moreover,

many jobs are so boring that students get the idea that work and boredom go hand in hand
...
The
pronoun they, which begins sentence 2, refers to jobs, a noun in the first sentence
...
Sentence 4 begins with On the other hand, a common transitional phrase used to
indicate that a contrasting idea will follow
...
Because of these linking
elements, these five sentences cannot be arranged in any other sequence without destroying the
paragraph’s coherence
...
But they are not in the proper order
...


1
...
In the end, morale got so low that members started quitting the team
...
Whether you were a polevaulter, a sprinter, or a distance runner, practices were the same
for everyone
...
He was forcing the team to work out the same way every day
...
Mr
...

2
...
First, put in the large, firm, and heavy items that won’t be crushed or damaged by putting
something on top of them
...
Meanwhile, think of all the items that can be easily bruised, crushed, or broken, such as
eggs, packages of bread, fruit, and light bulbs
...
To fill up a paper bag with groceries usually takes about fifteen seconds if you do it right
...
Immediately after that, put in light but firm items such as crackers, cereal, and butter
...
Canned goods and bottles fit the bill perfectly
...
Those should be saved for last
...
____ a
...

____ b
...

____ c
...

____ d
...

4
...
As blood circulates, it cleans out body waste, like the collector who cruises the
neighborhood picking up trash
...
In return, it deposits oxygen and food in every body part, from the top of the head to the
little toe
...
Yet human life depends on those four quarts of blood that are pumped from the heart, flow

to every cell in the body, and return to the heart to be pumped again
...
If you drained the blood from the body of a girl weighing about 125 pounds, you would fill
little more than a gallon milk container
...
____ a
...

____ b
...

____ c
...

____ d
...

____ e
...

6
...
His mistake was corrected fifty years later by Carl Blegen of the University of Chicago
...
He figured out that every few centuries a new city had been built upon the ruins of the old
...
In the 1870s, the archeologist Heinrich Schliemann dug in the correct spot and discovered
nine ancient cities of Troy, one lying on top of the other
...
But without realizing it, Schliemann had dug right past the layer he had been seeking, the
layer containing the ruins of the famous city of the Trojan Horse
...
By then, it was too late for Schliemann, who had been dead for fifty years
...
____ a
...

____ b
...

____ c
...

____ d
...

8
...
He felt terribly anxious about his wounded leg
...
The slightest movement of his knee caused a sudden and intense pain, unlike anything he
had ever felt before
...
He could not sleep, in spite of the sedative administered to him by the British nurse
...
In Milan, the lieutenant lay in a hospital bed
...
It was even worse than the pain he recalled when, as a child, he had pulled a pot of steaming
water over on himself
...
____ a
...

____ b
...

____ c
...

____ d
...

10
...
He became blind in 1652 and used his daughter as an instrument to write some of his finest

poems
...
His daughter, with her quill pen in hand, sat with her father to record his thoughts, to read
them back, to make revisions in whatever way Milton wanted
...
The first poet to use a word processor was John Milton
...
The actual processing of words went on in Milton’s head
...
____ a
...

____ b
...

____ c
...

____ d
...


Functions of Paragraphs
A paragraph-improvement question may single out a paragraph or one of its parts and ask you to
identify its role in the essay
...
Part III, on essay writing, offers a thorough discussion of this topic, but here is a brief
overview
...
An effective opening paragraph introduces the essay and makes the intent of the
essay clear to the reader
...
SAT questions often refer
to sentences in the first paragraph that are irrelevant to the essay’s main idea
...
The final paragraph should give the reader a sense of completion
...
No ending is as effective
and emphatic as one that grows logically out of a thoughtful arrangement of the writer ’s ideas
...
Or it
may call on the reader to think about an issue or perform an action
...

Developmental Paragraphs
...
For example, a paragraph may carry forward the main point of the essay by
contributing a solution to the problem being discussed
...

On the SAT, you may be asked to identify the main function of a particular paragraph
...
Rather, it pertains to the role the paragraph plays in the journey from the
beginning to the end of the essay
...


A Review
While looking for errors in Improving Paragraphs Questions, use this checklist as a guide
...

How topic sentences signal the purpose and organization of each paragraph and of the essay as
a whole
...

The unity and coherence of each paragraph and of the whole essay
...

Transitional words and phrases
...


Answer Key to Practice Exercises
Writing Correct Sentences
Answers will vary
...

1
...


2
...


3
...


4
...


5
...
I force my eyes open and crawl to
the shower
...


6
...


7
...
Out into the morning sunshine emerges a rider
on a road bike
...
What are the rules? What happens if we break them?

9
...


10
...


Establishing Noun–Verb Agreement
1
...
heroes … die

3
...
Correct

5
...
are … levels

7
...
team … is

9
...
reforms … have

11
...
Politics … has

13
...
Katie Green … and accompanist … are

15
...
sale … has

17
...
insistence … is

19
...
are … guards

Choosing the Case of Pronouns
1
...
her, him

3
...
me

5
...
he

7
...
me

9
...
he

Recognizing Pronoun Shift and Pronoun Agreement
Answers may vary
...
The English teacher announced that everyone in the class must turn in his term paper no later than
Friday
...
When fired from a job, one collects unemployment
...
The library put its collection of rare books on display
...
Each of my sisters owns her own car
...
Correct

6
...


7
...
Then each student writes it down in English and hands it
in
...
Each horse in the procession followed its rider down to the creek
...
The school’s chess team has just won its first match
...
When you visit the park, ask a park ranger if you can’t find a rest room
...
Yours may be different but equally valid
...
When teenagers loiter outside the theater on Friday night, the police give them a hard time
...
Before collecting my pencils and pens, I handed in the test questions I had answered
...
At the root of Barbara and Ken’s problem is that she wanted only a short wedding trip to Florida
...
With only an hour to get to the airport, his father was in a rush and told him so
...
During her tenure in office, Dr
...


6
...


7
...


8
...


9
...
He also
married Felicia, which made his parents very happy
...
Because he had lived in California, Eddie grew fond of John Steinbeck’s novels
...
in battle

2
...
comply with

4
...
preoccupation with

6
...
Correct

8
...
to identify employees

10
...
with respect to

12
...
that kind of pain

14
...
as expensive as

16
...
between teaching and devoting

18
...
far from harm

20
...
bitter

2
...
smoothly

4
...
horrible

6
...
cynically

8
...
smoothly

10
...
oblivious

12
...
slowly

14
...
easily

16
...
badly

18
...
sincerely

20
...

1
...


2
...


3
...
I thought that without chemistry I couldn’t go to a good college
...
As a result of the election, the state will have its first female governor
...
My father habitually watches the sun set
...
Harold hasn’t stopped painting since picking up a brush at age ten
...
Research shows that avid sports fans suffer fewer depressions and are generally healthier than
those not interested in sports
...
He is a chemist
...
The cough recurred twice
...
Other answers may also be correct
...
Jane is more efficient than any other member of the committee
...
Andy looks more like his father than his brother does
...
In The Great Gatsby, I disliked Daisy as much as I disliked Tom
...
Correct

5
...


6
...


7
...


8
...


9
...


10
...


11
...


12
...


13
...


14
...
Scotch’s teaching style more than Mr
...


15
...


16
...


17
...
The lawyer insisted that her job took more hours than a teacher ’s job
...
Carrying iPods is more common among students than carrying cell phones
...
Cindy has applied to as many colleges as, if not more than, Joanne
...
… her physical

2
...
… her schedule

4
...
… their money

6
...
… it conducted itself

8
...
No error

10
...
Other answers may also be correct
...
Doing what she loves, Mrs
...


2
...


3
...


4
...


5
...


6
...


7
...


8
...


9
...


10
...


Recognizing Shifts in Pronoun Person and Errors in Pronoun Case
1
...
Sam and me

3
...
they collect

5
...
Jonathan and him

7
...
He and I

9
...
Their singing

11
...
No error

13
...
as I

15
...
came

2
...
No error

4
...
had reached

6
...
had finished

8
...
gave

10
...
stopped

12
...
Having worked

14
...
suffered

Identifying Faulty Verb Forms
1
...
arisen

3
...
blew

5
...
had burst

7
...
had never sung

9
...
to attract

Combining Sentences
Because many different answers are possible, these are suggestions only
...

1
...


2
...


3
...


4
...
Strickman’s picture window, shattering it into a thousand pieces
...
Since the storm dumped two feet of snow on the roads, I could not go out
...
The time passed slowly
...
The Earth revolves around the sun every 365 days
...
The Earth’s revolution around the sun determines the length of a year just as its
rotation determines the duration of a day
...
The 2,000-year-old tragedies of Euripides, an ancient Greek playwright, are still performed today
...
Music has the unique power to transport people’s minds
...


9
...


10
...
Lacking competitiveness, Hopis rarely feel tense
...
But the highest value is the family, consisting of the
entire Hopi tribe
...
a
...
3 c
...
1

2
...
2 b
...
1 d
...
3 f
...
a
...
1 c
...
2

4
...
3 b
...
2 d
...
a
...
2 c
...
1 e
...
a
...
2 c
...
3 e
...
a
...
2 c
...
3

8
...
3 b
...
2 d
...
5

9
...
3 b
...
2 d
...
a
...
4 c
...
3

11
...
1 b
...
3 d
...
Use the essay as an
opportunity to show how clearly and effectively you can express and develop ideas
...
Include specific evidence or examples to support your point of
view
...
The number of words is up to you, but quantity
is less important than quality
...
You’ll have enough space if you write
on every line and avoid wide margins
...

BE SURE TO WRITE ONLY ON THE ASSIGNED TOPIC
...

If you finish in less than twenty-five minutes, check your work
...
REMEMBER, for Practice Tests A-D, since this is an e-Book, to record all answers
separately
...

Describing his vision for the world’s future, President Franklin D
...
The
first is freedom of speech and expression…
...
The third is freedom from want…
...

All four freedoms are crucial in a free society
...

Assignment: Given the opportunity to add another freedom, what would you choose? Plan and write
an essay in which you explain your choice
...
Support your position with reasoning and examples
taken from your observations, experience, studies, or reading
...
Do not skip lines
...

The following pages for each practice test are for reference only
...

Do not proceed to Section 2 until the allotted time
for Section 1 has passed
...


SECTION 2
MULTIPLE-CHOICE QUESTIONS
TIME: 25 MINUTES
Improving Sentences
Directions: The underlined sentences and sentence parts below may contain errors in standard
English, including awkward or ambiguous expression, poor word choice (diction), incorrect
sentence structure, or faulty grammar, usage, and punctuation
...
Indicate your choice by filling in the corresponding space on the answer
sheet
...
Choose A if none of the other choices improves the
original sentence
...
The captain standing on the bridge of the ship, he had never seen such a strong wind in all his
years at sea
...
What can beat the thrill of kicking a soccer ball past the goalie for a score and to hear applause
from the crowd?
(A) of kicking a soccer ball past the goalie for a score and to hear applause from the crowd
(B) of kicking a soccer ball past the goalie for a score and to hear applause by the crowd
(C) of kicking a soccer ball past the goalie for a score and hearing the crowd applaud
(D) to kick a soccer ball past the goalie scoring a goal, and to hear the crowd applaud
(E) of hearing the crowd applaud after having kicked a soccer ball past the goalie for a score
3
...

(A) give audiences a thrill
(B) thrills the people who are watching
(C) give a thrill to the people in the audience watching it

(D) give thrills in the watching of it to the audience
(E) gives people thrills in watching it
4
...

(A) honoring those alumni who had graduated fifty years ago
(B) ceremonies for graduates of fifty years ago were held in honor of these alumni
(C) alumni graduating fifty years ago received honors
(D) alumni who had graduated fifty years ago were honored
(E) graduating alumni of fifty years ago were honored
5
...

(A) Bromley, the first historian to reveal the story of the Mandan tribe, doing it
(B) Bromley was the first historian to reveal the story of the Mandan tribe, and who did so
(C) Bromley was the first historian to have revealed the story of the Mandan tribe and does it
(D) Bromley was the first historian to reveal the story of the Mandan tribe, telling the tale
(E) Bromley, the first historian revealing the story of the Mandan tribe, the tale was told
6
...

(A) team, their performance in this having been exceptional
(B) team; they have performed exceptionally in this
(C) team, for they have performed exceptionally in this
(D) team; their performance having been exceptional
(E) team, for their performance has been exceptional
7
...

(A) increasingly influence the administration’s policies
(B) are being more and more influential in the administration’s policies
(C) are increasing their influence on the administration’s policies
(D) is becoming increasingly more influential in determining the administration’s policies
(E) increasingly influences the administration’s policies
8
...

(A) as if already they had died
(B) if they had already
(C) whether they had died already
(D) as though they had already died
(E) like they had died already
9
...

(A) Although the speakers never having reached their audience, they couldn’t
(B) The speakers never reached their audience, they couldn’t

(C) Never having reached their audience, the speakers couldn’t
(D) The speakers never reached their audience; however, they couldn’t
(E) The speakers never reached their audience because they couldn’t
10
...

(A) If we agree to purchase the house today
(B) Had we agreed to purchase the house today
(C) If we would of agreed to purchase the house today
(D) If the purchasing of the house had been agreed to today
(E) If today we will agree to purchase the house
11
...

(A) future because it requires
(B) future because it would require
(C) future because they require
(D) future, each requires
(E) future since they are requiring

Identifying Sentence Errors
Directions: The underlined and lettered parts of each sentence below may contain an error in
grammar, usage, word choice (diction), or expression (idiom)
...
Indicate your choice by filling in the corresponding
space on the answer sheet
...
Some sentences may contain
no error
...

EXAMPLE
Jill went

to the

of the hill in a

faster time

her friend, Jack
...


ANSWER

12
...


13
...

14
...


the same upper respiratory infection as

no doubt that the safe-driving campaign scheduled to begin next week
postponed for another week

we lack the time to plan it

16
...


17
...

18
...


as two of America’s greatest science fiction

more

500 books, is by far the

on the size of our national debt, one might
nation on Earth

how the most powerful

itself into such a mess
...
The achievements as well as the failures of the space program
many years as many interest groups compete
21
...

to arrest the suspect

the airport before he is
24
...


successfully use computers

23
...


lost in the mountains of Colorado for two days

22
...


that afternoon,

arrested him at

a flight to Atlanta
...
The names Leno and Letterman
comedian and Letterman

alphabetically close together,


Leno is

inland

26
...


a presentation

forming

groups, discussing the questions about the book,

of the class
...
At Burger King, the hamburgers are very
contain

a staff, a team of volunteers, a

they are



and

fat
...
Although a sales tax may seem like an efficient way for cities
harmful to low-income consumers

wealthy

revenues,

more



Improving Paragraphs
Directions: The passage below is the draft of a student’s essay
...
Read the passage and answer the questions that follow
...
Choose the answer that best follows the requirements of standard
written English
...

[1] It is difficult to deny that modern technology has changed human behavior
...
[3] Thousands of unknown musicians and actors, aiming to become famous, have turned to
electronic means to demonstrate their talent to producers and casting directors
...
[5] In the past, it
would cost many thousands of dollars to put out a sample tape or CD
...
[7] They don’t do their best work in a rush to save money
...
[9] It is expensive to send out hundreds of CDs to agents,
radio and TV stations, or any other place where someone might listen or look
...

[11] Now, however, flip cameras and other common video equipment can help budding performers
get their foot into show business with three-minute audition tapes posted on YouTube, Facebook, and
other Web sites
...

[13] Broadway has embraced this technology
...
[15] It may take weeks to screen them all
...

[17] Using technology is no guarantee of success
...
[19] Within a month over 4,000 listeners had
downloaded it
...
The sentence that best states the main idea of the essay is
(A) sentence 1
(B) sentence 3
(C) sentence 4
(D) sentence 11
(E) sentence 17
31
...

(A) can
(B) will
(C) might
(D) shall
(E) decided to
32
...
And they would incur huge expenses to
record a brief performance
...
Which of the following is the best revision of the underlined portion of sentence 12 (reprinted
below)?
Recording at their own pace, each audition won’t be ready to send out until it represents the
performer’s best work
...

34
...
Hundreds of actors, dancers, and singers show up for
in-person auditions
...
With online presentations, casting
directors can make decisions in seconds
...

(B) Because of the speed at which online presentations can be viewed, in contrast to in-person
auditions that usually attract crowds of Broadway hopefuls that may take weeks to screen,
Broadway’s casting directors have embraced this technology
...

(D) Decision making in seconds begins when actors, dancers, and singers by the hundreds sent
presentations online to Broadway casting directors, who have embraced this technology in
place of in-person auditions that take weeks to complete
...

35
...

(B) The group’s music, by the way, is an eclectic mix of buzz-saw drone, twangy guitar, and
hardcore drumming
...

(D) The experience of the Thrice-Towed Sloths, however, is a rare exception to the rule
...


End of Section 2
...
Do not proceed to Section 3 until the
allotted time for Section 2 has passed
...
Read each sentence carefully and
identify which of the five alternative versions most effectively and correctly expresses the meaning
of the underlined material
...
Choice A always repeats the original
...

1
...

(A) author has been criticized for plagiarizing
(B) author was criticized for plagiarizing
(C) author will be criticized for plagiarizing
(D) author would have been criticized for plagiarizing
(E) author, being criticized for plagiarizing
2
...

(A) plant, being built on the river bank, and is costing far more than expected
(B) plant is costing far more than expected, it is being built on the river bank
(C) plant being built on the river bank is costing far more than expected
(D) plant is costing far more than expected being built on the river bank
(E) plant to be built on the river bank and to cost far more than expected
3
...

(A) At five years old, my father took me to get a haircut for the very first time
(B) At five years old, my father took me to get my first haircut
(C) My father took me at five years old to get a haircut for the very first time
(D) When I was five, my father took me for my first haircut
(E) At age five I was taken by my father to get my first haircut, something I had never had
before
4
...

(A) doctors diagnosed his cancer
(B) when doctors diagnosed his cancer

(C) his cancer had been diagnosed by his doctors
(D) when cancer was diagnosed for him by doctors
(E) he had cancer diagnosed by his doctors
5
...

(A) these are the impressions that he incorporated
(B) the incorporation of these impressions were
(C) these impressions having been incorporated
(D) his incorporation of these impressions
(E) incorporated his impressions
6
...

(A) In this article it characterizes Collins as being brilliant, ruthless, and likely to resign soon
(B) Collins, characterized in this article as being brilliant, ruthless, and likely to resign soon
(C) In this article, Collins is characterized as brilliant, ruthless, and he is likely to resign soon
(D) This article, which characterizes Collins as brilliant and ruthless, says that he is likely to
resign soon
(E) This article, in which Collins is characterized as being brilliant, ruthless and likely to
resign soon
7
...

(A) even when they are for selfish aims
(B) even if its motive is selfish
(C) even if their motive is to be for its own selfish aims
(D) whether or not it is for their own selfish aims
(E) even whether or not their motive is for their own selfish aims
8
...

(A) photographs that offer vivid reminders to both veterans and
(B) photographs, which offers vivid reminders to both veterans and
(C) photographs, which offer both vivid reminders to veterans plus
(D) photographs; it offers vivid reminders to both veterans as well as
(E) photographs; this offers vivid reminders both to veterans and
9
...

(A) Maxwell Perkins, an editor at Scribners, helping
(B) Maxwell Perkins who worked as an editor and helped
(C) Maxwell Perkins edited at Scribners who helped
(D) Maxwell Perkins, an editor at Scribners, helped
(E) the editor, Maxwell Perkins, helped at Scribners
10
...

(A) Still being bought for its
(B) Still having been bought for its
(C) They are consistently bought for their
(D) Because they were bought for their
(E) Consistently bought for their
11
...

(A) and he has been responsible for maintaining public lands ever since
(B) since then his responsibility has been maintaining public lands
(C) wherever since he is responsible for maintaining public land
(D) he has been responsible for maintaining public lands since then
(E) and since then is responsible for maintaining public lands
12
...

(A) variations in the speed of underground water are many
(B) underground water travels at various speeds
(C) the speed of underground water varies
(D) underground water speeds vary
(E) running underground water vary in speed
13
...

(A) They not only spoke enthusiastically about the new fertilizer but also praised
(B) They not only spoke enthusiastically about the new fertilizer but also praising
(C) They not only spoke enthusiastically about the new fertilizer but also to praise
(D) They spoke enthusiastically not only about new fertilizer but also praising
(E) They spoke both enthusiastically about the new fertilizer, but they also praised
14
...

(A) gang, and the dialogue was made to sound authentic
(B) gang and making the dialogue sound authentic
(C) gang and made the dialogue sound authentic
(D) gang, with the result being that the dialogue is authentic-sounding
(E) gang in where the dialogue sounds authentic
End of Section 3
...

END OF WRITING TEST
...
The essay you
wrote may contain some of the same or similar ideas
...
Your approach to the question may be at least as valid as any of those described
here:
Because the prompt invites you to choose any freedom you wish, you can pick one that affects you
personally or one with broad implications for society and the world
...
Or you might discuss freedom more broadly in an essay that argues for a
freedom that releases mankind from the scourge war or one that decreases our dependence on fossil
fuel
...

A good essay can also be written on a less weighty matter
...
You might speak up for the freedom to
stay out as late as you wish
...
Come to think of it, at this time in your life, you might
have a good deal to say about being free of the SAT and other college admissions hurdles
...
Enter your scores in the spaces
provided, and calculate the average of the six ratings to determine your final score
...
The score will be reported to you as the sum of the two ratings,
from 12 to 0
...

Therefore, this scoring guide may not yield a totally accurate prediction of the score you can expect
on the exam
...


Overall Impression
6 Consistently outstanding in clarity and competence; very insightful; clearly demonstrates a
command of writing skills; few, if any, errors
5 Generally effective and reasonably consistent in clarity and competence; occasional errors
or lapses in quality; contains some insight
4 Adequate competence; some lapses in quality; fairly clear and with evidence of insight
3 Generally inadequate but demonstrates potential competence; contains some confusing
aspects

2 Seriously limited; significant weaknesses in quality; generally unclear or incoherent
1 Demonstrates fundamental incompetence; contains serious flaws; significantly undeveloped
or confusing
Score

Development of Point of View
6 Fully developed with clear, convincing, and appropriate supporting material; demonstrates
high level of critical thinking
5 Generally well developed with relevant examples, reasons, and other evidence to support a
main idea; demonstrates strong critical-thinking skills
4 Partly develops a main idea with relatively appropriate examples and reasons; shows some
evidence of critical thinking
3 Weak development of main idea and little evidence of critical thinking; barely appropriate
examples or other supporting material
2 Lacks a focus on a main idea; weak critical thinking; inappropriate or insufficient evidence
1 Fails to articulate a viable point of view; provides virtually no evidence of understanding
the prompt
Score

Organization of Ideas
6 Extremely well organized and focused on a main idea; supporting evidence presented in an
effective, logical sequence
5 Generally well organized and reasonably focused on a main idea; mostly coherent and
logical presentation of supporting material
4 Reasonably organized; shows some evidence of thoughtful sequence and progression of
ideas
3 Limited organization and vague focus on main idea; contains some confusion in the
sequence of ideas
2 Barely recognizable organization; little coherence; serious problems with sequence of
ideas
1 No discernable organization; incoherent sequence of ideas
Score

Language and Word Choice
6 Highly effective and skillful use of language; varied, appropriate, and accurate vocabulary
5 Demonstrates competence in use of language; appropriate and correct vocabulary
4 Adequate but inconsistent use of effective language; conventional but mostly correct use of
vocabulary
3 Some minor errors in expression; generally weak or limited vocabulary; occasionally
inappropriate word choice
2 Frequent errors in expression; very limited vocabulary; incorrect word choice interferes

with meaning
1 Seriously deficient in use of language; meaning obscured by word choice
Score

Sentence Structure
6 Varied and engaging sentence structure
5 Reasonably varied sentence structure
4 Some sentence variation
3 Little sentence variation; minor sentence errors
2 Frequent sentence errors
1 Severe sentence errors; meaning obscured
Score

Grammar, Usage, and Mechanics
6 Virtually or entirely error-free
5 Relatively free of technical flaws
6 Some minor errors; one or two major errors
3 Accumulated minor and major errors
2 Contains frequent major errors that interfere with meaning
1 Contains severe errors that obscure meaning
Score

ANSWERS TO MULTIPLE-CHOICE QUESTIONS
SECTION 2
1
...
C
3
...
D
5
...
E

13
...
B
15
...
B
17
...
D

25
...
C
27
...
A
29
...
B

7
...
D
9
...
B
11
...
C

19
...
A
21
...
B
23
...
E

31
...
E
33
...
A
35
...
D
7
...
A
9
...
E

11
...
B
13
...
C


SECTION 3
1
...
C
3
...
A
5
...
The essay counts for roughly 30 percent of the final score; the multiplechoice questions, for roughly 70 percent
...




Answer Explanations
Note: Although some choices contain multiple errors, only one or two major errors are explained
for each incorrect choice
...
C A
...
The construction that begins with The captain standing has no
grammatical relationship with the main clause
...
Redundancy
...

D
...
The infinitive phrase that begins To stand is not logically related to the
main clause
...
Faulty coordination
...

EASY

2
...
Faulty parallelism
...
The phrases of kicking and to hear are not parallel
...
Idiom error
...
Use of or from
...
Idiom error
...

E
...
The phrase after having kicked a soccer ball should not refer to crowd
but to the scorer of the goal
...

MEDIUM

3
...
Subject–verb agreement
...
Use thrill
...
Wordiness
...

D
...

E
...
The pronoun it fails to refer specifically to a noun or other pronoun
...
D A
...
The construction lacks a subject and a verb
...
Wordiness and redundancy
...

C
...
The verb graduating implies present action, but the sentence refers to events in
the past
...
Diction error
...

HARD

5
...
Sentence fragment
...

B
...
The clause beginning and who did so is grammatically unrelated to the
first clause
...
Shift in verb tense
...

E
...
The subject shifts from Bromley to the tale
...
E A
...
The pronoun this fails to refer to a specific noun or other pronoun
...
Same as A
...
Same as A
...
Sentence fragment
...
The construction that starts
their performance lacks a verb and is, therefore, incomplete
...
E A
...
Chief of staff is singular; influence is plural
...
Subject–verb agreement
...

C
...
Chief of staff is singular; are increasing is plural
...
Wordiness
...

HARD

8
...
Syntax error
...

B
...
The verb should be had died
...
Diction error
...

E
...
Like introduces a phrase; as introduces a clause
...

HARD

9
...
Faulty subordination
...

B
...
A comma may not be used to separate two independent clauses
...
Sentence shift
...

D
...
The word however makes little sense in the context
...
B A
...
In context, the verb agree indicates future action and is inconsistent with
the main verb of the sentence would have saved, which indicates action already completed
...
Faulty diction
...
Use would have
...
Clumsy construction
...

E
...
The future action indicated by the verb will agree is inconsistent with the
main verb would have saved, which indicates action already completed
...
C A
...
Flights is plural; it is singular
...

B
...

D
...
A comma may not be used to separate two independent sentences
...
Verb tense
...
The verb are
requiring pertains to the present
...
C Noun–verb agreement
...
Use gets
...
D Faulty parallelism
...
Use creating
...
B Pronoun choice
...

MEDIUM

15
...
Use the future tense to describe events taking place in the future
...

MEDIUM

16
...
Use and instead of with
...
E No error
...
D Faulty comparison
...

MEDIUM

19
...
The sentence, cast in third person, shifts to second person
...

EASY

20
...
The subject achievements is plural; the verb has been is singular
...

EASY

21
...
The participle being should be used only to describe an action occurring at the
same time as the action described by the main verb
...

HARD

22
...
The phrase in the bettering of is nonstandard
...

EASY

23
...
The sentence is cast in the past tense
...

EASY

24
...

MEDIUM

25
...
Add –er to the positive form when comparing two entities
...
Use quicker
...
C Faulty parallelism
...
Use giving
...
B Subject–verb agreement
...
Use stand
...
A Faulty comparison
...
Use similar to those at
McDonald’s
...
B Pronoun–antecedent agreement
...
The pronoun–verb phrase they
are is plural
...

HARD

SECTION 2—IMPROVING PARAGRAPHS
30
...
Ideally, it ought to be eliminated
...
It is the best answer
...

Choice D is too limited to be the main idea
...

Choice E steers the essay in a new direction
...

MEDIUM

31
...

Choice B, a verb in the future tense, does not fit a passage that uses the conditional word
would before each verb
...

Choice D expresses a future condition that, in context, is both irrelevant and inappropriate
...

MEDIUM

32
...
It needs punctuation—either a period or a semicolon—between
money and they
...

Choice C represents the syntax of a linguistically challenged writer
...

Choice E is an accurate and cogent version of the original
...

HARD

33
...
The phrase Recording at their own pace should
modify performers, not audition
...

Choice C corrects the dangling modifier in sentence 12 and correctly says what the author
intended
...

Choice D, besides being wordy, contains two pronouns (they and their) with ambiguous
antecedents
...
The phrase Recording at their own pace should
modify performers, not auditions
...

HARD

34
...
It is the best answer
...
In an effort to contrast
online presentations with in-person auditions, however, the writer has mistakenly and
illogically contrasted speed and auditions
...

Choice D fails to emphasize the point of the four original sentences, namely, Broadway’s
embrace of technology
...
A
shift of verb tenses from present to past weakens the sentence still further
...
But it uses the word their, a
pronoun without a specific antecedent
...

HARD

35
...
Only choice D
refers directly to that point
...

MEDIUM

SECTION 3—IMPROVING SENTENCES
1
...
Shift in verb tense
...

C
...
The verb will be criticized shifts the sentence, cast in the past tense, to
the future tense
...
Shift in verb tense
...

E
...
The construction lacks a main verb
...
C A
...
The use of and introduces a construction grammatically and logically
unrelated to the earlier part of the sentence
...
Comma splice
...

D
...
The word order is not standard English
...
Sentence fragment
...

MEDIUM

3
...
Misplaced modifier
...

B
...
At five years old should modify I (the speaker), not my father
...
Wordiness
...

E
...
First and never had before are redundant
...
A B
...
The construction lacks a main verb
...
Shift in verb tense
...

D
...
The construction lacks a main verb
...
Wordy
...

EASY

5
...
Faulty coordination
...

B
...
The singular subject incorporation requires a singular verb
...

C
...
The second clause lacks a main verb
...
Sentence fragment
...

MEDIUM

6
...
Faulty pronoun reference
...

B
...
The construction lacks a main verb
...
Faulty coordination
...

E
...
The grammatical subject, article, lacks a verb
...
B A
...
The noun organization is singular; the pronoun they is
plural
...

C
...
The pronoun shifts plural to singular
...
Pronoun reference
...

E
...
The phrase their own selfish aims is redundant
...

HARD

8
...
Noun–verb agreement
...
Use offer
...
Misplaced modifier
...

D
...
Both and as well as are redundant
...

E
...
The pronoun this lacks a specific referent
...
D A
...
The –ing form of a verb may not serve as the main verb of a sentence
without a helping verb
...
Sentence fragment
...

C
...
The pronoun who refers to Scribners instead of Perkins
...
Idiom error
...
is not standard
English
...
E A
...
The noun teddy bears is plural; the pronoun its is singular
...

B
...
The sentence is cast in the present tense, then incorrectly shifts to the
past participle, having been
...
Comma splice
...

D
...
The subordinating conjunction because fails to establish a logical
relationship between clauses
...
A B
...
A comma may not be used to separate two independent clauses
...
Idiom error
...

D
...
A comma may not be used to separate two independent clauses
...
Verb tense
...
Use has been
...
B A
...
The participial phrase that begins Flowing through should modify
water instead of variations
...
Dangling participle
...

D
...
The participial phrase that begins Flowing through should modify
water instead of speeds
...
Subject–verb agreement
...
Use varies
...
A B
...
The verb spoke is not parallel in form to praising
...
Faulty parallelism
...

D
...
The phrase about the new fertilizer spoke is not parallel to the verb
praising
...
Incomplete construction
...

MEDIUM

14
...
Mixed construction
...

B
...
The –ing form of a verb may not serve as the main verb of a sentence
without a helping verb
...
Wordy
...

E
...
The phrase gang in where is not standard English
...
Use the essay as an
opportunity to show how clearly and effectively you can express and develop ideas
...
Include specific evidence or examples to support your po nt of
view
...
The number of words is up to you, but quantity
is less important than quality
...
You’ll have enough space if you write
on every line and avoid wide margins
...

BE SURE TO WRITE ONLY ON THE ASSIGNED TOPIC
...

If you finish in less than twenty-five minutes, check your work
...

See Practice Test A
...


Passage 1
Educators recognize that academic growth is the highest priority of a school
...
These restrictions are intended for the
students’ guidance and assistance and are meant to contribute to the achievement of the goals set
by schools in carrying out their mission
...

Of that number, about 250 make it to the NFL and about 50 make an NBA team
...

• The odds of a high-school football player making it to the pros at all—let alone having a
career—are about 6,000 to 1; the odds for a high school basketball player—10,000 to 1
...
Dempsey,
President of the National Collegiate Athletic Association

Assignment: In many high schools, students who fail a certain number of academic courses are
ineligible to participate in varsity athletics and other time-consuming extracurricular activities
...
Support your opinion with evidence and examples
drawn from your studies, reading, observation, or experience
...
Do not skip lines
...


End of essay
...




Answer Sheet for
Multiple-Choice Questions

SECTION 2
MULTIPLE-CHOICE QUESTIONS
TIME: 25 MINUTES
Improving Sentences
Directions: The underlined sentences and sentence parts below may contain errors in standard
English, including awkward or ambiguous expression, poor word choice (diction), incorrect
sentence structure, or faulty grammar, usage, and punctuation
...
Indicate your choice by filling in the corresponding space on the answer
sheet
...
Choose A if none of the other choices improves the
original sentence
...
All along the road in Pennsylvania, one sees wonderful old horse barns, each with its unique
decorations
...
As a student at Tulane University, where Julie grew to love history with a passion, ultimately
leading to a job as a historian at the United States Library of Congress
...
The letter was intended for Betsy and him, but the actual recipients of the bad news were Peter
and I
...

(D) Betsy and he, but the actual recipients of the bad news turned out to be Peter and me
(E) Betsy and I, but the bad news was actually received by Peter and I
4
...

(A) Funds that are earned as tips is
(B) How money that gets earned as tips is
(C) Earning tips are
(D) Tips are
(E) The funds earned by tipping is
5
...

(A) are as similar as
(B) being similar to
(C) resembling
(D) is the same as
(E) are similar to
6
...

(A) than
(B) then there was in
(C) than in
(D) than around
(E) compared to
7
...

(A) that was when Clarissa sighted her old guitar in the pawn shop window
(B) Clarissa sighted her old guitar in the pawn shop window
(C) then the sighting of Clarissa’s old guitar took place
(D) Clarissa’s old guitar was sighted in the pawn shop window
(E) in the window of the pawn shop her old guitar was sighted by Clarissa
8
...
Smart, agreed to study the school’s schedule and she would meet with
students, teachers, and administrators to discuss it
...
Because dinosaurs were the hugest creatures ever to roam the Earth is the reason why they are
fascinating to us
...
Reciting poems, one of the earliest forms of entertainment, it increased in complexity as well as
sophistication as time went on
...
Thomas Wolfe, the early 20th-century American writer often confused with the contemporary
novelist Tom Wolfe, grew up in Asheville, North Carolina
...
Read each sentence carefully and
identify which item, if any, contains an error
...
No sentence contains more than one error
...
In that case, the correct choice will always be E (No error)
...



...


Baker tells stories

before he entered high school, when he

deliver newspapers to the huge estates and mansions
13
...


the two Hemingway novels I have read, I like A Farewell to Arms
of its structure but also

of

14
...


success making sales

they

customers in the

late in the day
...
Child psychologists

young children

are pushed

activities prematurely

the pain of failure and frustration
...


Martin Luther King’s birthday January 15,
January,

on the third Monday of

of the date
...
Work in specialized fields

bacteriology, public health, and physics

at least a

bachelor ’s degree, and for a career in management or research, a master ’s degree

a

doctorate required
...
The governor
from

reductions in the sales tax
they go to the store
...
If Daniel Radcliffe

alive

the heyday of Hollywood’s debonair leading actors, he

probably would have been considered

20
...


consumers can benefit

and boyish

as a big star
...


the president nor

of his aides

able to

attend the ceremony
...
Melissa was taught early in life

feelings, she should

wear a smile,

try to be cheerful and upbeat, and never say anything about
23
...


24
...

25
...

26
...

27
...

28
...

29
...


Improving Paragraphs
Directions: The passage below is the draft of a student’s essay
...
Read the passage and answer the questions that follow
...
Choose the answer that best follows the requirements of standard
written English
...

[1] Much of Russia lies under a cover of snow and ice for most of the year
...
[3] Ports in northern Russia are not navigable for most of the year because they are frozen in
...
[5] The reason that the Black

Sea is important is because it gives them the ability to export timber, furs, coal, oil, and other raw
materials that are traded for food and manufactured goods
...

[7] The English Channel has served as a barrier between Great Britain and the rest of Europe
...
[9] Except for the Norman invasion more
than 900 years ago
...
[11] The isolation of Great Britain allowed the industrial revolution to begin in England
...
[13] The desert is irrigated by the Nile River
...
[15] The land along the river has historically been the site of farms
and other settlements
...
[17] Since building the Aswan High Dam in 1968, the farmers downstream from
the dam have been using artificial fertilizer
...
[19] Therefore, Egypt’s people depend on the Nile
...

30
...

(A) that it enables Russia to export
(B) its ability to allow exports of
(C) the ability of Russia to export
(D) because of exporting opportunities of
(E) for Russian exports of
31
...
Except for the Norman invasion
more than 900 years ago
...

(B) It has prevented attacks, except for the Norman invasion in 1066, on Great Britain for
hundreds of years
...

(D) It has prevented attacking Great Britain for 900 years, except the Normans
...

32
...
The desert is irrigated by the Nile River
...

(A) The Nile, the longest river in the world, irrigates the desert that covers much of Egypt
...

(C) The desert, which covers much of Egypt is irrigated by the Nile, which is longer than any
river in the world
...

(E) Much of the desert covering much of Egypt lies alongside the Nile, the longest river in the
world, and much of it is irrigated by it
...
To improve the coherence of paragraph 3, which of the following sentences would be best to
delete?
(A) Sentence 15
(B) Sentence 16
(C) Sentence 17
(D) Sentence 18
(E) Sentence 19
34
...
Considering the essay as a whole, which one of the following least accurately describes the
function of sentence 20?
(A) It summarizes the essay’s main idea
...

(C) It proves the validity of the essay’s main idea
...

(E) It gives the essay a sense of completion
...

Do not return to Section 1
...




SECTION 3
MULTIPLE-CHOICE QUESTIONS
TIME: 10 MINUTES
Improving Sentences
Directions: The underlined sentences and sentence parts below may contain errors in standard
English, including awkward or ambiguous expression, poor word choice (diction), incorrect
sentence structure, or faulty grammar, usage, and punctuation
...
Indicate your choice by filling in the corresponding space on the answer
sheet
...
Choose A if none of the other choices improves the
original sentence
...
The president of the company grew up in poverty, he gradually turned his life around
...
The strength and appearance of denim fabric account for its popularity among campers, hikers,
and other outdoor enthusiasts
...
In his speech, the candidate made a comment of spending sleepless nights worrying over the
large number of people without health insurance
...
As the sales of hybrid cars continue to rise, automakers asserting that it is working on the
improvement of mileage by spending profits on research and development
...
During Andy’s freshman year in college, his academic advisor suggested that he consider
majoring in music, English, creative writing, or studying medicine
...
The game warden believes that bow-hunting is safer than a rifle
...
Convinced that her all-night study sessions on Thursdays enabled her to pass math tests on
Friday, Susan is shocked to learn that last week she got an F
...
When someone works as a ranger in a national park, you will be employed by the U
...

Department of the Interior, a branch of the federal government
...
Having ordered a cup of black coffee at the Starbucks counter, the fumes smelled pleasing to
Howard
...
When Charlotte arrived at school on Tuesday morning, her friend Thalia told her that her first
period class had been cancelled
...
When you plan a plane trip, you choose a flight, make a reservation, and then that reservation
entitles you to a seat on the aircraft
...
Better military equipment, such as stronger bullet-proof vests, has been valuable so that it helps
soldiers feel more secure when they go into battle
...
A teacher ’s job is to set a good example for children as well as teaching them the material they
need to know
...
This book shows readers not only what might happen if they try to deal with the problem by
themselves but it’s all right to seek help
...

Do not return to Sections 1 or 2
...




Answer Key
SECTION 1—THE ESSAY
What follows are ideas for answering the essay question given in this practice test
...
If not, however, don’t be alarmed
...
After all, you may know people who are disasters in the
classroom but are super jocks
...
Considering how few high school athletes turn professional later in life, nothing should
keep students from developing and mastering essential survival skills
...

A third position might be to take a middle ground
...
One
student might work harder in class if he knows that he can’t participate in sports or other activities
without better grades
...


SELF-SCORING GUIDE
Using this guide, rate yourself in each of these six categories
...

On the SAT itself, two readers will score your essay on a scale of 6 (high) to 1 (low), or zero if you
fail to write on the assigned topic
...

Remember that SAT essays are judged in relation to other essays written on the same topic
...
Because it is difficult to read your own essay with total objectivity, you might improve
the validity of your score by getting a second opinion about your essay from an informed friend or a
teacher
...
A
2
...
B
14
...
D
26
...
A
4
...
E
6
...
B
8
...
B
10
...
A
12
...
B
16
...
B
18
...
A
20
...
D
22
...
B
24
...
C
28
...
C
30
...
E
32
...
C
34
...
C


6
...
B
8
...
E
10
...
B
12
...
B
14
...
C
2
...
D
4
...
C

Performance Evaluation Chart

Conversion Table
This table will give you an approximation of what your score would be if this practice test had been
an actual SAT Writing Test
...

For example, if your Multiple-Choice Raw Score was 35 and your Essay Subscore was 6, the table
indicates that your final score on the test would be approximately halfway between 500 and 710, or
600
...


TIP
Key to Levels of Difficulty

Rating
EASY
MEDIUM
HARD

Percentage of students
likely to answer correctly
>80%
>65%
<65%

SECTION 2—IMPROVING SENTENCES
1
...
Pronoun–antecedent agreement
...

C
...
The construction each of them have unique decorations and the phrase of their
own are redundant
...
Pronoun–antecedent agreement
...

E
...
A comma may not be used to separate two independent clauses
...
B A
...
The construction consists only of subordinate clauses and lacks a main
verb
...
Mixed construction
...
Misplaced modifier
...

E
...
The construction lacks a main verb
...
A B
...
The pronoun in Betsy and I is an object of a preposition, and, therefore,
should be me
...
Pronoun error
...

D
...
He should be him
...

E
...
In both instances, I should be me
...

EASY

4
...
Subject–verb agreement
...

B
...
Choice B is not grammatically or logically related to the rest of the
sentence
...
Subject–verb agreement
...

E
...
In standard English, the phrase is earned from tips
...
E A
...
In this context, the phrase are as similar as is not standard English
...
Sentence fragment
...

C
...
The construction lacks a main verb
...
Subject–verb agreement
...

MEDIUM

6
...
Faulty comparison
...

B
...
In making comparisons, use than instead of then
...
Faulty idiom
...

E
...
Violence is compared to the 1950s and 1960s, an illogical comparison
...
B A
...
The use of both While and that was when creates a redundancy
...
Mixed construction
...

D
...
The construction While walking down Market Street modifies guitar
instead of Clarissa
...
Passive construction
...

HARD

8
...
Wordiness
...

B
...
The phrase meeting … for discussing it is awkwardly expressed
...
Faulty idiom
...

D
...
The phrase discussing about it is not standard English
...
B A
...
Because and the reason why are redundant
...
Subject–verb agreement
...

D
...
As a result of dinosaurs fails to relate grammatically to they are
fascinating to us
...
Comma splice
...

MEDIUM

10
...
Mixed construction
...

B
...
The pronoun they fails to refer to a specific plural noun or other
pronoun
...
Sentence fragment
...

D
...
The coordinate phrases in their complexity and growing more
sophisticated are not in parallel form
...
A B
...
The phrase mistaken with is not standard English
...
Faulty coordination
...

D
...
Growing up in Asheville … modifies the contemporary Tom Wolfe
instead of the earlier writer Thomas Wolfe
...
Verb tense error
...

HARD

SECTION 2—IDENTIFYING SENTENCE ERRORS
12
...
Although the sentence is cast in the present tense (tells), it recalls past events
...

EASY

13
...
For comparing two objects, use the comparative degree instead of the
superlative
...

MEDIUM

14
...
The sentence is cast in the present tense
...

EASY

15
...
Use who instead of which when referring to people
...
E No error
...
B Subject–verb agreement
...
Use requires
...
C Pronoun–antecedent agreement
...

MEDIUM

19
...
Because this sentence calls for use of the past perfect tense, use had been instead
of was
...
D Faulty parallelism
...
Use making instead of he
made
...
D Subject–verb agreement
...
Use was
...
B Idiom error
...
Use regardless of
...
B Shift in pronoun person
...

MEDIUM

24
...
The nouns diversity and uniqueness should have a plural verb
...

HARD

25
...
Pronouns should refer to specific nouns or other pronouns
...

MEDIUM

26
...
An adjective may not be used where an adverb is needed
...

HARD

27
...
The phrase parents and I is the object of the preposition to
...

MEDIUM

28
...
In standard English, the phrase is different from instead of different than
...
C Faulty comparison
...
Use any other musical film
...
A Only Choice A is concisely expressed in standard English
...

MEDIUM

31
...

Choice B divides the main clause awkwardly
...
It unnecessarily repeats the English Channel, the subject of
sentence 7
...

HARD

32
...
The Nile cannot be longer than itself
...

Choice D is accurate but also wordy and repetitious
...

HARD

33
...
Therefore, choice C is the
best answer
...
D Sentence 11 states a complex idea that needs further explanation
...

HARD

35
...
The essay’s main idea is
validated by the contents of the essay’s three main paragraphs, not by the final paragraph
...
C A
...
A comma may not be used to separate two independent clauses
...
Idiom error
...

D
...
The clause beginning with but fails to relate logically to the earlier
part of the sentence
...
Mismatched sentence parts
...

EASY

2
...
Subject–verb agreement
...

C
...
The pronoun their should refer to fabric, but because their is plural, it
seems to refer to strength and appearance
...
Subject–verb agreement
...

E
...
The pronoun their should refer to fabric, but because their is plural, it
seems to refer to strength and appearance
...
D A
...
The construction made a comment of spending is not standard English
...
Clumsy construction
...

C
...
In context the verb gave is not standard English
...
Wordiness
...

EASY

4
...
Pronoun–antecedent agreement
...

B
...
The construction lacks a main verb
...
Faulty parallelism
...
Will improve is not parallel
to spending
...
Subject–verb agreement
...

MEDIUM

5
...
Faulty parallelism
...
Studying medicine is not
parallel to music, English, and creative writing
...
Faulty parallelism
...
Working in the field of
medicine is not parallel to music, English, and creative writing
...
Faulty parallelism
...
A profession in medicine is
not parallel to music, English, and creative writing
...
Mixed construction
...

MEDIUM

6
...
Faulty comparison
...

C
...
Bows is plural; is is singular
...

D
...
Add –er to one-syllable adjectives to indicate a higher degree
...

E
...
In this context, the phrase in hunting is awkward
...
B A
...
The sentence, cast in the past tense, shifts to the present
...
Misplaced modifier
...

D
...
The sentence, cast in the past tense, shifts to the present
...
Misplaced modifier
...

EASY

8
...
Tense shift
...

B
...
The pronoun it fails to refer to a specific noun or other pronoun
...
Pronoun shift
...

E
...
The construction is not in standard English
...
E A
...
The construction that begins Having ordered … should modify
Howard instead of the fumes
...
Dangling participle
...

C
...
The construction that begins Having ordered … should modify
Howard instead of smells
...
Diction error
...

MEDIUM

10
...
Ambiguous pronoun reference
...

B
...
The second her may refer either to Charlotte or to Thalia
...
Ambiguous pronoun reference
...

E
...
The construction lacks a main verb
...
B A
...
The sentence contains unnecessary repetition
...
Comma splice
...

D
...
The construction that begins with then entitling you fails to relate
grammatically with the previous part of the sentence
...
Idiom error
...
Put it after are
...
B A
...
The construction does not accurately convey the intended meaning
...

C
...
The phrase even though conveys just the opposite of the intended meaning
...
Pronoun reference
...

E
...
The construction is not in standard English and fails to convey a logical
meaning
...
B A
...
Coordinate elements in a sentence must be in parallel form
...

C
...
The pronoun they fails to refer to any specific noun or other
pronoun
...
Redundancy
...

E
...
See A
...
E A
...
The verb shows lacks a grammatical parallel, namely a verb in the same
form
...
Clumsy construction
...

C
...
The verb explaining is not parallel in form to shows
...
Idiom error
...

MEDIUM

Practice Test C
SECTION 1
ESSAY
TIME: 25 MINUTES
Directions: Plan and write an essay in response to the assigned topic
...
Present your
thoughts logically and precisely
...
A plain, natural writing style is probably best
...

Limit your essay to two sides of the lined paper provided
...
Write or print legibly because handwriting that’s hard or
impossible to read will decrease your score
...
AN ESSAY WRITTEN ON
ANOTHER TOPIC WILL BE SCORED “ZERO
...
Do not turn to another section of
the test
...

Think carefully about the following passage and the assignment below
...

The kind of envy to which Robinson refers may serve as a strong motivating force for some
people to improve their condition and place in life
...


Assignment: Is envy generally a positive force in people’s lives? Plan and write an essay in which you
develop your point of view on this issue
...


Section 1
Essay
Time allowed: 25 minutes
Limit your essay to two pages
...
Write only inside the box
...

Do not proceed to Section 2 until the allotted time
for Section 1 has passed
...
Read each sentence carefully and
identify which of the five alternative versions most effectively and correctly expresses the meaning
of the underlined material
...
Choice A always repeats the original
...

EXAMPLE
ANSWER


My old Aunt Maud loves
to cook, and eating also
...
The book’s descriptions of the country and the town, in addition to its recent release as a movie,
explains why sales of the book have suddenly boomed
...
Jogging a mile uses the same number of calories as if you walk two miles
...
Because Lisa expected not to go to college, she is taking little interest in school and doing
poorly
...
No one was more happier than me that you won a college scholarship
...
After 9/11, the American public accepted the government’s authority about more thorough
screening of airline passengers
...
The pollution of the municipal well was discovered, the town posted notices urging people to
boil their water
...
Whether Troy in fact communicated or did not with Gabriella is unclear, but a letter or phone
call would definitely have given her morale a boost
...
Last year, Fairfield High School defeated every one of its rivals in girls’ field hockey,
nevertheless, they did not win the state championship
...
Because it is blessed with a mild and pleasant climate, southern California has become more
popular than Florida as a place to retire
...
Wandering through the town that he had once called home, everything had changed: the
barbershop was gone, the deli was now a laundromat, and his old house had been turned into a
condo
...
An event in Richard’s life story that moved me greatly was when he was separated from the
family
...
Read each sentence carefully and
identify which item, if any, contains an error
...
No sentence contains more than one error
...
In that case, the correct choice will always be E (No error)
...



...
The start of freshman year in college

the same anxieties

experienced by ninth graders entering high school
...
Edith Wharton’s novel Ethan Frome

into a movie

failed

the mood and

meaning of the
14
...


of scholarship winners than than last

will

from the dean of discipline and

after being told that they


...


16
...

17
...

18
...
Harrison that without

unable

every child to read at grade level
...


the schools

Katie opened the refrigerator, she instantly noticed that a huge chunk of chocolate icing had
been

the birthday cake and immediately suspected that Mark

20
...
Davis is a first-rate trial lawyer who,
of capitalizing
21
...


a predator on the prowl,
opponents in the courtroom
...


22
...
An incident that embittered the colonists occurred
a group of protesters,
24
...


a Boston street

British troops

wounding six of them
...


25
...


results students

traditional family values
parents and

a dull lecture
...


28
...
Mr
...
Foremost

learning far more

boys

in a romantic way
...


Improving Paragraphs
Directions: The passage below is the draft of a student’s essay
...
Read the passage and answer the questions that follow
...
Choose the answer that best follows the requirements of standard
written English
...

[1] Back in 1900, who could have predicted the amazing technological achievements that would be
made during the next hundred years? [2] By the year 2000, advances in communications, nuclear
energy, and medicine transformed people’s lives in many unexpected ways
...
[4] One advantage of this
technology is that current events can be sent worldwide instantly
...
[6] When disaster struck the World Trade Center in
2001, most of the world saw it immediately
...
[8] As a result, many poor countries have been left behind and can
barely participate in so-called “globalization
...
[10]
One positive feature of nuclear energy is that energy is cheaper and can be made easy
...
[12] But the dangers of
nuclear energy dampen the world’s enthusiasm
...
[14] They can cause huge disasters such as the one in Chernobyl in 1986, which killed
countless people and radiated half the Earth
...
[16] One such
development was the MRI
...
[18] One positive effect of MRIs is that doctors can diagnose brain tumors and brain
cancer at an early stage
...
[20] But MRIs are expensive, so many
uninsured Americans and the poor of all countries have been deprived of this modern medical marvel
...
Considering the main idea of the essay, which of the following is the best revision of sentence 1
(reproduced below)?
Back in 1900, who could have predicted the amazing technological achievements that would be
made during the next hundred years?
(A) Back in 1900 who would anticipate the technological developments in the twentieth
century?
(B) Recent technological achievements would blow the mind of people in 1900
...

(E) Technological progress in communications, nuclear energy, and medicine is wonderful,
but in the process we are destroying ourselves and our environment
...
Which is the best revision of the underlined segment of sentence 10 (reproduced below)?
One positive feature of nuclear energy is that energy is cheaper and can be made easy
...
To improve the coherence of paragraph 2, which of the following is the best sentence to delete?
(A) Sentence 4
(B) Sentence 5
(C) Sentence 6
(D) Sentence 7
(E) Sentence 8
33
...

(A) Human error and careless workmanship are almost unavoidable
...

(C) There’s hardly no foolproof way to prevent human error and careless workmanship
...

(E) Accidents can happen accidentally by human error and careless workmanship
...
With regard to the entire essay, which of the following best explains the writer ’s intention in
paragraphs 2, 3, and 4?
(A) To compare and contrast three technological achievements
(B) To provide examples of the pros and cons of technological progress
(C) To analyze the steps needed for achievement in three areas
(D) To convince readers to be open to technological change
(E) To advocate more funds for technological research and development
35
...
Which of the following is the best way to continue the
paragraph?
(A) An MRI allows your doctors to make pictures of a brain to see if it has a growth on it, a
cancer is growing, or tumors at an early stage
...

(C) Making images with an MRI, your brain is studied by doctors for growths, brain tumors,
and cancer at an early stage
...

(E) With this device a doctor may look into a patient’s brain to check for growths and to detect
cancerous tumors at an early stage
...

Do not return to Section 1
...




SECTION 3
MULTIPLE-CHOICE QUESTIONS
TIME: 10 MINUTES

Improving Sentences
Directions: The underlined sentences and sentence parts below may contain errors in standard
English, including awkward or ambiguous expression, poor word choice (diction), incorrect
sentence structure, or faulty grammar, usage, and punctuation
...
Indicate your choice by filling in the corresponding space on the answer
sheet
...
Choose A if none of the other choices improves the
original sentence
...
During this decade, paying for a college education is more difficult for the average family than
it was in the past
...
Gwen Harper, one of the most popular students in the school, winning the election for class
president three years in a row
...

3
...

(A) Cervantes, and Cervantes was clearly ahead of his time
(B) Cervantes, who was clearly ahead of his time
(C) Cervantes, being clearly ahead of his time
(D) Cervantes, which novel was clearly ahead of its time
(E) Cervantes, Cervantes was clearly ahead of his time
4
...

(A) have high motivation or are significantly talented

(B) have either high motivation or else talent in significant amounts
(C) are highly motivated or who have significant talent instead
(D) are highly motivated or significantly talented
(E) have high motivation or else significant amounts of talent
5
...

(A) she dislikes city living and has never been east of the Mississippi
(B) she is without liking for city living nor having been east of the Mississippi
(C) she dislikes city living nor has she visited there
(D) she does not like city living and has never been a visitor
(E) it is without a liking for city living nor having visited there
6
...

(A) When you visit a foreign country, we can
(B) When visiting a foreign country, one can
(C) When we visit a foreign country, one can
(D) While you are on a visit to a foreign country, one can
(E) During our visiting of a foreign country, we
7
...

(A) Having Kelly Collins as our talented coordinator
(B) The having of the coordinating talent of Kelly Collins
(C) Kelly Collins as our talented coordinator
(D) To be coordinated by talented Kelly Collins
(E) The coordination talents of Kelly Collins
8
...

(A) When Beethoven’s music was introduced to the public for the first time, they found it
(B) When it was introduced to the public for the first time, they found Beethoven’s music
(C) When the music of Beethoven was introduced, the public found it
(D) When they were introduced to Beethoven’s music, the public had found it
(E) Introducing Beethoven’s music for the first time, the public found it
9
...

(A) an anonymous informant’s warning, and he
(B) the warning of an anonymous informant that he
(C) that an anonymous warning of an informant he
(D) the anonymous informant’s warning saying he
(E) information anonymously warning that he
10
...


(A) even if it is motivated by selfishness
(B) even when their motives are selfish
(C) even when the motive is to be for its own selfish aims
(D) whether or not it’s for their own selfish aims
(E) whether their motive is for selfish aims or not for selfish aims
11
...

(A) which is when a person completely relaxes their mind and body
(B) which is when a person completely relaxes his mind and body
(C) which is when someone completely relax their mind and body
(D) the complete relaxation of the mind and body
(E) which completely relaxes their mind and body
12
...

(A) lives, the reason was that they collected plant food for the family while protecting herself
and her children
(B) lives, it was that she had to collect plant food for the family while also protecting herself
and her children
(C) lives; collecting plant food for the family while protecting herself and her children
(D) lives because its responsibilities included collecting plant food for the family while
protecting themselves and their children
(E) lives because they not only collected plant food for the family but also protected themselves
and their children
13
...

(A) an important statewide problem, and it has become
(B) a problem of statewide importance, that is
(C) a problem of statewide importance; it was, therefore
(D) an important statewide problem that is
(E) an important statewide problem, which they have become
14
...

(A) more people drive on I-95 than any highway
(B) travelers are driving on I-95 in the largest numbers
(C) the largest amount of drivers are on I-95
(D) I-95 is the more heavily traveled
(E) I-95 is the most heavily traveled

End of Section 3
...

END OF WRITING TEST
...
The essay you
wrote may contain some of the same or similar ideas
...
Your approach to the question may be at least as valid as any of those described
here:
To argue that envy can occasionally be beneficial, find examples that show its potential to do good
...
Envy of others’ intellect, physical prowess, talents, and social
graces—all these and more can lead ambitious individuals to emulate, even surpass the
accomplishments of those they envy
...
)
For every instance of envy as a force for good, however, there are numerous situations that
illustrate the harm that envy can bring about
...
It often
indicates dissatisfaction with one’s own life and circumstances
...
In Shakespeare’s play Othello, the title character, a brave and
respected warrior, is tragically destroyed by envy, or more accurately, by jealousy, a common variety
of envy
...
James knows he can’t match Scott’s good fortune
...

Envy can cause trouble among workers in a factory, office, or school
...
Envy can poison the
atmosphere in a workplace and lower productivity
...
In addition,
history is filled with instances of nations craving the land, resources, seaports, or the riches of other
countries and sending armies to seize them
...


SELF-SCORING GUIDE
Using this guide, rate yourself in each of these six categories
...

On the SAT itself, two readers will score your essay on a scale of 6 (high) to 1 (low), or zero if you
fail to write on the assigned topic
...

Remember that SAT essays are judged in relation to other essays written on the same topic
...
Because it is difficult to read your own essay with total objectivity, you might improve
the validity of your score by getting a second opinion about your essay from an informed friend or a
teacher
...
B
2
...
B
4
...
B
6
...
D
8
...
A
10
...
C
12
...
E
14
...
A
16
...
C
18
...
B
20
...
B
22
...
E
24
...
C
26
...
D
28
...
B
30
...
C
32
...
A
34
...
E


SECTION 3
1
...
D
3
...
D
5
...
B
7
...
C
9
...
B

11
...
E
13
...
E


Performance Evaluation Chart

Conversion Table
This table will give you an approximation of what your score would be if this practice test had been
an actual SAT Writing Test
...

For example, if your Multiple-Choice Raw Score was 35 and your Essay Subscore was 6, the table
indicates that your final score on the test would be approximately halfway between 500 and 710, or
600
...


TIP
Key to Levels of Difficulty

Rating
EASY
MEDIUM
HARD

Percentage of students
likely to answer correctly
>80%
>65%
<65%

1
...
Subject–verb agreement
...

C
...
The participle having boomed is used instead of the present perfect have
boomed
...
Word choice
...

E
...
The singular verb is fails to agree with the plural subject
descriptions
...
D A
...
Elements being compared in a sentence must be in the same
grammatical form
...

B
...
See A
...
Faulty comparison
...

E
...
The pronoun it fails to refer to a specific noun or other
pronoun
...
B A
...
The verb tense improperly shifts from the past to the present progressive in the
second clause
...
Tense shift
...

D
...
The absence of the pronoun she causes meaning to be distorted
...
Tense shift
...

EASY

4
...
Faulty comparison
...
Use happier or more happy
...
Idiom error
...

C
...
In context the construction more happy like myself is not standard English
...
Tense shift
...

EASY

5
...
Idiom error
...

C
...
The phrase improve more thoroughly is awkward
...
Wordy and repetitious
...

E
...
In context the adjective thorough is incorrect
...

HARD

6
...
Comma splice
...

B
...
The first and second clauses are grammatically unrelated
...
Wordy
...

E
...
A comma may not be used to separate two independent clauses
...
D A
...
The phrase or did not is both misplaced and incomplete
...

B
...
The phrase or did not is both misplaced and incomplete
...

C
...
The phrase communicating by Troy with Gabriella is awkwardly
worded
...
Clumsy construction
...

MEDIUM

8
...
Comma splice
...

B
...
Fairfield High School is singular; they is plural
...

C
...
A comma may not be used to separate two independent clauses
...
Word choice
...

MEDIUM

9
...
Tense shift
...

C
...
The construction lacks a main verb
...
Faulty comparison
...

E
...
The phrase has greater popularity as is nonstandard English
...
E A
...
The phrase that begins Wandering through should modify he instead of
everything
...
Comma splice
...

C
...
The construction awkwardly separates the subject and the verb
...
Dangling participle
...

HARD

11
...
Idiom error
...

B
...
See A
...
Clumsy construction
...

E
...
Pronouns in prepositional phrases should be in the objective case
...

HARD

SECTION 2—IDENTIFYING SENTENCE ERRORS
12
...
Use as instead of than when pointing out likenesses
...
E No error
...
D Faulty comparison
...
Use last
year’s (senior class)
...
A Pronoun–antecedent agreement
...
The pronoun
they is plural
...

EASY

16
...
Use in instead of for
...
C Faulty parallelism
...
Use stole
instead of were stealing
...
E No error
...
B Verb form
...
Use had been bitten
...
D Pronoun–antecedent agreement
...
Davis is singular; the pronoun
their is plural
...

EASY

21
...
Use at instead of for
...
D Diction error
...
Use beautiful
...
E No error
...
B Subject–verb agreement
...
Use was
...
C Diction error
...

HARD

26
...
The subject following is singular; the verb have is plural
...

HARD

27
...
A pronoun that serves as the object of a preposition (without) must be in the
objective case
...

HARD

28
...
In standard English neither is paired with nor, not with or
...
B Subject–verb agreement
...
Use are
...
C Choice A implies that the essay’s purpose is to point out technological marvels of the
twentieth century
...

Choice B is similar to choice A and also contains an inappropriate colloquialism
...

Choice E summarizes the three areas discussed in the essay but indicates that we would be
better off without technological progress, an idea neither stated nor implied by the essay
...
C Choice A repeats energy unnecessarily and contains an incomplete comparison
...
Energy is cheaper than what? It also contains
an error in parallel construction
...
The word both and the phrase as well as are redundant
...
Use cheaper
...
B Although related to communications, the information contained in sentence 5 is not relevant
to the discussion of modern communications satellites
...
A Choice B is a sentence fragment
...

Choice D improperly shifts to the second person
...

HARD

34
...

Choice E is a remote possibility but is not justified by evidence in the essay
...
E Choice A unnecessarily repeats MRI and contains faulty parallelism
...

Choice C contains a dangling participle
...

Choice D has no discernible connection with the previous sentence
...
A B
...
Paying is compared to families, an illogical comparison
...
Faulty comparison
...

D
...
Paying is compared to families, an illogical comparison
...
Redundancy
...

EASY

2
...
Sentence fragment
...

B
...
Gwen Harper, the grammatical subject of the sentence, lacks a verb
...
Faulty coordination
...

E
...

EASY

3
...
Faulty coordination
...
The second
clause should be a subordinate clause
...
Mismatched sentence parts
...

D
...
Which has no logical meaning
...

E
...
A comma may not be used to separate two independent clauses
...
D A
...
High motivation is a noun phrase; significantly talented is an adjective
phrase
...

B
...
The word either and the phrase or else are redundant
...

C
...
The use of two successive subordinate clauses each beginning with who is
wordy
...
Faulty parallelism
...
The
phrase significant amounts of talent consists of an adjective, a noun, and a prepositional
phrase
...

MEDIUM

5
...
Idiom
...

C
...
The word there doesn’t refer to a specific place
...
Incomplete construction
...

E
...
The construction fails to conform to standard English idiom
...
B A
...
The sentence shifts from second to third person—from you to we
...
Shift in pronoun number
...

D
...
The pronouns shift from second person (you) to the impersonal
one
...
Awkwardness
...

EASY

7
...
Faulty parallelism
...
Having
Kelly Collins … is not parallel to the enthusiastic support…
...
Awkwardness
...

C
...
Coordinate elements in a series should be parallel in form
...

D
...
The phrase To be coordinated … has no grammatical relation
to the rest of the sentence
...
C A
...
The construction introduced … for the first time is redundant
...
Faulty pronoun reference
...

D
...
The sentence, cast in the past tense, shifts to the past perfect
...
Dangling modifier
...

HARD

9
...
Mismatched sentence parts
...

C
...
The construction fails to use standard English
...
Awkwardness
...

E
...
The pronoun he does not refer to a specific noun or other pronoun
...
B A
...
The pronoun it lacks a specific referent
...
Tense shift
...

D
...
The word it’s is a contraction of it is
...

E
...
The construction is repetitious and long-winded
...
D A
...
The pronoun their is plural; the antecedent person is
singular
...

B
...
In standard usage, nouns are defined by other nouns, not by clauses
...

C
...
Someone is singular; relax is plural
...
Pronoun reference
...

HARD

12
...
Comma splice
...

B
...
The pronoun it fails to refer to a specific noun or other pronoun
...
Sentence fragment
...

D
...
The pronoun its is singular; the antecedent lives is plural
...
D A
...
The pronoun it fails to refer to a specific noun or pronoun
...
Comma splice
...

C
...
The sentence, cast in the present tense, improperly shifts to the past tense
...
Idiom
...

MEDIUM

14
...
Faulty comparison
...
Use any other highway
...
Misplaced modifier
...

C
...
Use amount to refer to mass quantities; use number to refer to anything that
can be individually counted
...
Faulty comparison
...

HARD

Practice Test D
SECTION 1
ESSAY
TIME: 25 MINUTES
Directions: Plan and write an essay in response to the assigned topic
...
Present your
thoughts logically and precisely
...
A plain, natural writing style is probably best
...

Limit your essay to two sides of the lined paper provided
...
Write or print legibly because handwriting that’s hard or
impossible to read will decrease your score
...
AN ESSAY WRITTEN ON
ANOTHER TOPIC WILL BE SCORED “ZERO
...
Do not turn to another section of
the test
...

Think carefully about the following passage and the assignment below
...
Courtesy of the Internet, we seem to have immediate access
to anything that anyone could ever want to know
...
We prefer knowing to thinking
because knowing has more immediate value
...
Ideas are too airy, too impractical, too much work for little reward
...
Everyone talks information, usually personal information
...

Adapted from Neal Gabler, “The Elusive Big Idea,”
an opinion essay in the New York Times, August 14, 2011

Assignment: Does easy access to a virtually unlimited storehouse of facts and information tend to
devalue the importance of thinking and talking about big ideas? Plan and write an essay in which you
develop your point of view on this issue
...


Section 1
Essay
Time allowed: 25 minutes
Limit your essay to two pages
...
Write only inside the box
...

Do not proceed to Section 2 until the allotted time
for Section 1 has passed
...
Read each sentence carefully and
identify which of the five alternative versions most effectively and correctly expresses the meaning
of the underlined material
...
Choice A always repeats the original
...

EXAMPLE
ANSWER


My old Aunt Maud loves to cook, and
eating also
...
Although many young painters learned to paint from Rembrandt himself, when the master died,
no immediate successors have come into being
...
Newspaper editorials across the country argued brilliantly against the Supreme Court’s decision
on the death penalty
...
The country’s most important city for the arts and entertainment, tourists by the millions visit
New York each year
...
There is plenty of Thoreau’s practical advice about life, which every reader can benefit from in
his Walden
...

(B) In Thoreau’s Walden, they give the reader plenty of practical and beneficial advice about
life
...

(D) In Walden, Thoreau offers readers plenty of practical and beneficial advice about life
...

5
...

(A) considering that no state permits radioactive material transported on its roads or to bury it
inside its borders
(B) considering that no state permits neither radioactive material transported on its roads or
buried inside its borders
(C) because no state permits radioactive material transported on its roads or buried inside its
borders
(D) because no state will permit radioactive material not only to be carried on its roads but in
addition also buried inside its borders
(E) being that no state had permitted radioactive material to be carried on its roads or buried
inside its borders
6
...

(A) the letters Jefferson wrote to his son should be read
(B) Jefferson’s letters to his son should be read
(C) you should read the letters Jefferson wrote to his son
(D) you should read his letters to his son
(E) a person should read his letters to his son
7
...

(A) the noisiest park and also the most tranquil of them

(B) not only the noisiest park, but also more tranquil than any
(C) the noisiest park, at the same time it is the most tranquil park
(D) at once the noisiest and also the most tranquil of them
(E) the noisiest and yet the most tranquil of parks
8
...

(A) him, this surprised Sadie
(B) him, therefore Sadie was surprised
(C) him, surprising Sadie
(D) him, which surprises Sadie
(E) him, that was surprising to Sadie
9
...

(A) so has Columbia’s applicant pool risen steadily
(B) Columbia attracted applicants in steadily rising numbers
(C) Columbia is steadily gaining applicants in its pool
(D) and so then, for Columbia, a rising applicant pool has grown steadily
(E) and like them Columbia’s steadily rising pool of applicants
10
...

(A) suffers from a high crime rate, while it is
(B) although suffering from a high crime rate, is
(C) suffering from a high crime rate made it
(D) which suffers from a high crime rate, although it is
(E) whose rate of crime is high, makes it
11
...
C
...

(A) but they have designed it beautifully
(B) although it is beautifully designed
(C) yet it is beautiful in its design
(D) while being designed so beautifully
(E) and pleasing because of its beautiful design

Identifying Sentence Errors
Directions: The underlined and lettered parts of each sentence below may contain an error in
grammar, usage, word choice (diction), or expression (idiom)
...
Indicate your choice by filling in the corresponding
space on the answer sheet
...
Some sentences may contain
no error
...


EXAMPLE
Jill went

to the

of the hill in a

faster time

her friend, Jack
...


ANSWER

12
...


13
...
A number of the athletes

appealing to young readers
...


and catching trout and

16
...
Mike read a novel
chapter
18
...


the author,

a new structure,

high school and

a separate

character
...


the lovers row across the lake in a small boat, Catherine fears the approaching storm, but
they arrive on the opposite shore she

20
...


birth to Carolyn on October 20, 1993
...


the year, the park attracts visitors who come
the mountains,

during the

that this cereal not only
container
...


packed in a smaller

21
...

22
...

23
...

24
...


voters

their ballots for her
...
The orchestra played

throughout the wedding reception, that

barely possible

a conversation without shouting
...
High school jocks who wish to
being successful
28
...


that one is often faced

between

the ethical thing acting in one’s

everyone in the condo, and

the other residents because

own self-interest
...
The false alarm
she

in a building that burned down
...
Some parts of the passage need
improvement
...
The questions are about
revisions that might improve all or part of the passage’s organization, development, sentence
structure, or choice of words
...


Questions 30–35 refer to the following passage
...
[2]
Parents bring their kids to gawk at the caged creatures
...
[4] There must be better reasons, imprisoning wild animals is simply barbaric
...

[6] If someone is so dumb that they don’t know what a zebra looks like, they should look it up online
...
[8] Animals need to run free and live, but by putting them in
zoos we are disrupting and disturbing nature
...
[10] When the animals have
been at the zoo for a while they adopt a particular lifestyle
...
[12] They get used to that
...
[14] They would never survive
...
[16] After a few
generations the animals become totally different from their wild and free ancestors, and visitors to the
zoo see animals hardly resembling those living in their natural habitat
...
In context, which of the following is the best phrase to insert at the beginning of sentence 2
(reproduced below)?
Parents bring their kids to gawk at the caged creatures
...
In context, which of the following revisions would most improve sentence 4 (reproduced
below)?
There must be better reasons, imprisoning wild animals is simply barbaric
...

(B) Change “There must” to “There’s got to
...

(D) Substitute “terribly” for “simply
...

32
...

(B) A book or a Web site can give you more information about zebras and other animals
...

(D) Doesn’t everyone know what a zebra looks like, even little children?

(E) But if someone is so dumb that they don’t know what a zebra looks like, they should look it
up online
...
Which of the following reasons most accurately describes the author ’s purpose in choosing the
words underlined in sentence 7 (reproduced below)?
But humans have no right to pull animals from their natural environment and to seal their fate
forever behind a set of cold metal bars
...
Which of the following revisions of sentence 9 is the best topic sentence for the third
paragraph?
(A) Captivity alters the basic nature of animals
...

(C) Living conditions for animals in the zoo are ordinarily harsh and cruel
...

(E) Life in the zoo for animals is not a bowl of cherries
...
Which revision most effectively combines sentences 12, 13, and 14?
(A) Because they would never be able to survive again back in their natural environment, they
grow used to being fed
...

(C) Growing accustomed to that, placing them back in their native habitat and being unable to
survive on their own
...

(E) Being unable to survive back in their natural environment, the animals have grown
accustomed to regular feedings
...

Do not return to Section 1
...




SECTION 3
MULTIPLE-CHOICE QUESTIONS
TIME: 10 MINUTES

Improving Sentences
Directions: The underlined sentences and sentence parts below may contain errors in standard
English, including awkward or ambiguous expression, poor word choice (diction), incorrect
sentence structure, or faulty grammar, usage, and punctuation
...
Indicate your choice by filling in the corresponding space on the answer
sheet
...
Choose A if none of the other choices improves the
original sentence
...
Having a mother who plays in a symphony orchestra and a father who teaches music in high
school, the violin and the piano are two of the instruments that Rosie learned at an early age
...
When children change from little boys and girls to young men and women, ordinarily causing
them to become more self-sufficient and independent
...
Susan does not have absolute free will because what she does would have to be determined by
the culture and the environment
...
The students’ final Social Studies exam has been stolen from the teacher ’s desk; this situation
forcing them to take a make-up test on Saturday
...
Waste products from cutting lumber, such as wood chips and sawdust, are some of the
ingredients of waferboard panels used in residential construction
...
If you compare the number of NY Yankee pennants with the Boston Red Sox, you’ll see that the
Yankees are traditional winners
...
Today’s newspaper says that mathematics is far more popular among Japanese high school
students than among American students
...
In the nineteenth century, immigrants entered the United States with few limitations and
restrictions, but they have multiplied since then
...
In Moscow, famous composers, artists, and writers are buried in a special cemetery, and they
only must be Russian
...
The Boston Tea Party was a minor historical event with which the colonists either intended to
challenge or abolish the king’s unfair tax on imports
...
Should a college application essay be required, one ought to set aside a large block of time and
avoid writing it at the last minute
...
The atmosphere in the classroom changed when the snow started to fall outside and the teacher
could not get them to pay attention to the lesson after that
...
Of the four seasons in New England, Granny most loves the autumn, of which she finds the
mild days and cool nights especially appealing
...
Many countries punish citizens who speak out against the government, keeping the U
...

Commission on Human Rights very busy, mostly using torture and imprisonment
...
N
...
N
...
N
...
N
...
N
...

Do not return to Sections 1 or 2
...




Answer Key
SECTION 1—THE ESSAY
What follows are ideas for answering the essay question given in this practice test
...
Don’t be alarmed if your essay is much
different, however
...
Your head
is undoubtedly full of information, or facts, about, say, American history, from the landing of the
Pilgrims to the latest political scandal
...

Ideas, on the other hand, can’t be gathered as easily
...
For
decades segments of our population—among others, blacks and women—have struggled for equal
rights
...

The idea of equality, as any literate American will tell you, lies at the heart of a democratic way of
life
...
Think for a moment about your interactions with friends, classmates, teachers,
and family
...

On the other hand, as a consequence of your studies, reading, observation, and experience your
head may be bursting with great, often stimulating, ideas
...
Perhaps a life of thought and contemplation is what you envision for yourself
...
You could even begin your essay by pointing out
that Gabler ’s own provocative idea—that we live in a world increasingly bereft of ideas—contradicts
the very thesis he expounds
...
You may discern, for
instance, that many people are both well stocked with information and attuned to big ideas
...
For them reflecting on ideas is more than
simply a way to pass the time, for ideas often have utilitarian value
...
In fact, thoughtful
individuals with a knack for judiciously combining information and ideas might serve as models of
rationality in an increasingly jumbled world
...


SELF-SCORING GUIDE
Using this guide, rate yourself in each of these six categories
...

On the SAT itself, two readers will score your essay on a scale of 6 (high) to 1 (low), or zero if you
fail to write on the assigned topic
...

Remember that SAT essays are judged in relation to other essays written on the same topic
...
Because it is difficult to read your own essay with total objectivity, you might improve
the validity of your score by getting a second opinion about your essay from an informed friend or a
teacher
...
B
2
...
E
4
...
C
6
...
E
8
...
A
10
...
E
12
...
E
14
...
C
16
...
A
18
...
C
20
...
B
22
...
B
24
...
D
26
...
B
28
...
B
30
...
C
32
...
C
34
...
B


SECTION 3
1
...
C
3
...
D
5
...
E
7
...
D
9
...
E

11
...
C
13
...
D


Performance Evaluation Chart

Conversion Table
This table will give you an approximation of what your score would be if this practice test had been
an actual SAT Writing Test
...

For example, if your Multiple-Choice Raw Score was 35 and your Essay Subscore was 6, the table
indicates that your final score on the test would be approximately halfway between 500 and 710, or
600
...


SECTION 2—IMPROVING SENTENCES
TIP
Key to Levels of Difficulty

Rating
EASY
MEDIUM
HARD

Percentage of students
likely to answer correctly
>80%
>65%
<65%

1
...
Shift in verb tense
...

C
...
The subject coming is singular
...
Use was
...
Idiom error
...

E
...
The phrase not immediate successors is not standard English
...
A B
...
The grammatical subject (editorials) lacks a verb
...

C
...
The grammatical subject (decision) lacks a verb
...

D
...
The construction lacks a main verb
...

E
...
The grammatical subject (arguments) lacks a verb
...

EASY

3
...
Misplaced modifier
...

B
...
The phrase most important city should modify New York instead of
millions
...
Misplaced modifier
...

D
...
The phrase most important city should modify New York instead of
tourists
...
D A
...
The pronoun his refers to reader instead of to Thoreau
...
Pronoun reference
...

C
...
The phrase Reading Thoreau’s Walden should modify reader instead
of plenty
...
Sentence fragment
...

MEDIUM

5
...
Faulty parallelism
...
The verb
phrase transported on its roads is not parallel to to bury it inside its borders
B
...
The construction neither … or is not standard English
...

D
...
The phrases not only and but in addition also are redundant
...
Shift in verb tense
...

HARD

6
...
Sentence shift
...

B
...
The first clause begins in the second person (you)
...

D
...
The pronoun in his letters lacks a specific referent
...
Pronoun reference
...

HARD

7
...
Pronoun reference
...

B
...
The adjective noisiest and the phrase more tranquil than any should be
in parallel form
...

C
...
Commas may not be used to separate two independent clauses
...
Pronoun reference
...

HARD

8
...
Comma splice
...

B
...
A comma may not be used to separate two independent clauses
...
Tense shift
...

E
...
A comma may not be used to separate two independent clauses
...
A B
...
The sentence, cast in present perfect tense (has grown), shifts to the past
tense (attracted )
...
Faulty comparison
...

D
...
The construction is not in standard English
...
Sentence fragment
...

MEDIUM

10
...
Faulty subordination
...

C
...
The verb made has no logical relationship with the subject of the
sentence
...
Faulty subordination
...

E
...
The verb makes has no logical relationship with the subject of the
sentence
...
E A
...
The pronoun they fails to refer to a specific noun or other pronoun
...
Faulty parallelism
...
The
phrase at once frustrating is not parallel to although it is beautifully designed
...
Faulty parallelism
...
The
phrase at once frustrating is not parallel to yet it is beautiful in its design
...
Faulty parallelism
...
The
phrase at once frustrating is not parallel to while being designed so beautifully
...
B Subject–verb agreement
...
The verb are is plural
...

EASY

13
...

EASY

14
...
Standard usage requires the use of who rather than which to refer to people
...
C Faulty parallelism
...
Water sports
is not parallel to hiking … climbing … and catching
...
E No error
...
A Idiom error
...

HARD

18
...
The compound subject career … and … murder requires a plural
verb
...

HARD

19
...
The sentence is cast in the present tense
...
Use
says
...
D Idiom error
...
In
addition, being may not be used as a main verb without a helping verb, as in is being
...
B Shift in pronoun person
...

MEDIUM

22
...

EASY

23
...
In standard usage, the phrase is concur with
...
C Noun–verb agreement
...
Use invite
...
D Faulty comparison
...
Use Charles’ or Charles’ score
...
B Diction error
...
Use loudly
...
B Shift in number
...
Use athletes
...
D Idiom error
...

HARD

29
...
Pronouns in the objective case are used to refer to persons who receive an
action
...

HARD

SECTION 2—IMPROVING PARAGRAPHS
30
...
Not a good choice because the purpose of the sentence is to cite one of the reasons used to
justify keeping animals in zoos
...
The phrase is used to introduce an illustration of an idea stated in the previous sentence
...

C
...

D
...

E
...

EASY

31
...
In context, disagree is a poor word choice
...

B
...

C
...
It is the best answer
...
Because the sentence contains a comma splice, this revision fails to improve the sentence
appreciably
...
Because the sentence contains a comma splice, this revision fails to improve the sentence
appreciably
...
C A
...
It lacks a main verb
...
This sentence contradicts the idea stated in sentence 5
...
This choice develops the point stated in sentence 5
...

D
...

E
...

MEDIUM

33
...
This is not a good answer because it contains information that readers already know, and
highly charged language is not a good vehicle for passing along information
...
This is unrelated to the words in question
...
This is the best answer because the words are meant to shock and disturb the reader
...
This suggests that the author is trying to be objective, but the words in question are
anything but objective
...
This describes the purpose of the whole essay, not the particular words in question
...
A A
...
It is the best answer
...
Because this choice raises an issue not mentioned in the remainder of the paragraph, it is
not a good topic sentence
...
This contains an idea not discussed in the paragraph, which focuses on how animals
behave in captivity, not on living conditions at the zoo
...
This contains a dangling modifier
...

E
...
B A
...

B
...

C
...
The –ing forms of verbs (growing,
placing, being) may not be used as the main verb without a helping verb, as in was growing, is
placing, and so on
...
This is grammatically correct but stylistically awkward because the subject They is too far
removed from the verb would … survive
...
This is virtually meaningless because the cause-effect relationship has been reversed
...
D A
...
The phrase that begins Having a mother should modify Rosie instead
of the violin and the piano
...
Dangling participle
...

C
...
The phrase that begins Having a mother should modify Rosie instead
of two instruments
...
Awkwardness
...

MEDIUM

2
...
The –ing form of a verb (causing) may not be used as the main verb or a
clause or sentence without a helping verb, as in is causing, will be causing, and so on
...
Same as A
...
Mixed construction
...

E
...
The phrase to become ordinarily is not standard English
...
B A
...
The sentence, cast in the present tense (does not have), shifts to the future
conditional tense (would have to be)
...
Pronoun reference
...

D
...
The second clause of the compound sentence lacks a main verb
...

E
...
A comma may not be used to separate two independent clauses
...
D A
...
The construction beginning with this, if meant to be a complete
sentence, lacks a main verb
...

B
...
The sentence, cast in the present perfect tense (has been) shifts to the past tense
...
Faulty verb form
...

E
...
The pronoun it fails to refer to a specific noun or other pronoun
...
A B
...
Products is plural; is is singular
...
Subject–verb agreement
...

D
...
The construction lacks a main verb
...
Noun–verb agreement
...

MEDIUM

6
...
Faulty comparison
...

B
...
The number of … pennants is being compared to the Boston Red Sox,
an illogical comparison
...
Faulty comparison
...

D
...
The subject Making is singular; the verb show is plural
...
A B
...
Illogically, mathematics is compared to American students
...
Faulty parallelism
...

D
...
Although grammatically correct, the construction is wordy
...
Mixed construction
...

HARD

8
...
Pronoun reference
...

B
...
The pronoun they refers to immigrants when it is meant to refer to
restrictions
...
Incomplete construction
...

E
...
The phrase since multiplied has no grammatical connection with the
previous part of the sentence
...
E A
...
To make the sentence more effective, the second clause, because it
contains information of secondary importance, should be subordinated to the first clause
...
Misplaced modifier
...

C
...
The construction lacks a main verb
...
Sentence fragment
...

HARD

10
...
Misplaced modifier
...
Then add to before abolish
...
Idiom
...

C
...
Coordinate elements must be in parallel form
...
Use or abolishing
...
Parallelism error
...
A challenge is not in the
same form as or it abolished
...

HARD

11
...
Pronoun shift
...

C
...
Because the sentence is cast with the impersonal pronoun (one), the second
person pronoun you should not be used
...
Idiom error
...

E
...
The construction necessary for anyone to write is clumsily worded
...
C A
...
The pronoun them fails to refer to any specific noun or other pronoun
...
Comma splice
...

D
...
The pronoun them fails to refer to any specific noun or other pronoun
...
Incomplete construction
...

HARD

13
...
Idiom
...

C
...
The construction are of appeal is awkwardly worded
...
Sentence fragment
...

E
...
A comma may not be used to separate two independent clauses
...
D A
...
The phrase that begins mostly using torture should modify countries,
not U
...
Commission
...
Misplaced modifier
...

C
...
The construction beginning Punishing citizens should modify
countries, not U
...
Commission
...
Dangling modifier
...
N
...

HARD


Title: SAT - Barrons Writing Book
Description: The best SAT Barrons Writing book on the market, with detailed concepts explained totally to give out that 800 on the Writing Section